Pro Res Casebook Connect Quizzes ALL QUESTIONS

Réussis tes devoirs et examens dès maintenant avec Quizwiz!

Contingent fees in criminal cases. A client retained an attorney to represent him in two cases: a criminal case and a divorce case. The attorney required that the client pay a retainer fee for the family law case, which billed at the attorney's hourly rate. The attorney then arranged for the client to pay him based on a contingency fee for the criminal case. The attorney and the client both signed the combined contract, which detailed each fee arrangement for each case, and the attorney's representation began. Are the attorney's actions proper? A. No, as attorneys cannot charge a contingent fee for representing a defendant in a criminal case. B. No, because attorneys must have separate contracts for each separate case the attorney is handling for a client. C. Yes, because attorneys can charge hourly rates for domestic relations matters and can charge contingency, hourly, or flat fees for criminal cases. D. Yes, because attorneys are restricted from charging contingency fees only in domestic relations matters when the payment is contingent upon the securing of a divorce or upon the amount of alimony, support, or property settlement.

A is the correct answer. Rule 1.5(d)(2) states that a lawyer shall not enter into an arrangement for, charge, or collect a contingent fee for representing a defendant in a criminal case. Rule 1.5(d)(2) states that a lawyer may not charge "a contingent fee for representing a defendant in a criminal case." For example, a lawyer cannot offer to charge a fee only if the defendant obtains an acquittal. Representation of criminal defendants must involve either hourly billing or a flat fee for certain types of cases (e.g., charging $100 for challenging traffic tickets, regardless of the time involved). This rule is absolute and applies to the pretrial stage (plea bargain negotiations and discovery), trials, and appeals of criminal convictions. Note that Rule 1.5(d) states that a lawyer cannot even agree to charge a contingent fee in a criminal case, even if the lawyer ends up not charging the contingent fee or decides to waive the fee entirely at some point.

Which of the following is least likely to result in a finding of professional misconduct? A. An attorney in a criminal matter pursues a legal strategy with which the client disagrees. The client is convicted. B. An attorney in a criminal matter prevents her client from testifying in his own defense. C. Without consulting her client, an attorney in a civil matter refuses a low-ball settlement offer. D. None of the above

A. An attorney in a criminal matter pursues a legal strategy with which the client disagrees. The client is convicted. Rule 1.2 allocates the authority between client and lawyer. There are certain decisions that the client has the authority to make; however, legal strategy is typically recognized as within the authority of the attorney.

Which of the following uses of social media is permissible under the Model Rules of Professional Conduct? A. As an investigative tool to research publicly accessible information B. As a means to solicit business from potential clients C. To contact jurors to get information about their deliberations D. To post updates on trial happenings in real time

A. As an investigative tool to research publicly accessible information

An attorney is a fifth-year associate at a large firm, hoping to make partner in the next two or three years. She supervises the first-year associates at the firm. She learns that the most recently hired associate recently shredded some evidence that would have undermined a client's case, and then told the judge and opposing party that the missing documents had been in a briefcase that went missing when a burglar broke into the associate's car. The attorney knows this is not true and discusses it with the senior litigation partner, who finds the story amusing. Neither reports the associate's deception to the judge or opposing party. Which of the following statements is true regarding this situation? A. Both the attorney and the litigation partner are subject to discipline for not taking action to correct the associate's false statements and misconduct. B. The attorney is subject to sanctions, as she was directly supervising the associate, but the senior litigation partner was not involved, did not know about it at the time, and has no responsibility in the matter. C. The associate is subject to discipline, but neither the attorney nor the senior litigation partner would be subject to discipline, as they were not aware of the misconduct until after the fact. D. The senior litigation partner is subject to discipline because he has a responsibility to take reasonable measures to ensure that everyone in the firm complies with their ethical duties, but the attorney is not subject to discipline, as she is merely an associate at the firm.

A. Both the attorney and the litigation partner are subject to discipline for not taking action to correct the associate's false statements and misconduct. Direct supervisors and managing lawyers have a duty to intervene if they acquire knowledge about an associate's misconduct when the consequences are avoidable. Although partners have indirect responsibility for all work in the firm, the partners who manage a particular area, like the litigation partner in this question, would have a greater supervisory responsibility for the other lawyers in that practice group. The supervisor or managing partner will be subject to discipline if he fails to take reasonable remedial action, such as notifying opposing counsel or the judge, after learning of the misconduct when the consequences are avoidable.

An attorney is representing the defendant in a personal injury trial between a celebrity plaintiff and a famous hotel, where the plaintiff claims to have suffered injuries due to unsafe conditions. On his way into the courthouse on the day of jury selection, reporters gather around the attorney hoping for comments. The attorney explains that his client has already made renovations to the hotel to ensure that no accidents happen in the future, even though they do not admit liability in the present case. He also explains that if his client loses, his insurance company will simply pay the damages, and lawsuits like this make everyone's insurance premiums go up. The client had previously given the attorney permission to talk to the media. Opposing counsel is standing nearby waiting for his turn to talk, and he expresses no objection to the first attorney giving interviews like this, or to the lawyer's comments. Are the attorney's statements proper? A. No, it violates the Model Rules for a lawyer to make public statements about information that the lawyer knows or reasonably should know is likely to be inadmissible as evidence in a trial and that would, if disclosed, create a substantial risk of prejudicing an impartial trial. B. Yes, because the other lawyer is present and did not object to the comments at the time, and the client has consented to the lawyer's media communications. C. No, because in civil trials a defendant's lawyer should not tell the press that his client denies liability in the case. D. Yes, the rules about trial publicity explicitly permit lawyers to talk about defenses in the case, and the client's mitigation efforts and public policy concerns over skyrocketing insurance rates could be the defendant's main arguments to the jury.

A. No, it violates the Model Rules for a lawyer to make public statements about information that the lawyer knows or reasonably should know is likely to be inadmissible as evidence in a trial and that would, if disclosed, create a substantial risk of prejudicing an impartial trial. Comment 5 to Rule 3.6 delineates several types of statements that "are more likely than not to have a material prejudicial effect on a proceeding, particularly when they refer to a civil matter triable to a jury, a criminal matter, or any other proceeding that could result in incarceration." In other words, these subjects are presumptively prejudicial for purposes of Rule 3.6 and the prohibition on media statements about litigation. The subjects are specific enough to be especially well-suited for multiple-choice questions, meaning they are good to study in preparation for the MPRE. The fifth subsection in Comment 5 specifies, "[I]nformation that the lawyer knows or reasonably should know is likely to be inadmissible as evidence in trial and that would, if disclosed, create a substantial risk of prejudicing an impartial trial." In the facts above, the lawyer is discussing information (remedial measures taken after an accident) that would normally be inadmissible in a personal injury trial. The fact that the other lawyer does not object does not change the fact that this constitutes a violation of the rule. It undermines the rules about admissible evidence if lawyers can simply disseminate inadmissible evidence through the media before trial.

An attorney is a second-year associate at a law firm with no supervisory responsibilities. He learns that another second-year associate is working on a case in which the client is suing a company that the other associate used to represent at his previous firm, and the attorney suspects it is a substantially related matter. The firm has done nothing to screen the other associate from the matter. No one ever discusses it with the attorney, and the attorney does not know all the facts of the situation. Later, the client sues the firm for malpractice due to the conflict of interest and reports the matter to the state disciplinary authority. Which of the following is true regarding the attorney's involvement in the situation? A. The attorney does not have disciplinary liability for the conduct of the other associate, because he is neither a partner nor in a supervisory position and did not participate in the violations directly. B. The attorney is subject to discipline because he had an affirmative duty to inquire about the potential conflict and the lack of screening of this other associate. C. The attorney is subject to discipline because the Rules of Professional Conduct impute violations of any lawyer in the firm to all other lawyers in the firm. D. The attorney does not have disciplinary liability in this matter because the Rules of Professional Conduct do not impute conflicts of interest to other lawyers in the same firm just because the firm failed to screen one associate with a possible conflict with former clients.

A. The attorney does not have disciplinary liability for the conduct of the other associate, because he is neither a partner nor in a supervisory position and did not participate in the violations directly. Partners, managers, and supervisors are required to make reasonable efforts to ensure a firm's compliance with the Rules of Professional Conduct. These individuals must establish internal policies and procedures, such as those used to detect and resolve conflicts of interest, to ensure compliance. As you may recall, Rule 1.9 specifically prohibits a lawyer who has formerly represented a client in a matter from representing another person in the same or a substantially related matter in which the new client's interests are materially adverse to the interests of the former client without informed consent, confirmed in writing. In this question, the associate's conflict of interest with his former client imputes to all lawyers in the same firm, unless the firm screened him from representation and received written informed consent from the former client. Otherwise, the firm's representation of the client would result in a violation of the rules. The partners, managers, and supervisors who failed to establish policies and procedures to avoid the conflict of interest would be subject to disciplinary action.

An attorney is an associate in a litigation firm representing plaintiffs. In her current case, her supervising partner instructs her to assert that the defendant had an affirmative statutory duty to protect the plaintiff's interests, even though the attorney can find no statute to support this assertion. The attorney has brought this to the attention of her supervising partner, who rebuked her for questioning his authority and insisted that she do as he said. He assures her that the defendants will settle before trial anyway, so the bogus claim merely gives some psychological leverage during settlement negotiations, and it cannot do any harm. Moreover, the partner says that the attorney may not last long at the firm if she cannot follow instructions, which could have been a threat of termination. At a preliminary hearing, however, the judge confronts the attorney about the unsupportable claim, and she concedes that no statutory duty exists. The judge is irate and considers reporting the attorney to the state bar disciplinary authority. Which of the following is correct regarding the attorney's situation? A. The attorney is responsible for asserting a frivolous claim, even though her supervising partner insisted that she do it and threatened her with termination. B. The attorney is not responsible for asserting a frivolous claim, because her supervising partner insisted that she do it and threatened her with termination. C. Neither the attorney nor the partner would be subject to discipline so long as the case settles before trial and the bogus claim about statutory duties was not the sole basis for their complaint. D. If the partner had terminated the attorney for refusing to assert the frivolous claim, the state bar disciplinary authority would have compelled the firm to reinstate her.

A. The attorney is responsible for asserting a frivolous claim, even though her supervising partner insisted that she do it and threatened her with termination. Although the attorney is acting under the supervising partner's directions and is induced by his threats, she can be held responsible for asserting the claim because she knows it is frivolous. Rule 3.1 prohibits a lawyer from asserting a claim that has no basis in law and fact. The attorney and the supervising partner know that the defendant has no legal duty to protect the plaintiff's interests. The supervising partner's tactics are an abuse of legal process and violation of Rule 3.1. The attorney is not relieved of responsibility for complying with the rules because she is following the supervising partner's orders. Therefore, the attorney remains subject to discipline for asserting a claim that has no basis in law or fact.

An attorney is a fifth-year associate at a large firm and is responsible for supervising the work of a first-year associate. The attorney, however, now spends most of his time in Singapore, trying to open a satellite office for the firm there to service one of its major corporate clients. He has not inquired into the associate's compliance with the Rules of Professional Conduct in over eighteen months, as they mostly communicate by e-mail regarding pending cases and assignments. To the best of his knowledge, though, the attorney believes the associate is following the rules, and he knows that the associate has attended two Legal Ethics CLE courses in the last year. Unbeknownst to the attorney, the new associate has been overbilling hours and has been neglecting certain client matters. Which of the following is true regarding the attorney's situation? A. The attorney is subject to discipline as a lawyer having direct supervisory authority over another lawyer who failed to make reasonable efforts to ensure that the other lawyer conforms to the Rules of Professional Conduct, even though there was no direction, ratification, or knowledge of the violation. B. The attorney is subject to discipline for effectively ratifying the associate's violations through his neglect of his supervisory role. C. The attorney is not subject to discipline because there was no direction, ratification, or knowledge of the violation. D. The attorney is subject to discipline because the Rules of Professional Conduct impute violations of any lawyer in the firm to all other lawyers in the firm.

A. The attorney is subject to discipline as a lawyer having direct supervisory authority over another lawyer who failed to make reasonable efforts to ensure that the other lawyer conforms to the Rules of Professional Conduct, even though there was no direction, ratification, or knowledge of the violation. Under Rule 5.1(b), supervising attorneys must make reasonable efforts to ensure the lawyers under their supervision comply with the Rules of Professional Conduct. A supervising attorney may be subject to discipline for neglecting his responsibilities if a lawyer under his supervision violates the rules. In addition, under Rule 5.1(c), the supervising attorney would be responsible for the other lawyer's violation if the supervisor ordered or, with knowledge, ratified the misconduct. The two potential violations for supervising lawyers are not mutually exclusive. The adequacy of a supervisor's efforts may depend on the firm's size, structure, and the nature of its practice. Periodic review and informal supervision may suffice in a smaller firm, but larger firms typically require supervision that is more formal. Any size firm may supplement its own ethics training with Continuing Legal Education courses.

An attorney is a partner in a medium-size firm. Another partner at the firm, the managing partner, is responsible for implementing policies and procedures to detect and resolve conflicts of interest, to account for client funds and property, to identify dates by which actions must occur in pending matters, and to ensure that inexperienced lawyers receive proper supervision. The managing partner, however, now spends most of his time in Singapore, trying to open a satellite office for the firm there to service one of its major corporate clients. The managing partner is rarely at the home office and has completely neglected the implementation of ethical policies in the firm, so that minimal safeguards or procedures are in place. One of the new associates has committed several serious violations of professional responsibilities in the last few months, including an egregious conflict-of-interest problem and several missed deadlines for filing responsive pleadings. The attorney knew nothing about the violations and was not directly supervising the associate, and tries not to meddle in any of the managing partner's responsibilities, including the implementation of ethical policies and procedures. Which of the following is correct? A. The attorney is subject to discipline as a partner in the firm for failing to make reasonable efforts to ensure that the firm has in effect measures giving reasonable assurance that all lawyers in the firm conform to the Rules of Professional Conduct. B. The attorney is not subject to discipline because it was the managing partner's job to implement measures giving reasonable assurance that all lawyers in the firm conform to the Rules of Professional Conduct. C. The attorney is not subject to discipline because he did not directly supervise the associate who violated the Rules of Professional Conduct. D. The attorney is subject to discipline because the Rules of Professional Conduct impute violations of any lawyer in the firm to all other lawyers in the firm.

A. The attorney is subject to discipline as a partner in the firm for failing to make reasonable efforts to ensure that the firm has in effect measures giving reasonable assurance that all lawyers in the firm conform to the Rules of Professional Conduct. Rule 5.1(a) delegates responsibility to the lawyers in the firm who have managerial authority over the professional work of the firm such as partners, shareholders, and members of other associations authorized to practice law. All partners share the responsibility for establishing internal procedures and policies to detect and resolve conflicts of interest, to account for client funds and property, to identify dates by which actions must occur in pending matters, and to ensure that inexperienced lawyers receive proper supervision. Additionally, all partners must make reasonable efforts to ensure the entire firm's compliance with the rules. The rules do not distinguish between managing partners and regular partners for purposes of compliance with Rule 5.1.

Former clients and new clients. An attorney worked at Big Firm for five years, and she specialized in real estate. During the last three years, the attorney has worked mostly on Conglomerate Corporation's properties and disputes that arose related to them. Shortly before leaving Big Firm, the attorney handled a matter for Conglomerate Corporation related to the construction of a new apartment complex. The attorney now works at Boutique Firm, and a prospective client explains during an initial consultation that she needs to sue Conglomerate Corporation over a disputed right of way through the very property that holds the new apartment complex—one of the last matters that she handled for Conglomerate. Conglomerate is still using Big Firm for its legal representation. Would it be proper for the attorney to represent the prospective client in this matter against Conglomerate Corporation? A. The attorney may represent the new prospective client if Conglomerate Corporation consents in writing to her doing so. B. The attorney may not represent the new prospective client in the same or substantially the same matter unless Conglomerate Corporation consents in writing. C. The attorney may not represent the new prospective client in the same or substantially the same matter unless the new prospective client consents in writing. D. The attorney may represent the new prospective client because she no longer works for Big Firm and no one in her new firm would have confidential information about Conglomerate Corporation.

A. The attorney may represent the new prospective client if Conglomerate Corporation consents in writing to her doing so. A is correct because even though the attorney will now have an adversarial role against her former client, as long as the former client consents in writing (waives the conflict), and the attorney reasonably believes she can provide fair representation and will not have to reveal confidential information from her former client, she may proceed with the representation.

The attorney is an associate at a small firm, and her supervising partner instructs her to draft pleadings in a case for a client. The supervising partner knows that the statute of limitations has already run on the claim, and that the client had little or no factual evidence to support the claim in any case. The partner believes the opposing party will want to settle the claim quickly for a modest sum and will not bother to investigate issues such as the statute of limitations or the factual support for either side. The attorney follows the partner's instructions and drafts the pleadings, without checking the statute of limitations for this specific claim or conducting her own investigation into the facts of the case. Opposing counsel, however, is upset over the frivolous claim and reports the attorney to the state bar. Which of the following is correct regarding the attorney's situation? A. The attorney should not be responsible for asserting a frivolous claim, and the fact that she was just following orders could support her defense that she was unaware that the claim was frivolous. B. The attorney is responsible for asserting a frivolous claim, even though her supervising partner insisted that she do it. C. Neither the attorney nor the partner would be subject to discipline so long as the case settles before trial. D. If the partner had terminated the attorney for refusing to assert the frivolous claim, the state bar disciplinary authority would have compelled the firm to reinstate her.

A. The attorney should not be responsible for asserting a frivolous claim, and the fact that she was just following orders could support her defense that she was unaware that the claim was frivolous. Before filing an action, all lawyers should inform themselves of the facts of the case and applicable law. Rule 3.1 prohibits a lawyer from bringing a frivolous claim, which is one with no basis in law or fact. While a lawyer should use the law for the client's full benefit, he should never abuse legal procedures to motivate the opposing party to settle like the partner in this question. The state disciplinary authority considers the lawyer's knowledge, or lack thereof, in determining whether to impose disciplinary liability. Since the partner knew the claim was frivolous, he will be subject to discipline actions or sanctions. On the other hand, the attorney did not know the claim was frivolous and was following the partner's orders so she is probably not responsible.

A state legislature enacted a statute governing the licensing of attorneys and discipline for practitioners. The preamble to the statute asserted "field preemption" over the regulation of lawyers in that jurisdiction, thereby abolishing all prior rules and codes of the state bar. An attorney came under discipline under the new law and contested the legal validity of the enactment itself. What should be the result? A. The court will hold the law invalid because the judiciary has inherent power to regulate the attorneys who practice in its courts. B. The court will reject the lawyer's claim about the law's invalidity and will uphold the sanction. C. The court will refuse to hear the case because it now lacks jurisdiction over the matter. D. The court will certify a question to the legislature to seek its decision in the matter.

A. The court will hold the law invalid because the judiciary has inherent power to regulate the attorneys who practice in its courts. The judiciary has inherent power (under common law, and probably under the separation of powers doctrine as well) to regulate the lawyers who appear in the courts representing others. Field preemption by the legislature would not survive a court challenge. Of course, legislatures have made some incursions into the regulation of lawyers. There are statutes (especially federal) permitting non-lawyers to represent certain types of parties, especially in administrative proceedings (Social Security hearings, Veterans' Benefits, Patent Office, and Immigration). In addition, some statutes impose some requirements for admission to the bar, and reporting requirements by lawyers.

An attorney is a partner in a newer law firm that has no effective measures in place to ensure that lawyers in the firm conform to the Rules of Professional Conduct. An associate at the firm violates the rules, and the state bar investigates the policies and procedures in place at the firm. The state disciplinary authority has determined that the attorney is subject to discipline for his failure to take reasonable measures to ensure conformity with the rules. Because of this determination and the subsequent sanction, which of the following is true? A. The determination of an ethical violation does not automatically mean that the attorney would be civilly or criminally liable. B. The fact that the state bar found the attorney guilty of a violation of the rules, and imposed a sanction, means that the attorney is automatically liable in any legal malpractice action related to the violation. C. The fact that the state bar found the attorney guilty of a violation of the rules, and imposed a sanction, means the attorney is automatically guilty in any criminal prosecution related to the violation. D. The fact that the state bar already imposed a sanction for the violation precludes being subject to damages in a malpractice action or criminal sanctions in a criminal prosecution related to the same violation, due to the double jeopardy rules.

A. The determination of an ethical violation does not automatically mean that the attorney would be civilly or criminally liable. The Preamble to the Model Rules flatly states, "They are not designed to be a basis for civil liability." The Restatement takes the same position. Both acknowledge, of course, that a violation could be relevant for deciding whether a lawyer should be liable in tort, but it is not automatic. The conduct in this problem clearly violates the Rules of Professional Conduct, but that does not guarantee that the violators will face tort liability. There are important differences between a civil suit and an enforcement action that could produce different results even when analyzing the same activity. A malpractice action includes showing damages (actual injury), causation, and must address various affirmative defenses, mitigating factors, contributory negligence by the plaintiff, and so on. In addition, hearsay evidence is admissible in an administrative hearing (like most disciplinary proceedings), but not in court, so the strongest evidence of the lawyer's violation could be inadmissible in a malpractice case, even though it is admissible at the disciplinary hearing.

A partner gives an associate the typed notes from a previous client interview conducted by the partner, and the associate has the task of drafting a complaint for a personal injury lawsuit based on the allegations in the notes. The associate has no direct contact with the client, and does not really have any way to verify whether the notes represent everything discussed in the interview (the notes are not a transcript) or whether the allegations are factually accurate, truthful, or tell the complete story. The associate completes the task as assigned, drafting the pleadings based on the notes. The associate then submits the drafted complaint to the partner for review. Later, the pleadings turn out to be frivolous, based on complete falsehoods. Which of the following is true regarding the associate attorney's role in drafting the complaint? A. The lack of opportunity for the associate to investigate or verify facts on her own will be a relevant factor for the state disciplinary authority in deciding whether to discipline the associate. B. The lack of opportunity for the associate to investigate or verify facts on her own will not be a relevant factor for the state disciplinary authority in deciding whether to discipline the associate. C. The client will be subject to discipline, but not her lawyers, who merely took her at her word and filed complaints based on what she told the lawyers. D. The associate will be subject to discipline for drafting a complaint based on interview notes rather than a transcript of the interview or a notarized affidavit.

A. The lack of opportunity for the associate to investigate or verify facts on her own will be a relevant factor for the state disciplinary authority in deciding whether to discipline the associate. Rule 3.1 prohibits a lawyer from asserting a claim that has no basis in fact or law. Before asserting a claim, a lawyer must be informed of the facts of the case and applicable law to prevent a frivolous lawsuit and violation of Rule 3.1. Since the associate did not have contact with the client, she is relying upon the partner's information to draft the pleadings. The format of the information, note or transcript, is not determinative. Rather, the associate attorney's lack of opportunity to verify the facts has greater relevance. Since the associate did not have the opportunity to verify the facts, she may not have the knowledge required to render her conduct a violation of the rules.

An attorney is a new associate at a law firm, and the managing partner assigns her a new case, in which the firm will represent two co-plaintiffs in a personal injury case. The attorney is concerned that a conflict of interest could arise between the two plaintiffs and suggests that the firm should represent only one of them. When she discusses this with the managing partner, the managing partner disagrees, because the interests of the two plaintiffs seem perfectly aligned, and they can have each sign an informed consent form waiving the conflict up front. Both admit the question is a close one in terms of the ethical rules for conflicts of interest, but the managing partner insists that they proceed. Which of the following is true regarding this situation? A. The supervisor's reasonable resolution of the question should protect the subordinate professionally if the resolution subsequently faces a challenge. B. The attorney has a duty to follow her own judgment about her ethical obligations to clients under the Rules of Professional Conduct, even if that means ignoring the managing partner's instructions. C. The fact that the two discussed the potential conflict at length will help shield both of them from any professional repercussions if they turn out to be wrong later. D. The answer depends on whether the firm will receive a contingent fee if they prevail, as there is a strict prohibition on representing co-plaintiffs in a contingent fee case.

A. The supervisor's reasonable resolution of the question should protect the subordinate professionally if the resolution subsequently faces a challenge. Under Rule 5.2(b), a subordinate lawyer does not violate the rules if he acts in accordance with a supervisory lawyer's reasonable resolution of an arguable question of a professional duty. This rule allows the lawyers to take a position and consistent course of action. Both the supervisor and subordinate lawyer are responsible for an ethical violation when the resolution is certain. When the ethical dilemma creates ambiguity, the supervisor assumes the responsibility for the decision if the resolution is challenged. This question illustrates the disciplinary liability of a supervisor and subordinate lawyer when faced with an arguable question of a professional duty. The ethical dilemma that the attorney and managing partner need to resolve involves a concurrent conflict of interest. The relevant rule, Rule 1.7, permits the attorney's representation of both co-plaintiffs if she believes that she can provide competent and diligent representation to both; the law does not prohibit the representation; the clients' interests are aligned; and each client gives informed consent in writing. While both lawyers admit the representation is questionable, the managing partner reasonably believes the attorney's representation of the co-plaintiffs is acceptable and the attorney may act upon his decision. The managing partner assumes the responsibility of making the decision because the resolution is uncertain. Meanwhile, the attorney is relieved of responsibility for the decision by Rule 5.2 if the decision is subsequently challenged.

An attorney is a partner in a newer law firm that has no effective measures in place to ensure that lawyers in the firm conform to the Rules of Professional Conduct. An associate at the firm violates the rules, and the state bar investigates the policies and procedures in place at the firm. The state disciplinary authority has determined that the first attorney is subject to discipline for his failure to take reasonable measures to ensure conformity with the rules. Because of this determination and the subsequent sanction, which of the following is true? A. Whether a lawyer may be liable civilly or criminally for another lawyer's conduct is a question of law beyond the scope of the rules; the determination of a violation does not automatically mean that the partner attorney would be civilly or criminally liable. B. The fact that the state bar found the partner attorney guilty of a violation of the rules, and imposed a sanction, means the attorney is automatically liable in any legal malpractice action related to the violation. C. The fact that the state bar found the partner attorney guilty of a violation of the rules, and imposed a sanction, means the attorney is automatically guilty in any criminal prosecution related to the violation. D. The fact that the state bar already imposed a sanction for the violation precludes being subject to damages in a malpractice action or criminal sanctions in a criminal prosecution related to the same violation, due to the double jeopardy rules.

A. Whether a lawyer may be liable civilly or criminally for another lawyer's conduct is a question of law beyond the scope of the rules; the determination of a violation does not automatically mean that the partner attorney would be civilly or criminally liable. This question illustrates the scope of the rules and the impact a violation has on a lawyer's civil and criminal liability. The rules provide a framework for the ethical practice of law, guidance for lawyers, and structure for disciplinary agencies in regulating lawyers' conduct. A violation of a rule by itself is not sufficient to create a cause of action against a lawyer. Additionally, it does not create a presumption that the lawyer has breached a duty imposed by law. A lawyer's failure to abide by the rules invokes the disciplinary process and is not a basis for criminal or civil liability.

A mutually beneficial arrangement. An attorney made an informal agreement with a physician that they would refer clients to each other when the situation seemed appropriate. They did not pay each other any money for referrals, but the relationship was explicitly reciprocal—the attorney referred patients who needed medical examinations to the physician, and when the physician had patients needing legal representation, he referred them to the attorney. The relationship was not explicitly exclusive—each was free to refer clients to others—but it happened that neither had similar reciprocal relationships with anyone else. They always informed their clients when making such referrals that they had a reciprocal relationship. Is such an arrangement proper? A. Yes, a lawyer may agree to refer clients to another lawyer or a non-lawyer professional, in return for the undertaking of that person to refer clients or customers to the lawyer, as long as clients are aware and the relationship is not exclusive. B. Yes, because the agreement is informal, not a written contract. C. No, because a lawyer may not agree to refer clients to another lawyer or a non-lawyer professional, in return for the undertaking of that person to refer clients or customers to the lawyer. D. No, because the relationship described here is de facto exclusive, even if they have not agreed specifically to keep the relationship exclusive.

A. Yes, a lawyer may agree to refer clients to another lawyer or a non-lawyer professional, in return for the undertaking of that person to refer clients or customers to the lawyer, as long as clients are aware and the relationship is not exclusive. Rule 7.2(b)(4) restricts reciprocal referral arrangements between lawyers and other professionals. Such arrangements are proper only if (1) the reciprocal referral agreement is not exclusive, and (2) the client is informed of the existence and nature of the agreement. In this question, it appears that the arrangement meets both of these criteria. The only point of ambiguity is that the two professionals here have not in fact referred clients to anyone else. Nevertheless, the fact that each of them has that option—neither is obligated to make referrals exclusively to the other—brings this arrangement properly under the rule.

An attorney agreed to represent a plaintiff in a personal injury lawsuit, and the next day agreed to represent a defendant in litigation where the defendant faces vicarious liability. Only after the attorney has conducted some investigation of the case, and has obtained confidential information from each client, does the attorney discover that the plaintiff client is actually suing another of the attorney's clients, under a theory of vicarious liability. The two clients are actually adverse parties in the same litigation. Must the attorney withdraw from representing both clients? A. Yes, a lawyer shall withdraw from the representation of a client if the representation will result in violation of the rules of professional conduct. B. Yes, the lawyer must withdraw unless both clients consent to the conflict of interest. C. No, the lawyer may withdraw, but withdrawal is optional and not mandatory. D. No, the lawyer may not withdraw once litigation is underway, regardless of the conflict of interest.

A. Yes, a lawyer shall withdraw from the representation of a client if the representation will result in violation of the rules of professional conduct. A is the best answer. Rule 1.16(a) requires lawyers to decline or withdraw from representation whenever continuation of the representation would result in a rule violation. A conflict of interest, especially between two parties in the same litigation, is a serious violation of the Model Rules, so the lawyer cannot continue.

Client of the lawyer who left. An attorney represented Small Business Associates while working at Big Firm, her first law firm after law school. When the attorney did not make partner at the firm, she ended her employment there and started her own new firm. The attorney took some of her clients with her, including Small Business Associates, whom she continues to represent. Big Firm no longer has Small Business Associates as a client. Big Firm then agrees to represent Conglomerate Corporation in a trademark infringement case against Small Business Associates, the first such case that the latter has ever faced. Can Big Firm represent Conglomerate in a case against its former client, Small Business Associates? A. Yes, as long as the matter is not the same or substantially related to that in which the attorney formerly represented the client; and no lawyer remaining in the firm has confidential information about Small Business Associates from when the attorney represented them at that firm. B. Yes, because otherwise the disqualification of the firm would constitute an agreement not to provide representation to specific clients in the future, which would violate the Rules of Professional Conduct. C. No, unless the attorney's new firm screens her from the litigation according to the procedures set forth in the Rules of Professional Conduct. D. No, unless Conglomerate provides written informed consent to the potential conflict of interest.

A. Yes, as long as the matter is not the same or substantially related to that in which the attorney formerly represented the client; and no lawyer remaining in the firm has confidential information about Small Business Associates from when the attorney represented them at that firm. A is correct because the trademark infringement claim is unrelated to any of Big Firm's prior work performed for Small Business Associates and none of Small Business Associates' confidential information remains at Big Firm.

A client, age eighteen, is facing criminal charges of sex with a minor, based on his sexual relationship with his thirteen-year-old girlfriend, who lives in the same tenement building. The relevant statute has strict liability for perpetrators—that is, no mens rea or scienter element—and places the victim's age cutoff for the most serious grade of felony at age fourteen. It is indisputable in the case that the defendant had a sexual relationship with the victim when she was thirteen, but the victim claims she wanted the relationship and willingly consented to the sexual contact with her boyfriend. A state psychologist examined the victim and included in his report that she was emotionally mature for her age and was making relationship decisions in the same way as an adult. Even though the attorney is certain that the trial court will convict the client, he believes there is a slight chance that he could convince an appellate court to take a loose view of the age-of-consent provision in the statute, either on substantive due process grounds or simply as a matter of progressive statutory construction. The attorney believes that many thirteen-year-olds, and even younger, are sexually active nowadays and that the criminal laws should reflect the changing values of society. The attorney agrees, therefore, to take the client's case and to use it as a test case to try to change the law of sexual consent in the appellate courts. Is it proper for the attorney to make a defense in a criminal case that goes against the clear statutory verbiage and established case precedent? A. Yes, because a claim or argument is not frivolous if the lawyer is making a good-faith argument for modification or reversal of existing law. B. Yes, because the statute has no mens rea requirement, but is a strict liability crime. C. No, because a lawyer shall not bring or defend a proceeding, or assert or controvert an issue therein, unless there is a basis in law and fact for doing so that is not frivolous. D. No, because the unlikelihood that the lawyer will win on appeal, in contradiction to the plain language of the statute, makes the lawyer's fee in the case a contingent fee, which is not permissible in a criminal case.

A. Yes, because a claim or argument is not frivolous if the lawyer is making a good-faith argument for modification or reversal of existing law. Rule 3.1 says that a "good faith basis in law or fact" should include "a good faith argument for an extension, modification, or reversal of existing law." Several landmark decisions in our legal history began with a lawyer asking a court to declare an existing statute unconstitutional, or to overturn prior precedent on a particular question. Asking the court to break with precedent or invalidate a statute is certainly an uphill battle, but it does not make a claim "frivolous" for purposes of the Model Rules.

An attorney is a litigator and finds it helpful to talk to jurors after a trial concludes to see what they thought about the performance of the lawyers in the case. Assuming the judge has not forbidden talking to jurors and the jurors are willing to talk to him, is it proper for the attorney to have conversations with jurors in their homes, a week after the trial? A. Yes, because a lawyer may communicate with a juror after the discharge of the jury and must respect the desire of the juror not to talk with the lawyer. B. Yes, so long as the lawyer does not talk about the merits of the case, the evidence, or the credibility of the witnesses. C. No, because a lawyer may talk to jurors after discharge only with opposing counsel present and while they are still at the courthouse. D. No, because a lawyer may not communicate ex parte with a juror, without an express authorization by the judge.

A. Yes, because a lawyer may communicate with a juror after the discharge of the jury and must respect the desire of the juror not to talk with the lawyer. Comment 3 for Rule 3.5 contemplates that lawyers will often take the opportunity to talk to jurors after the conclusion of a case, which is normally permissible. The prohibition against talking to jurors applies during the proceedings; after the discharge of the jury, it is usually proper for the lawyers in the case to interview them or chat with them. Some lawyers hope to receive constructive feedback about the trial—what arguments or witnesses seemed most convincing, what the lawyers could have done better, and so on. This can help lawyers improve their own courtroom performance, but also understand how jurors think. Other lawyers see it as a public relations opportunity—jurors who liked one of the lawyers may refer that lawyer to their friends or relatives later on. Still others are hoping to discover information that would furnish the basis for an appeal. As mentioned above, Rule 3.5(b) imposes some restraints on lawyers during this posttrial discussion with jurors. First, judges can order the lawyers to refrain from talking to the jurors in a specific case, and it is possible that some state statute could bar such conversations, at least in specific types of cases. Second, jurors never have any obligation to talk to the lawyers, and if a juror does not wish to talk to a lawyer (and says as much), the lawyer must refrain from pursuing any further contact. Some jurors feel uncomfortable talking with lawyers (especially the lawyer whom they ruled against), and others may be in a hurry to get home after the inconvenience of having jury duty. Finally, a lawyer may not harass, coerce, threaten, or lie to a juror. "Harass" includes pestering the juror with repeated phone calls or home visits, and "coerce" includes threatening to "tell the judge" something the lawyer learned about the juror's background or conduct during the trial.

An attorney agreed to prepare a will for a client, a wealthy widow with three grown children. An earlier will divided her estate equally between her children, but the client now wants to modify the will to disinherit her only daughter, who disobeyed the client's wishes by marrying outside their nationality. The daughter is also a lawyer and is married to a lawyer, and the estate is substantial. The client's two sons are both working as manual laborers and they struggle financially. In the past, there had been some tension between the brothers and their sister, although the relationships seem to be cordial now. The attorney believes that disinheriting the daughter will ensure that the daughter and her husband will contest the will after the client's death and will rupture the tenuous relationship between the siblings. The client did not ask for the attorney's advice about disinheriting the daughter, she just insisted on it. The attorney initiated a debate about it, explaining that he believed it could be against the client's best interests and would cause unnecessary acrimony between her children. Was it proper for the attorney to initiate such advice when the client did not ask for it? A. Yes, because a lawyer may offer unsolicited advice to a client when doing so appears to be in the client's interest. B. Yes, because a lawyer has a duty to refer not only to law but also to other considerations such as moral factors that may be relevant to the client's situation. C. No, because a lawyer is not expected to give advice until asked by the client, and should normally wait until asked for such advice, especially when the advice is not strictly a statement of the law on a subject. D. No, because a testator has a sacred right to devise her estate as she wishes.

A. Yes, because a lawyer may offer unsolicited advice to a client when doing so appears to be in the client's interest. Comment 5 for Rule 2.1 concludes, "A lawyer ordinarily has no duty . . . to give advice that the client has indicated is unwanted, but a lawyer may initiate advice to a client when doing so appears to be in the client's interest." (See also the recent ABA clarification in ABA Formal Op. 05-434, note 15.) There is no prohibition against an attorney offering unsolicited advice for the benefit of the client.

During a personal injury trial, the court took a lunchtime recess for an hour. The plaintiff's attorney from the case walked across the street from the courthouse to a familiar diner to buy lunch. The diner was very crowded, so it was difficult for patrons to find a table to sit and eat. After ordering his sandwich at the counter, the attorney noticed two jurors from his own trial standing with their food, waiting for a free table. One of the jurors asked the attorney if they could share a table with him when one became available. The attorney agreed, but he reminded them that they could not talk about the case. The three sat together and ate their sandwiches. The two jurors talked most of the time, getting to know each other—discussing their children, their jobs, and their pets. The attorney did not participate in the conversation except to answer their questions about how many children he had, and whether he owned any pets. Another juror from the trial was at the diner, and noticed the attorney sitting with the other two jurors, which he reported to the judge when court reconvened. Could the attorney be subject to discipline for sharing a table with the jurors during a lunch break? A. Yes, because a lawyer should not have any ex parte social contact with jurors during a proceeding, even if they do not discuss the case. B. Yes, because he allowed the two jurors to sit with each other and get to know each other, which makes it more likely they will influence each other during deliberations. C. No, because this is a civil trial rather than a criminal trial. D. No, because the lawyer did not discuss the case with the jurors at all and barely participated in their socializing.

A. Yes, because a lawyer should not have any ex parte social contact with jurors during a proceeding, even if they do not discuss the case. Rule 3.5(b) prohibits all ex parte communications between lawyers and jurors during a proceeding. Courts have held that lawyers should be subject to discipline on facts similar to these—sharing a table with jurors at a restaurant during a trial recess, even without ulterior motives. The lawyer in this case should have politely explained that he could not share a table with them or talk to them at all until the case was over—and should probably have left the restaurant, if several jurors were there.

An attorney represents a client, who lost his criminal appeals and is now serving a life sentence in a federal penitentiary. The client confesses to the attorney that he (the client) committed a murder for which a jury incorrectly convicted another (innocent) man. The client says he is happy that someone else took the fall for that crime and that he will never tell anyone. The attorney lectures the client about the morality of this situation, allowing an innocent man to face life imprisonment or even capital punishment for a crime that the client committed, and pleads with the client to reveal the truth. Was it proper for the attorney to bring morality into his consultation with the client, and to sermonize on this point for a few moments? A. Yes, because in rendering advice, a lawyer may refer not only to law but to other considerations such as moral factors. B. Yes, because the attorney will have an obligation under the Model Rules to disclose the information if the client does not reveal the truth. C. No, because in rendering advice, a lawyer may refer only to legal and financial considerations, and not to personal views about morals or politics. D. No, because urging the client to reveal information that could overturn a final jury verdict undermines the finality of court decisions and the public's confidence in the legal system.

A. Yes, because in rendering advice, a lawyer may refer not only to law but to other considerations such as moral factors. The facts are similar to two actual cases—in both instances, a client refused to come forward and let an innocent person face a murder conviction and life sentence in prison. Rule 2.1 says that it is proper for a lawyer to bring in moral, social, and other factors when giving legal advice. While a client may not agree, it is not a violation for the lawyer in this question to urge the client to do the right thing, especially when the consequences for an innocent third party are so serious.

Money laundering. An attorney knew about another lawyer's involvement in an illegal money laundering enterprise, although the money laundering was unrelated to the other lawyer's law practice or representation of clients. Eventually, when federal law enforcement officials bring criminal charges against the other lawyer, who is part of another firm, the first attorney's awareness of the situation becomes evident. Could the attorney who knew of the wrongdoing and ignored it be subject to discipline? A. Yes, because it is a violation of the Rules of Professional Conduct to fail to report serious fraud or criminal activity by another lawyer. B. Yes, because the lawyer who knew and did nothing was an accomplice after the fact. C. No, because the attorney had no duty to report misconduct of lawyers in other firms. D. No, because the attorney could have put himself in danger by reporting an organized crime effort, and lawyers do not have to report misconduct when doing so might expose the reporting lawyer to retaliation criminal organizations.

A. Yes, because it is a violation of the Rules of Professional Conduct to fail to report serious fraud or criminal activity by another lawyer. Rule 8.3 requires lawyers to report serious misconduct by other attorneys. In this case, the attorney knows that another lawyer is committing a serious crime and violating various rules pertaining to the integrity of the profession. The lawyer has a duty to report this to the bar, and a lawyer could be subject to discipline (in theory) for failing to do so.

An attorney represented a client in a criminal prosecution. The client agreed to a plea bargain, and the case moved on to a sentencing hearing. The prosecution's pre-sentencing report to the judge erroneously indicates that the client has no prior convictions, and the trial judge asked the client directly whether that is true. The client affirmed that he had no prior criminal record, and the judge sentenced him leniently, giving him six months' probation. Yet the attorney had represented the client previously in another jurisdiction in a criminal matter, and he knew that the pre-sentencing report was erroneous. Before adjourning, the judge asked the attorney if he had anything else to say. Could the attorney be subject to discipline if he does not correct the judge's misperception about the client's criminal record? A. Yes, because the attorney must not allow his client to offer evidence that he knows to be false to a tribunal. B. Yes, because the client committed perjury when he answered the judge's question in the courtroom, once the court was in session for the sentencing hearing. C. No, because a lawyer cannot violate his ethical duty of confidentiality to his client. D. No, because the attorney did not make the false statement, and has no duty to correct the false statements of others.

A. Yes, because the attorney must not allow his client to offer evidence that he knows to be false to a tribunal. Rule 3.3(a)(3) provides, "If a lawyer, the lawyer's client, or a witness called by the lawyer, has offered material evidence and the lawyer comes to know of its falsity, the lawyer shall take reasonable remedial measures, including, if necessary, disclosure to the tribunal." We assume that the client's statement on the record during a sentencing hearing constitutes "material evidence," given that it is the crucial information influencing the leniency or severity of the sentence in this case, and in most sentencing cases—and in such a case, the lawyer has a duty to correct his client's false statement to the judge. As Comment 2 to Rule 3.3 explains, "Consequently, although a lawyer in an adversary proceeding is not required to present an impartial exposition of the law or to vouch for the evidence submitted in a cause, the lawyer must not allow the tribunal to be misled by false statements of law or fact or evidence that the lawyer knows to be false."

An attorney lives in the border town of Nashua, New Hampshire, which is a forty-five-minute drive from Boston, Massachusetts. The attorney took the Massachusetts bar exam and passed it, and now seeks admission to the bar in that state, as she has a job offer from a firm in Boston and plans to commute there every day from her home in New Hampshire. The state bar of Massachusetts has a rule that lawyers must be residents of the state to obtain a license to practice law there on a regular basis, so it declines her application to the bar. When the attorney challenges this decision in federal court, will she prevail? A. Yes, because the residency requirement violates the Privileges and Immunities Clause of the U.S. Constitution. B. Yes, because the residency requirement violates the Equal Protection Clause of the U.S. Constitution. C. No, because state courts have inherent authority to regulate the lawyers who practice in that state. D. No, because as an out-of-state resident, she lacks standing to challenge a regulation in that state.

A. Yes, because the residency requirement violates the Privileges and Immunities Clause of the U.S. Constitution. In Supreme Court of New Hampshire v. Piper, 470 U.S. 274 (1985), the United States Supreme Court held that a state's refusal to admit an out-of-state resident (New Hampshire had excluded lawyers who lived in neighboring Vermont), despite passing the state's bar exam, violated the Privileges and Immunities Clause. Similarly, the Court invalidated a residency requirement imposed by a federal court in Louisiana, in Frazier v. Heebe, 482 U.S. 641 (1987). Then in Supreme Court of Virginia v. Friedman, 487 U.S. 59 (1988), the Court struck down a state rule that let permanent Virginia residents licensed out of state waive into the Virginia bar, but required non-Virginia residents to take the state bar exam. Finally, in Barnard v. Thorstenn, 489 U.S. 546 (1989), the Court struck down U.S. Virgin Islands' one-year residency requirement for lawyers. Remember that residency requirements violate the Privileges and Immunities Clause rather than the Equal Protection Clause.

An attorney is engaged in civil litigation. On his way into the courthouse on the day of jury selection, reporters gather around the attorney hoping for comments. The attorney explains that the (unrelated) criminal trial happening at the courthouse that day is far more important, and he expresses regret that he is not involved in that case at all. He states that he believes the criminal case should result in an acquittal because the police (who are testifying as witnesses in the case) violated the defendant's civil liberties, and because the relevant penal statute itself, which furnished the basis for charges in the case, violates the Bill of Rights. His own civil case, he says, is a brief matter scheduled for a one-day trial, so he hopes to observe the closing arguments tomorrow in the important criminal case in the other courtroom. Are the attorney's statements proper? A. Yes, because the rule limiting trial publicity applies only to lawyers who are, or who have been involved in the investigation or litigation of a case. B. Yes, because the attorney is expressing opinions about the constitutionality of a law and of the state's actions, and such statements receive special protection under the First Amendment. C. No, because the attorney is commenting on the character or reputation of police who will be witnesses in the case. D. No, because the attorney's arguments would be inadmissible at trial, if the courts have already upheld the constitutionality of the statute and the police actions in this circumstance.

A. Yes, because the rule limiting trial publicity applies only to lawyers who are, or who have been involved in the investigation or litigation of a case. Rule 3.6(d) extends the prohibition on media discussions about pending litigation to other lawyers in the same firm or government agency as the lawyer handling the case. At the same time, Comment 3 to Rule 3.6 provides, "Recognizing that the public value of informed commentary is great and the likelihood of prejudice to a proceeding by the commentary of a lawyer who is not involved in the proceeding is small, the rule applies only to lawyers who are, or who have been involved in the investigation or litigation of a case, and their associates."

An attorney represents Conglomerate Corporation in a lawsuit against the company brought by an individual plaintiff. The lawsuit could bring bad publicity to Conglomerate Corporation and could adversely affect its stock share price. Conglomerate offers to settle the matter quietly, but the plaintiff rejects the settlement offer. The attorney then files a counterclaim against plaintiff, alleging libel and slander of Conglomerate Corporation, vexatious litigation, and tortious interference with contract, for which he demands millions of dollars in damages. The attorney and plaintiff's counsel both know these counterclaims lack any basis in fact, and that they will be costly for plaintiff to defend. The attorney uses the counterclaims as leverage in reopening the settlement negotiations, offering to withdraw the counterclaims if plaintiff will accept a new, slightly higher settlement offer. The plaintiff calculates the cost of defending against the counterclaims and the difference between the settlement offer and the expected damages if plaintiff wins at trial, and reluctantly agrees to accept the terms of the offer. Could the attorney be subject to discipline for filing the counterclaims? A. Yes, because there is no factual basis for the claims, and the lawyer did not bring them in good faith. B. Yes, because the lawyer used the counterclaims as leverage to induce the opposing party into accepting an unfavorable settlement. C. No, because an advocate has a duty to use legal procedure for the fullest benefit of the client's cause. D. No, because the claims and counterclaims settled before going to trial, so the lawyer did not violate his duty of candor to the court.

A. Yes, because there is no factual basis for the claims, and the lawyer did not bring them in good faith. This question illustrates the basic rule set forth in Rule 3.1, prohibiting frivolous claims and assertions by lawyers. Such claims are an abuse of the legal system and attorneys have an ethical duty to avoid such behavior. In this case, the lawyer on the losing side is using frivolous counterclaims to extort a more favorable settlement agreement from the other party. Such behavior is unfair, and it tarnishes the reputation of the legal profession as a whole, and clogs the court system with unmeritorious claims and motions.

The shareholder. An attorney is a partner in a law firm, and owns $100,000 worth of stock in Conglomerate Corporation, the named defendant in a new antitrust suit. The attorney's total compensation from the firm is around $15 million per year, including bonuses, and his net worth is around $500 million. His home is worth about $7 million and the attorney inherited it, so the property is unencumbered by any mortgage or liens. The attorney works in a specialized area of law at the firm and does not have much interaction with the other lawyers, except at parties and occasional partners' meetings. Another lawyer in the firm seeks to represent the plaintiffs in the antitrust action against Conglomerate Corporation, which is not a client of the firm. Would it be proper for the firm to represent the plaintiffs in litigation against Conglomerate Corporation? A. Yes, so long as the attorney is not involved in the representation, there will be no imputation of the attorney's conflict of interest to the other lawyers in the firm, because it is the attorney's personal interest and does not present a significant risk of materially limiting the representation of the client by the remaining lawyers in the firm. B. Yes, so long as the plaintiffs provide written informed consent to the potential conflict of interest, and the firm carefully screens the other lawyer representing them from the rest of the firm. C. No, because the personal interest of the firm's managing partner in Conglomerate is so great, relative to his earnings and assets, that there is a significant risk of materially limiting the representation of the plaintiffs in their cause of action against Conglomerate. D. No, because it is a nonconsentable conflict of interest for the firm to represent both adverse parties in litigation.

A. Yes, so long as the attorney is not involved in the representation, there will be no imputation of the attorney's conflict of interest to the other lawyers in the firm, because it is the attorney's personal interest and does not present a significant risk of materially limiting the representation of the client by the remaining lawyers in the firm. A is correct because the attorney's interest is personal and he will not be involved in the representation of the plaintiffs, therefore no conflict imputes to the firm or other lawyers in the firm.

An attorney worked for several years for a federal government agency in regulatory enforcement. Big Firm then hired the attorney for a much higher salary, and the attorney accepted the position and left her government position. One of the attorney's first assigned cases at Big Firm was a new action by the client against Conglomerate Corporation. The attorney had worked on an enforcement against Conglomerate Corporation and learned confidential government information about the entity during the litigation. The government agency gives its informed consent, confirmed in writing, to the representation. Is the attorney nevertheless subject to disqualification in the client's matter against the attorney's former employer? A. Yes, the attorney has confidential government information about a person acquired while working for the government agency, and therefore may not represent a private client whose interests are adverse to that person in a matter in which the information could be used to the material disadvantage of that person. B. No, because the attorney did not previously represent the client or Conglomerate Corporation, so there is no attorney-client privilege or conflict of loyalties here between two clients that the attorney is representing or has represented. C. No, because the appropriate government agency gave its informed consent, confirmed in writing, to the representation. D. No, as long as Big Firm has policies and procedures in effect to supervise the attorney's work closely enough to ensure compliance with the Rules of Professional Conduct, including training sessions about the conflict-of-interest rules.

A. Yes, the attorney has confidential government information about a person acquired while working for the government agency, and therefore may not represent a private client whose interests are adverse to that person in a matter in which the information could be used to the material disadvantage of that person. A is correct because the client's interest is adverse to Conglomerate's. The attorney has learned Conglomerate's confidential information, and there is no waiver or exception to this kind of disqualification.

Court administrator. An attorney was a judge for several years. Near the end of her tenure as a judge, she functioned in the role of the chief administrative judge in that court, assigning cases to the other judges and supervising their work, and had only a limited docket of her own trials. The attorney then left the bench and opened her own law practice. The attorney agrees to represent the client in a matter in the same courthouse where the attorney formerly served as a judge. The attorney even remembers the case, but only the names of the parties and the nature of the action, because she assigned it to the trial judge who currently has the case on his docket, but the attorney had no other involvement in the matter. The client's previous lawyer in the matter was subject to disqualification at the motion of the opposing party due to a conflict of interest. Is it proper for the attorney to represent the client in this matter? A. Yes, the fact that a former judge exercised administrative responsibility in a court does not prevent the former judge from acting as a lawyer in a matter where the judge had previously exercised remote or incidental administrative responsibility that did not affect the merits. B. Yes, as long as all the parties to the matter provide informed consent, confirmed in writing, to the representation. C. No, because she previously supervised the trial judge hearing the case, and even assigned the case to that judge. D. No, because the client's previous lawyer was already subject to disqualification due to a conflict of interest in the matter.

A. Yes, the fact that a former judge exercised administrative responsibility in a court does not prevent the former judge from acting as a lawyer in a matter where the judge had previously exercised remote or incidental administrative responsibility that did not affect the merits. A is correct because the attorney had no personal participation in the case while she was a judge.

In a discussion with his attorney, a client admits to a murder for which another person has been wrongfully convicted. That person has been convicted of capital murder and sentenced to death. Under Model Rule of Conduct 1.6 the attorney A. may reveal the exculpatory information. B. must reveal the exculpatory information. C. must withdraw from continued representation of the confessed murderer. D. None of the above

A. may reveal the exculpatory information. The correct answer is A. Rule 1.6 allows—not mandates—an attorney to reveal confidences as necessary to avoid bodily injury. Because the wrongfully convicted inmate faces death, this situation falls in the "may" exception to the mandate to keep clients' confidences. Answer B is incorrect because the Rule includes "may" not "must" language. Answer C is incorrect because the Rule does not mandate withdrawal when a client confesses to a crime for which another is wrongfully convicted.

Let's see if you read Rule 6.1 and the comments carefully. Do the following activities count toward the lawyer's aspirational goal of performing 50 hours per year of pro bono services? Assume the State of Perplexity requires lawyers to report pro bono activities annually on their licensing form. (A) A lawyer has spent five hours assisting a woman whose income is just above the poverty line in obtaining a temporary restraining order against an abusive partner. The lawyer and client had a written fee agreement providing that the client would pay the lawyer's customary fee of $150 per hour. (The client told the lawyer that her parents would help her with her legal expenses.) At the conclusion of the representation, however, the client could not pay and told the lawyer that she made up the story about her parents helping her with the expenses. The lawyer decided to write off the $750 as uncollectable and not attempt to recover it from the client. May the lawyer report having performed five hours of pro bono service for the client? (B) A lawyer represents several plaintiffs suing a landlord for violating a state statute prohibiting discrimination in rental housing. The lawyer works for Equal Housing Now, a non-profit legal services organization, funded by grants from the state Interest on Lawyers' Trust Accounts (IOLTA) fund. Equal Housing Now represents only clients whose incomes fall below the poverty level, as defined by federal law. The state housing discrimination statute includes an attorneys' fees provision, giving the trial court discretion to award attorneys' fees in addition to damages if the court finds that the discrimination was particularly egregious. Plaintiffs represented by Equal Housing Now are told that under no circumstances will they be required to pay for the services of the organization's lawyers. After a trial concluded with a judgment in favor of the plaintiffs, the trial court entered an award of $50,000 in attorneys' fees, payable to Equal Housing Now, concluding that the landlord's conduct was outrageous. (C) A lawyer serves on the board of Primitive Pathways, a local non-profit organization that provides wilderness survival and leadership training for teenagers. Fees for the summer Primitive Pathways program are $200 per week. The kids served are mostly middle-class, although a few families with more modest incomes pay no fee. An annual chicken barbecue raises money to provide scholarships for these families. Recently Primitive Pathways was sued by a family who claimed that its programming was not suitable for teenagers with certain types of disabilities. With the consent of the other members of the board, the lawyer represented Primitive Pathways in the litigation. She agreed to do so for no fee. After the lawyer worked approximately 200 hours on the matter, the lawsuit was settled and Primitive Pathways agreed to make changes to its activities. May the lawyer report any of this time on her license renewal form? Does it matter whether she has performed a substantial majority of 50 hours of pro bono service in other matters?

Although a lawyer is not subject to discipline for failing to live up to the aspiration to render 50 pro bono hours per year, in a mandatory reporting state the lawyer must be truthful on her license renewal form. See Rule 8.4(c). Do these matters satisfy the tests set out in Rule 6.1? (A) Nope. Rule 6.1(a) defines pro bono services as those performed with no expectation of a fee. That means no ex ante belief that the client will be able to pay. In this case, the lawyer thought the client had someone to help her pay her bills, so the lawyer had an expectation of receiving a fee. See Rule 6.1, cmt. [4] ("services cannot be considered pro bono if an anticipated fee is uncollected"). The situation also does not come within Rule 6.1(b)(2), because the lawyer charged her customary fee. If the facts had been different, and the lawyer had offered to charge a modest fee of, say, $25 per hour, then the lawyer may report the five hours as part of the "category (b)" hours—not a substantial majority, but filling out the remainder of her 50-hour requirement. See Rule 6.1, cmt. [7]. (B) Yes. This is the flipside of the previous example. Go through the requirements of Rule 6.1(a): The lawyer is providing legal services to persons of limited means, and is doing so for no fee. What about the language, "expectation of a fee?" Here, the lawyer had no ex ante expectation of receiving a fee. Comment [4] clarifies the effect of the court's award of attorneys' fees: "[T]he award of statutory attorneys' fees in a case originally accepted as pro bono would not disqualify such services for inclusion under [Rule 6.1(a)(1)]. (C) This is a close call. Look carefully at Rule 6.1(a)(2) and Rule 6.1(b)(1). The first rule, covering services that can make up a substantial majority of a lawyer's reported pro bono hours, pertains to legal services to organizations "that are designed primarily to assist the needs of persons of limited means." See Comment [3], which mentions homeless shelters, domestic violence centers, and food pantries as examples of this type of organization. While outdoor leadership is important, it is not the same type of basic interest as food or shelter, and in any event Primitive Pathways is not primarily aimed at meeting the needs of persons of limited means. Thus, the case does not fall within Rule 6.1(a)(2). As for (b)(1), there are a similar limitation, but the language is somewhat broader. To qualify for pro bono under Rule 6.1(b)(1), an organization must be aimed either at meeting legal needs, such as "First Amendment claims, Title VII claims and environmental protection claims," Rule 6.1, cmt. [6], or at charitable, religious, civic, etc., purposes, and it must be the case that the payment of standard legal fees would "significantly deplete the organization's economic resources." Primitive Pathways fails the first prong of this test, but it may be the case that paying the lawyer for 200 hours of work would significantly deplete the organization's resources. The example does not say so explicitly, but it was written to make Primitive Pathways sound like a modest community-based organization with a relatively limited budget.

What is the impetus for the promulgation of the Model Rules of Professional Conduct 8.4(g)? A. The need to hold attorneys accountable for bias or discrimination in which their clients may engage B. A recognition that bias and discrimination should be included in what constitutes professional misconduct C. A desire to expand the reach of Rule 8.4 because of the numerous instances of attorney misconduct D. None of the above

B. A recognition that bias and discrimination should be included in what constitutes professional misconduct The correct answer is B. The 2016 amendment to Rule 8.4 adds section (g) and is a result of a desire to eliminate bias and enhance diversity in the practice of law. A is incorrect because attorneys are not typically accountable for the biases of their clients. C is incorrect because widespread bias and discrimination—not general misconduct—led to the promulgation of Rule 8.4(g).

Which of the following would most likely result in a finding of professional misconduct? A. An attorney notified his client that he was in the middle of a trial on behalf of another client and would be less accessible during the following week. The client's case was not prejudiced, but the client was frustrated that he was unable to have the undivided attention of his attorney. B. An attorney without any experience in medical malpractice law takes on a medical malpractice case. Although he completely mismanages the case, he gets a default judgment. C. An attorney gives legal advice during a free consultation. D. All of the above

B. An attorney without any experience in medical malpractice law takes on a medical malpractice case. Although he completely mismanages the case, he gets a default judgment. Even though the attorney lucked out and received a judgment in his client's favor, the attorney is still subject to discipline because he violated Rule 1.1, which requires competence.

An attorney practices personal injury law, representing plaintiffs on a contingent fee basis. The attorney employs a paralegal to assist with preparing documents for litigation. The paralegal's salary arrangement is 10 percent of the firm's total net revenue each year. In years when the attorney wins several large cases, the paralegal receives higher wages, and in years when the attorney has no big wins, the paralegal receives almost nothing. The paralegal does not bring clients to the firm and does not participate in judgments about which clients to represent, or about how to handle the cases. Is the attorney subject to discipline for this arrangement? A. Yes, unless the paralegal has a law degree and has obtained a law license in another state. B. No, because non-lawyers may participate in a firm compensation plan based on overall profit sharing. C. Yes, because the paralegal here is engaged in the unauthorized practice of law. D. No, because the rules treat paralegals the same as lawyers for purposes of sharing fees or profits.

B. No, because non-lawyers may participate in a firm compensation plan based on overall profit sharing. Rule 5.4(a)(3) allows that "a lawyer or law firm may include nonlawyer employees in a compensation or retirement plan, even though the plan is based in whole or in part on a profit-sharing arrangement." Lawyers cannot pay a non-lawyer a direct contingent fee related to a particular case, but they can share profits with non-lawyer employees based on the firm's total revenue, even though that revenue comes (at least partly) from contingent fees earned by the lawyer.

A client recently posted pictures on social media that are unfavorable to the client's civil case. The client's lawyer instructs the client to take down the post. A week later, opposing counsel sends a discovery request specifically asking for any and all pictures posted on social media for the last month. The lawyer instructs the client to produce only those pictures currently posted and not provide any photos that were unposted, including the photos that were unfavorable. In regard to the lawyer's instruction about responding to the discovery request, was the lawyer's advice proper under the ethical rules? A. Yes, because the client's interest will be adversely affected. B. No, if the lawyer is counseling the client to obstruct access to the pictures that may have evidentiary value. C. Yes, because no valid obligation exists to produce the photos. D. No, unless the lawyer reasonably believes the client's interest will not be adversely affected.

B. No, if the lawyer is counseling the client to obstruct access to the pictures that may have evidentiary value.

An attorney gets a favorable settlement in a divorce hearing and posts on social media, "After months of dealing with his hateful estranged wife, my client won enough money to treat his cancer!" Assuming the client knows nothing about the social media post, which of the following is true? A. The attorney is not subject to discipline because the court's finding is likely public information. B. The attorney is subject to discipline under MRPC 1.6. C. The attorney is subject to discipline only if the information posted on the attorney's social media is open to the public. D. The attorney is not subject to discipline if the information is true.

B. The attorney is subject to discipline under MRPC 1.6. The correct answer is B. The duty to keep client confidences under Rule 1.6 extends to digital media, even if that information is not posted to the general public. Answer A is incorrect because under the Model Rules of Professional Conduct, there is no exception based on the availability of the information elsewhere. Notably, the attorney also included confidential information about the client's health condition and relationship with his wife. Answer C is incorrect because confidential information under Rule 1.6 remains confidential irrespective of the source. Answer D is incorrect because the veracity of information does not impact the obligations under Rule 1.6.

A client hired an attorney to represent her in a litigation matter. At the end of the first day of trial, the client is unhappy with her lawyer's performance in the courtroom and informs the attorney that she is firing him and will find another lawyer. The attorney wants to continue representing this client until the end of the trial. May the client discharge the attorney after a trial has begun? A. Yes, as long as a client obtains permission from the court to discharge an attorney, it is permissible. B. Yes, a client has a right to discharge a lawyer at any time, with or without cause, subject to liability for payment for the lawyer's services. C. No, a client may not discharge a lawyer once a trial is underway, because the disruption could be prejudicial to the opposing party. D. No, a client cannot discharge a lawyer once the lawyer has received confidential information about the client's case. Submit

B. Yes, a client has a right to discharge a lawyer at any time, with or without cause, subject to liability for payment for the lawyer's services. B is correct—clients have a right to discharge their attorneys at any time, for any reason. Usually, this can occur even in the middle of a trial, although in exceedingly rare instances a judge may forbid the discharge if it seems calculated merely to disrupt and delay the proceedings.

Tie us over for the weekend. A client retains an attorney to handle a criminal matter. The client delivers a retainer check to the attorney on Friday afternoon. The retainer check will only cover the work the attorney anticipates he will begin and complete the following Monday. Because the following Monday is a banking holiday, if the attorney deposits the retainer check into his client trust account on Friday afternoon, he will not have access to the funds until Tuesday. The attorney deposits the retainer check into his business checking account and pays himself on Friday before the firm closes with those funds. Is the attorney subject to discipline? A. Yes, attorneys shall not accept amounts paid in advance for criminal matters. B. Yes, attorneys shall deposit amounts paid in advance into a client trust account and the attorney shall not withdraw the funds until fees are earned or expenses are incurred. C. No, if an attorney believes the funds will be earned within a short period, the attorney may deposit the amount he anticipates will be earned directly into his business account. D. No, when an event out of an attorney's control, such as a bank holiday, causes the funds to be unavailable when the attorney anticipates he will need to withdraw the funds, the attorney may deposit the amount he reasonably believes will be earned or needed for expenses into his business account instead of the client trust account.

B. Yes, attorneys shall deposit amounts paid in advance into a client trust account and the attorney shall not withdraw the funds until fees are earned or expenses are incurred. Rule 1.15(c) says, "A lawyer shall deposit into a client trust account legal fees and expenses that have been paid in advance, to be withdrawn by the lawyer only as fees are earned or expenses incurred." Commingling funds is simply impermissible under any circumstances except the one narrow exception (covering the bank fees on a trust account).

A prosecutor in a felony drug case addressed a group of reporters outside the District Attorney's office. In response to questions about the specific case underway, the prosecutor explained that the judge had consolidated the trials of three co-defendants into a single proceeding and had postponed the proceeding until the next summer, four months away. Was it proper for the prosecutor to disclose such details about the case to reporters? A. Yes, because the public has a right to know how the details of a criminal prosecution, as the taxpayers are paying the prosecutor's salary. B. Yes, because a lawyer may tell reporters the scheduling or result of any step in litigation. C. No, because no lawyer associated in a firm or government agency subject to the Rules of Professional Conduct shall make a statement prohibited by the rules. D. No, because criminal jury trials will be most sensitive to extrajudicial speech.

B. Yes, because a lawyer may tell reporters the scheduling or result of any step in litigation. Rule 3.6(a)(4) permits lawyers to make extrajudicial statements in the public media regarding "the scheduling or result of any step in litigation." The various provisions of Rule 3.6 are the ABA's attempt to balance the public's need to know about court proceedings with the need to avoid prejudicing the outcome of a pending adjudication. An example of information that is not too prejudicial, and therefore permissible to disclose to the media, is the expected trial date or other scheduling information about a proceeding.

A client, who happened to be a judge, hired an attorney to represent her in her divorce proceeding against her husband, who is guilty of marital infidelity. Their fee agreement stipulates that the attorney would bill the client every month for the work performed in the previous thirty days. After two months of representation, the attorney has sent the client two bills, and has received no payments. Is it proper for the attorney to seek to withdraw from the case based on unpaid fees? A. Yes, because otherwise the attorney will develop a conflict of interest with his own client, as the share of the marital assets will impact the client's ability to pay all the outstanding fees at the end of the proceeding. B. Yes, because a lawyer may withdraw if the client fails substantially to fulfill an obligation to the lawyer regarding the lawyer's services and has been given reasonable warning that the lawyer will withdraw unless the obligation is fulfilled. C. No, because a lawyer representing a judge may not withdraw without the judge's approval or permission. D. No, because withdrawing over unpaid fees turns the representation into a contingent fee arrangement, which is impermissible in a divorce case.

B. Yes, because a lawyer may withdraw if the client fails substantially to fulfill an obligation to the lawyer regarding the lawyer's services and has been given reasonable warning that the lawyer will withdraw unless the obligation is fulfilled. B is the correct answer, as it accurately states the rule in 1.16(b)(5). Forcing lawyers to continue to represent delinquent clients would be an undue hardship for many and could create a perverse incentive to decline representation for many needy clients.

A client is aware that he is under investigation for student loan fraud. A friend who works at the courthouse tips off the client that a magistrate issued a warrant to search the client's home for evidence the next day in the early morning. In a panic, the client calls his attorney and asks what he should do. The attorney informs him that the agents executing the warrant will surely seize any computers and hard drives that they find, and that the client should probably wipe and reformat all his drives or dispose of his computers, that he should probably smash his cell phone, and that he might want to go on a long vacation immediately. Is the attorney subject to discipline for this advice? A. Yes, because the Sixth Amendment right to counsel does not arise until formal adjudicatory proceedings begin. B. Yes, because a lawyer shall not counsel or assist another person to destroy or conceal a document or other material having potential evidentiary value. C. No, because the traditional rules against destroying documentary evidence apply only to printed copies, not to electronic files stored on a computer hard drive. D. No, because until the police execute the warrant and legally seize the computers, they are the client's private property and he can do whatever he wants with them.

B. Yes, because a lawyer shall not counsel or assist another person to destroy or conceal a document or other material having potential evidentiary value. Rule 3.4(a) says that a lawyer must not "unlawfully alter, destroy or conceal a document or other material having potential evidentiary value. A lawyer shall not counsel or assist another person to do any such act." This type of question should be intuitive for students, as the rule about destruction of evidence resonates with popular morality about cheating. Nevertheless, it is useful for students to clarify their thinking about the rules, as the MPRE can offer a confusing selection of answers that echo other procedural or ethical rules that seem familiar to students.

An attorney faced criminal sanctions for having over two thousand unpaid traffic and parking tickets, and several instances of failure to appear for jury duty. Could the attorney be subject to professional discipline for these minor offenses? A. Yes, because any illegal activity by a lawyer constitutes professional misconduct. B. Yes, because a pattern of repeated offenses, even ones of minor significance when considered separately, can indicate indifference to legal obligation. C. No, because traffic violations or neglecting jury duty would have no specific connection to fitness for the practice of law. D. No, because these activities do not arise from or pertain to the attorney's representation of a client.

B. Yes, because a pattern of repeated offenses, even ones of minor significance when considered separately, can indicate indifference to legal obligation. Comment 2 for Rule 8.4 concludes, "A pattern of repeated offenses, even ones of minor significance when considered separately, can indicate indifference to legal obligation." There are recorded cases of disciplinary authorities imposing sanctions on lawyers for egregious amounts (hundreds) of traffic violations.

Enforcing child support orders. A court orders that a certain client should receive child support from her ex-husband. The client's ex-husband stopped making child support payments twelve months ago. The client hires an attorney to handle the enforcement of child support against the client's ex-husband. The attorney agrees to take the case on a contingency basis because the client cannot afford to hire an attorney since she has not been receiving child support from her ex-husband. The client also asks the attorney to pay her court costs, as she cannot afford those either. The attorney prepares a contract that states the attorney will only be paid for his representation if the client prevails on the enforcement motion, but that court costs will be reimbursed by the client within thirty days of the finalization of the case regardless of whether the client prevails. Is the attorney's conduct proper? A. Yes, because attorneys may represent clients, regardless of the type of case, on a contingency basis, as long as clients are required to reimburse the attorney for the actual expenses paid by the attorney for the client. B. Yes, because attorneys may accept cases on a contingency basis in domestic relations issues if the case is merely to enforce a prior order, and attorneys may pay for court costs for clients. C. No, because attorneys cannot advance funds to clients for any expenses, whether or not those expenses are related to the case. D. No, because contingency fees are impermissible in any case involving domestic relations, including enforcement of prior orders.

B. Yes, because attorneys may accept cases on a contingency basis in domestic relations issues if the case is merely to enforce a prior order, and attorneys may pay for court costs for clients. Rule 1.5(d)(1) prohibits contingent fees in divorce or child custody cases. The policy concern behind this rule is to avoid giving family law attorneys any incentive to make a divorce or child custody proceeding more acrimonious than it already is, or more melodramatic, merely to earn a higher fee. Nevertheless, Comment 6 to Rule 1.5 qualifies this prohibition with a specific exception: "This provision does not preclude a contract for a contingent fee for legal representation in connection with the recovery of post-judgment balances due under support, alimony or other financial orders because such contracts do not implicate the same policy concerns." The MPRE in the recent past has included a question about this specific exception.

An attorney represented a client in an action for replevin. After the filing of the case, but before the court had sent any notices about the docket number, the attorney spoke to a clerk at the courthouse, and inquired whether the case had received an assignment yet to a judge. The clerk said it was still unassigned. The attorney then asked the clerk to mention to the Director of Judicial Administration, who was also the Chief Presiding Judge, that they should not assign the case to a particular judge, who was notorious for having a bias against parties like the attorney's client, and who had an extraordinarily high reversal rate from the appellate courts in replevin cases. The clerk said he would mention the conversation to the Director, which he did. The Director said she could not accommodate special requests from lawyers regarding case assignments, but when it came time to assign the case, she assigned the case to another judge merely to avoid another embarrassing reversal from the appellate courts. Was it improper for the attorney to ask the clerk to pass his concerns along to the Director? A. Yes, because he should have waited until the case was assigned before asking the administrator to reassign it to another judge. B. Yes, because during a proceeding a lawyer may not communicate ex parte with persons serving in an official capacity in the proceeding, such as judges, masters, or jurors, unless authorized to do so by law or court order. C. No, because the attorney did not actually speak ex parte with the judicial officer, but instead spoke with a front-counter clerk. D. No, because the case had not yet been assigned to any judge.

B. Yes, because during a proceeding a lawyer may not communicate ex parte with persons serving in an official capacity in the proceeding, such as judges, masters, or jurors, unless authorized to do so by law or court order. Rule 3.5(a) prohibits lawyers from having ex parte contact with anyone serving in an official capacity in a case. The purpose of this problem is to emphasize for students that the prohibition applies to more individuals than the judge who presides at a trial. It can apply to hearing officers or commissioners in administrative proceedings, court administrators (as in this case), bailiffs, and judicial clerks. Note that it does not matter if the ex parte communication produces the result that the attorney had sought—an attorney can be subject to discipline even if the official or judge ignores the communication. Similarly, it does not matter that the official or judge does do what the attorney wanted, but for another (perfectly legitimate) reason—that is, whether the judge or official would have done it anyway, even without the ex parte communication. What matters is the action taken by the lawyer.

A clean start in another state. An attorney obtained admission to the bar in New York and practiced there for two years. She worked for Big Firm, which has offices in five states and a few locations overseas. After her two years in the New York office, the firm transferred her to its office in San Diego, California. The attorney then applied for admission to the California bar under a reciprocity arrangement, and the state bar admitted her without making her re-take the bar exam. After practicing in California for three years, somehow the New York state bar learned that the attorney had made false statements on her original bar application about misdemeanor arrests during college. The New York bar informed the California state disciplinary authority about this problem, and the California state bar commenced disciplinary proceedings against the attorney in California. Can the attorney be subject to discipline in California for false statements made on a bar application in another state? A. Yes, because the states depend on each other to help enforce their own attorney disciplinary rules, and California therefore has a legal duty to enforce disciplinary rules from New York. B. Yes, because if a person makes a material false statement in connection with an application for admission, it may be relevant in a subsequent admission application or disciplinary proceeding elsewhere. C. No, because the alleged misconduct occurred on a bar application in a non-contiguous state, so California has no jurisdiction over the matter. D. No, because the fact that the attorney has now practiced for five years means that the estoppel doctrine prevents a state bar from revisiting her original bar application.

B. Yes, because if a person makes a material false statement in connection with an application for admission, it may be relevant in a subsequent admission application or disciplinary proceeding elsewhere.

An attorney located a witness who could corroborate his client's story. The witness, however, was afraid of retaliation from others if she testified, and did not want to be involved. The witness also lives 1,000 miles away and works as a waitress, so she cannot afford the travel expenses and lodging, and cannot afford to miss work, because she receives no wages if she does not work. The attorney offers to pay all the witness's expenses. The attorney then pays for airfare and pays to put the witness in one of the nicest hotels in the city and pays for all of the witness's dining bills at expensive downtown restaurants. The witness reluctantly agrees. Was it proper for the attorney to offer to pay the expenses for a favorable witness to undergo the trouble of testifying at the trial? A. Yes, because expert witnesses routinely charge large sums to testify at trial, so it is proper for a non-expert to receive a modest amount of compensation, especially if she is fearful of adverse consequences from testifying. B. Yes, because it is proper to pay a witness's expenses, so long as the attorney does not offer to pay the witness an inducement to provide favorable testimony. C. No, because it is improper to pay an occurrence witness any fee for testifying. D. No, because it is proper to compensate a witness only if the lawyer will also compensate a witness for the opposing party.

B. Yes, because it is proper to pay a witness's expenses, so long as the attorney does not offer to pay the witness an inducement to provide favorable testimony. This problem begins in a very similar way to the previous problem, but the attorney here takes a different approach—instead of offering a lump sum to a witness to testify, the attorney offers to cover travel (airfare), lodging, expensive meals, and possibly lost wages from missing work. Compensating an occurrence witness for costs related to testifying is proper, according to Comment 3 for Rule 3.4, but offering to pay the witness to testify is not proper. Of course, in some cases, the distinction is mostly semantic— an attorney may end up (properly) spending thousands of dollars for a witness' travel, lodging, food, and so on—the same amount that would have been improper to offer the witness as an inducement to testify. The difference is that compensating the witness leaves the person no better or worse off than if the person had not testified, so it is not a true inducement. Note also that "reasonable" reimbursement or compensation for costs includes nice hotels and nice restaurants—perhaps nicer than what the witness could normally afford personally. There is no requirement that lawyers put witnesses in the cheapest hotel in town or force them to eat cheap fast food. Of course, it is conceivable that compensating for "costs" could become too excessive and unreasonable—say, a luxury hotel or restaurant that normally caters only to the super-wealthy. Courthouses are typically in downtown locations, and the nearby restaurants and hotels typically have downtown prices.

Supporting the cause. An attorney is active within a new political movement and she has represented several members of the movement, who faced arrest or criminal charges for protesting and picketing. The attorney learns that police have arrested one of the prominent leaders of the movement for trespassing on private property during a protest, but that the movement leader is already out on bail. In response, the attorney calls the leader and offers to represent him in his case free of charge, explaining that she has experience representing other members of the movement in similar cases. The leader agrees to have the attorney represent him on a pro bono basis. The attorney wants to represent the leader because she admires him, but also because she believes it will generate terrific publicity for the firm's practice. Was it proper for the attorney to make this telephone solicitation? A. Yes, because the attorney believes in the leader's cause and is an active member of the movement. B. Yes, because the attorney did not charge for providing these legal services. C. No, because the attorney made a live telephone solicitation of a prospective client. D. No, because the attorney hopes to receive indirect benefit from the publicity that the representation will bring.

B. Yes, because the attorney did not charge for providing these legal services. Rule 7.3 broadly prohibits telephone solicitations by lawyers to prospective clients, but with an important proviso: "when a significant motive for the lawyer's doing so is the lawyer's pecuniary gain." Here, the lawyer offers to represent the leader without charge, motivated mostly by support for the cause. The fact that there will also be an indirect financial benefit in the form of publicity for the firm does not create a violation.

An attorney agreed to draft a will for a new client who wanted to leave his entire estate to his children, but he wanted to disinherit his estranged wife entirely. The will stated that the entire estate would pass to the children. After the client died, the wife claimed her statutory share, which in that jurisdiction was 50 percent of the estate, in spite of the instructions in the will. It should have been foreseeable to the attorney at the time of drafting that the will would not be sufficient to overcome the wife's claim to her statutory share, but he did not explain this to the client or recommend measures to circumvent the problem. Could the frustrated children have a viable claim against the attorney for legal malpractice? A. Yes, because the heirs under a will always have privity to sue the lawyer who drafted the will. B. Yes, because the client's intention was clear on the face of the will, and the lawyer therefore could be liable to the heirs even though they are not clients. C. No, as there is no client-lawyer relationship between the attorney and the children. D. No, because the lawyer did not violate any ethical or fiduciary duty in this case.

B. Yes, because the client's intention was clear on the face of the will, and the lawyer therefore could be liable to the heirs even though they are not clients. This is reminiscent of a recent question that appeared on the MPRE and an example from the Restatement. A lawyer can be liable to the intended heirs under a will drafted by the lawyer if the client's intent was clear on the face of the document, even though the heirs were not the lawyer's clients. The lawyer here did not exercise due competence or diligence, and he failed to communicate to the client sufficiently enough for the client to make proper decisions.

A firm specializing in sports law represented several professional athletes as clients. The state disciplinary authorities suspended and eventually disbarred one of the associates at the firm, but the disciplinary action did not implicate the rest of the firm (the lawyer's misconduct had occurred completely outside the scope of his duties there). The firm retained the disbarred lawyer as a sports agent for some of the athletes who were clients of the firm. The disbarred lawyer would draft contracts for the athletes and negotiate deals for the firm's clients with their sports teams or with companies seeking the athlete's product endorsement. Could the partners at the firm be subject to discipline for facilitating the disbarred lawyer in the unauthorized practice of law? A. Yes, because a firm may never have any business dealings with a disbarred attorney. B. Yes, because the disbarred lawyer is engaging in the unauthorized practice of law by drafting and negotiating contracts for the firm's clients. C. No, because the firm is not representing that the disbarred lawyer is an attorney and the agent does not have his own clients. D. No, because the disbarred attorney is working under the supervision of licensed attorneys.

B. Yes, because the disbarred lawyer is engaging in the unauthorized practice of law by drafting and negotiating contracts for the firm's clients. Rule 5.5(a) says, "A lawyer shall not practice law in a jurisdiction in violation of the regulation of the legal profession in that jurisdiction, or assist another in doing so." Jurisdictions vary in what they allow disbarred lawyers to do. Some states, such as Indiana, forbid a disbarred or suspended lawyer from "maintaining a presence or occupying an office where the practice of law is conducted." Others, like New York, allow law firms to employ disbarred attorneys as legal assistants, investigators, or paralegals, so long as the individual has no contact with clients. No state would permit a disbarred lawyer to continue drafting contracts and negotiating deals for a law firm's clients. The disbarred lawyer is therefore engaging in the unauthorized practice of law, and the firm is facilitating it, which is also a violation. A recent MPRE tested on this issue.

During a trial, the judge overruled an objection by one of the attorneys. The attorney felt that the judge had made a fundamental error and had ignored a clear provision of the official Rules of Evidence. Court adjourned for the day a few minutes later, and the judge retreated to his chambers. The attorney approached the judge's clerk, who was still in the courtroom, and gave him a handwritten note, folded into a square, to pass along to the judge. The clerk gave the note to the judge. The note thanked the judge for recently inviting the attorney to the judge's home, along with sixty other people from the legal community, for a holiday party. It also said that the judge had made a mistaken ruling on the attorney's objection that day, and it referred the judge to the relevant provision of the Rules of Evidence. Could the attorney be subject to discipline for his actions? A. Yes, because the lawyer was mixing personal matters with his representation of a client. B. Yes, because the lawyer communicated ex parte with a judge during the proceeding, without being authorized to do so by law or court order. C. No, because the note did not directly ask the judge to take a position on the merits of the case. D. No, because the lawyer did not speak to the judge directly, but instead gave a note to the clerk, who is not a judicial officer.

B. Yes, because the lawyer communicated ex parte with a judge during the proceeding, without being authorized to do so by law or court order. Rule 3.5(b) prohibits all ex parte communications with a judge during a proceeding. Assuming the objection and the judge's ruling are part of the trial record or transcript, the lawyer can appeal this decision (usually as part of an appeal of an unfavorable verdict after the trial). The purpose of this question is to highlight for students that lawyers cannot communicate ex parte with a judge even indirectly. Ex parte communications do not have to be in-person conversations between a lawyer and a judge—using a neutral intermediary is a violation of the rules as well.

A lawyer is defending a corporation in a civil personal injury lawsuit wherein a former employee filed a claim that they were injured while employed by the corporation. The plaintiff hires an expert witness who is a vocational rehabilitation expert to assess whether the plaintiff is able to continue future work. The corporation's lawyer calls the expert directly and asks him to consider several factors that would impact the plaintiff's potential future work providing those details. Opposing counsel was not present during this phone call but was made aware after the conversation by the corporation's lawyer. The local court pretrial discovery order prohibits ex parte contact with an opposing counsel's retained expert but allows scheduled depositions wherein all parties may be present. Did the corporation's lawyer potentially violate an ethical rule? A. No, because the lawyer let opposing counsel know after the conversation. B. Yes, because the lawyer knowingly disobeyed an obligation under the rules of a tribunal. C. No, because the lawyer is allowed to contact an expert witness on a pending case. D. Yes, if the lawyer reasonably believed the information is relevant.

B. Yes, because the lawyer knowingly disobeyed an obligation under the rules of a tribunal.

A prosecutor working in a very busy office feels her caseload is so heavy that she is unable to thoroughly review each case. She is just leaving a courtroom and sees a defense attorney in the hallway of the courthouse who asks about another case. She remembers the case is a misdemeanor charge against the defendant and vaguely remembers glancing at the police report and the criminal background check that showed a prior conviction. She offers a plea to the defense attorney. Has the prosecutor violated her ethical duty? A. No, because the prosecutor relied on the police report and background check. B. Yes, because the prosecutor's workload is too heavy to permit the independent assessment of the charge. C. No, because the prosecutor has reasonable confidence in the thoroughness of the fact finding in the police report. D. Yes, because the defendant is represented by counsel.

B. Yes, because the prosecutor's workload is too heavy to permit the independent assessment of the charge.

A client intends to purchase a parcel of real estate and retained an attorney to analyze the seller's title to the property. The attorney requests information from the seller regarding the seller's original acquisition of the property and obtains additional information from the local tax assessors and title registry. The attorney concludes that the seller does not have clear title to the property and informs the seller of this opinion when the seller asks him about it. The seller forbids the attorney to disclose the information to the prospective purchaser of the property and insists that he showed the attorney his documents about the original acquisition of the parcel with the understanding that the attorney would not say anything unfavorable. May the attorney inform the prospective purchaser of his opinion about the title? A. Yes, because remaining silent or withdrawing from representation at this point would make it easier for the seller to perpetrate a fraud on the purchaser. B. Yes, because the seller does not have a client-lawyer relationship with the attorney. C. No, because the attorney is bound by the duty of confidentiality to keep the information private. D. No, because the seller did not provide informed consent.

B. Yes, because the seller does not have a client-lawyer relationship with the attorney. Comment 2 for Rule 2.3 warns readers to pay special attention to which party is in fact the lawyer's client: The question is whether the lawyer is retained by the person whose affairs are being examined. When the lawyer is retained by that person, the general rules concerning loyalty to client and preservation of confidences apply, which is not the case if the lawyer is retained by someone else. For this reason, it is essential to identify the person by whom the lawyer is retained. In this problem, the purchaser retained the lawyer, not the seller. The seller is the one whose property the lawyer is examining, and the seller made some disclosures to the lawyer. For purposes of answering MPRE questions on Rule 2.3, it is important to pay close attention to which party retained the lawyer—sometimes it is the party under examination, but sometimes it is not. The lawyer does not have a duty of confidentiality to the seller. Of course, the lawyer should make clear to a non-client under examination that he represents the other party, so that the person does not make disclosures against their own interest under the misimpression that the lawyer has a duty to protect the information. Comment 2 adds, "This should be made clear not only to the person under examination, but also to others to whom the results are to be made available." The facts given in this problem are silent as to whether the lawyer made this clear to the seller.

A client wants to sell a parcel of commercial real estate, and he hired an attorney to represent him in the matter. As part of the representation, the client asked his attorney to prepare a thoroughly researched opinion memorandum concerning the title of the property, for the information of a prospective purchaser and the purchaser's prospective lender. The attorney gave the title opinion to the client, who gave it to the prospective purchaser, who in turn submitted it to the prospective lender. The prospective lender received and reviewed the attorney's title opinion but was not aware that the lawyer who prepared the title opinion represented the seller of the property rather than the buyer. Could the attorney be subject to discipline for failing to disclose explicitly in the memorandum what party he represents and that he has a duty of loyalty and confidentiality to the seller? A. Yes, because when a lawyer knows that third parties may rely on his written legal opinions, he has a diminished duty of loyalty or confidentiality to the original client. B. Yes, because the title opinion should identify the person by whom the lawyer is retained, and should make this clear not only to the client under examination, but also to others to whom the results are to be made available. C. No, because the lawyer's duty of loyalty, confidentiality, and candor runs only to the client who retained the lawyer. D. No, because everyone in a commercial real estate transaction presumes that title opinion letters from lawyers represent the best interest of the seller of the property.

B. Yes, because the title opinion should identify the person by whom the lawyer is retained, and should make this clear not only to the client under examination, but also to others to whom the results are to be made available. Comment 2 for Rule 2.3 requires that lawyers identify the client who retained the lawyer within the report or evaluation that third parties might rely on. The comment leads into this requirement by explaining that in some cases (as when there is a single prospective purchaser of a business or a parcel of real estate) the other party might misunderstand that the lawyer has a duty of loyalty to the seller instead. A lawyer may not disclose information to the non-client if the client has forbidden it, but the lawyer must disclose the identity of the client, to avoid misleading the other party in the transaction. In some situations, the other party may have been the first to request the report or evaluation (from the client), so it can become confusing for non-lawyers to see that the lawyer actually prepared the report for someone else, in the sense of the lawyer's duty of loyalty.

An attorney represents a client, who wants to sell his business. A prospective purchaser has required from the client an evaluation of the business' solvency, detailing its current liabilities, potential liabilities, revenue, and assets. The client provides the attorney with documents pertaining to each of these issues, and explains to the attorney in confidence that he has often understated the earnings of the business to avoid paying taxes on the business profits. Now he is concerned that the prospective purchaser will undervalue the profitability of the business and refuse to pay an appropriate price to purchase it. He asks the attorney to adjust the earnings figures upward by 25 percent, the same amount by which the client falsely lowered them in the corporate records, to portray the business accurately to the potential purchaser. The attorney finds this objectionable and prepares a report based on what the records say regarding the earnings and gives the evaluation directly to the purchaser. When the client learns about this, he explains to the prospective purchaser over the phone what happened. Despite the low reported earnings, the purchaser pays the client's asking price for the business, because of the client's truthful representations over the phone. Could the attorney be subject to discipline for his conduct in this matter? A. Yes, because even with the client's truthful disclosures about the earnings, the report does not account for the fact that the profits appear different than they would be if the business had paid its taxes. B. Yes, because under no circumstances is the lawyer permitted to knowingly make a false statement of material fact or law in providing an evaluation for a third party. C. No, because the lawyer's evaluation accurately represented the earnings reported in the corporate records. D. No, because the client's phone conversation with the purchaser ensured that the purchaser was not relying on false information when he made his decision.

B. Yes, because under no circumstances is the lawyer permitted to knowingly make a false statement of material fact or law in providing an evaluation for a third party. Comment 4 for Rule 2.3 expressly prohibits knowingly providing false information in a report to a non-client third party. This rule applies to false statements of law or statements of fact. The lawyer in this problem learned from the client that the business records had understated the earnings of the business—in other words, the client had falsified the records that formed the basis of the attorney's report. The attorney prepared a report based on records that he knew were misleading and gave the report directly to the other party. Even though the lawyer may appear scrupulous or honest at first, the lawyer violated the rule nonetheless and gave the seller a false report—he refused to adjust the numbers upward somewhat arbitrarily solely to make the business more appealing to the potential buyer. It also does not matter that the client explained the entire situation to the buyer after the fact and that the buyer was agreeable.

The third wife. An attorney represented a man twenty years ago in a divorce with the client's first wife. The client was a well-known local celebrity, a retired professional athlete who became a semi-successful actor and an outspoken advocate of a radical political cause. Recently, former client's third wife approached the same attorney seeking representation in obtaining a divorce from the former client. There are no children from the marriage—their children from previous marriages are now adults—and the distribution of assets will follow the terms of a carefully drafted prenuptial agreement between the former client and his third wife. The former client (that is, the husband) long ago provided written informed consent for future conflicts of interest if the attorney represented another party with adverse interests. The attorney does not believe that any confidential information learned from representing the husband twenty years ago in his first divorce will be relevant to the pending third divorce. On the other hand, there is regular media coverage of the husband's trysts and on-and-off sexual relationships with various actresses and female socialites in the area, and marital infidelity could trigger certain exception clauses in the prenuptial agreement. Would it be proper for the attorney to use the information about husband's recent indiscretions in representing the third wife? A. Yes, if the husband's new lawyer provides written informed consent to the use of the information in the divorce proceeding. B. Yes, the fact that a lawyer has once served a client does not preclude the lawyer from using generally known information about that client when later representing another client. C. No, not if the attorney learned confidential information about the husband having a pattern of marital infidelity during his prior representation of Husband. D. No, the fact that there is a prenuptial agreement with exceptions triggered by marital infidelity should preclude the attorney from using such information.

B. Yes, the fact that a lawyer has once served a client does not preclude the lawyer from using generally known information about that client when later representing another client. This question involves the use of the husband's recent indiscretion against him. Remember, the rule states that a lawyer may not use an ex-client's confidential information against the client. So the issue is whether the husband's recent indiscretion is considered confidential. B is the correct answer because it recognizes that Husband's recent indiscretion is not confidential. The information is public information that is regularly disseminated in the media. Since it is public, the attorney does not need the husband's written informed consent to use the husband's recent indiscretion against him.

An attorney is representing the defendant in a highly publicized criminal trial. On his way into the courthouse on the day of jury selection, reporters gather around the attorney hoping for comments. The attorney explains that the prosecutor already held a press conference in which she shared that the defendant had refused to take a polygraph test, that DNA tests had confirmed the defendant's guilt, and that the defendant had refused several offers of guilty pleas. To set the record straight before trial, the defense attorney explains that his client had already agreed to take a polygraph test but that none had occurred. He adds that defense experts would testify about problems with the DNA tests, and that the plea offers had all been the same (a life sentence instead of the death penalty) and were unacceptable to the client. Were the defense attorney's statements proper? A. Yes, because the First Amendment and Sixth Amendment protect a defendant's right to defend himself publicly through his attorney against false accusations. B. Yes, when others have publicly made prejudicial statements, responsive statements may have the salutary effect of lessening any resulting adverse impact on the adjudicative proceeding. C. No, because there is a presumption of prejudicial effect on the proceedings when a lawyer comments publicly about the possibility of a guilty plea, or a party's refusal to confess to a crime. D. No, because there is a presumption of prejudicial effect on the proceedings when a lawyer comments publicly about the performance or results of any examination or test or the refusal or failure of a person to submit to an examination or test.

B. Yes, when others have publicly made prejudicial statements, responsive statements may have the salutary effect of lessening any resulting adverse impact on the adjudicative proceeding. Comment 7 for Rule 3.6 says, "When prejudicial statements have been publicly made by others, responsive statements may have the salutary effect of lessening any resulting adverse impact on the adjudicative proceeding." Rule 3.6(c) specifically permits lawyers to engage in "self-defense" disclosures to the media. Even statements that would have been impermissible for the lawyer to make to the media could be proper if offered solely to offset negative publicity (especially negative or false information improperly disseminated by the opposing side); the other side has opened the door, so to speak. In this problem, the prosecutor may have violated several of the provisions of Comment 5 to Rule 3.6. It is proper, therefore, for the lawyer to give reporters specific denials in response.

Bill, a state prosecutor, goes to see Defendant Jay, who will soon be tried for assault. Jay refused his court-appointed attorney in favor of self-representation. Bill knows that Jay makes a very sympathetic witness and is worried about going to trial. Bill would like to advise Jay to forgo the jury in favor of a bench trial. What advice can Bill give Jay?

Bill cannot advise Jay to give up a jury trial. As an unrepresented accused, Bill has a special duty to protect Jay and to not have Jay waive substantial trial rights. MR 3.8(c).

Former prosecutor. An attorney began her career as a prosecutor at the District Attorney's Office. During her tenure as a prosecutor, she brought charges against an individual suspected of sending ricin, a deadly toxin, in an envelope to a prominent politician, apparently in an unsuccessful attempt to assassinate the public official. The jury found the evidence too attenuated, and it acquitted the defendant. Shortly thereafter, another person, who was a member of a terrorist organization, confessed to sending the ricin and provided extensive evidence of his plot to kill the politician to make a political statement. The attorney resigned from the District Attorney's Office, partly out of humiliation over this case, and went into private practice. Eighteen months later, the accused individual decides to sue the government over wrongful arrest, slander, libel, and wrongful prosecution over the case in which he obtained an acquittal. The attorney feels that her superiors at the D.A.'s Office had pressured her to press charges in order to satisfy the public uproar over the ricin letters, despite having scanty evidence that the accused individual was actually guilty. The attorney offers to represent the accused individual in his lawsuit against the government, partly to make amends or atone for her role in what she now views as an abuse of government power and a great injustice. Would it be proper for the attorney to handle this case, given her good intentions? A. Yes, because the test for determining if an improper conflict of interest exists between former clients and a new client is the lawyer's subjective motivations in undertaking the new representation, and in this instance, the attorney is merely trying to make amends for her past mistakes. B. Yes, because the attorney has a duty to repudiate her previous wrongful actions, and her representation of the individual will send a strong message to other prosecutors, which in turn serves the public interest. C. No, a lawyer who has prosecuted an accused person could not properly represent the accused in a subsequent civil action against the government concerning the same transaction. D. No, because a prosecutor cannot ethically "switch sides" and start representing criminal defendants in public practice, regardless of whether the same individuals are involved as clients.

C. No, a lawyer who has prosecuted an accused person could not properly represent the accused in a subsequent civil action against the government concerning the same transaction. C is the correct answer because it recognizes the conflict of interest. The attorney cannot represent the accused individual because the accused individual wants to sue the state for the same case that the attorney trialed.

An attorney spent several years working for the state Office of the Attorney General in its environmental litigation division. While there, the attorney began a case against a scrap metal facility for burying toxic materials on its grounds. The attorney then left government service and went to work for Big Firm. There, the attorney began representing a group of neighboring landowners in a lawsuit against the same scrap metal facility over the same burying of toxic material, as it had polluted the groundwater and had migrated to adjacent properties underground. Is it proper for the attorney to represent these plaintiffs? A. Yes, so long as the new clients provide written informed consent. B. Yes, because the new clients' interests match those of the attorney's government employer, and there is no indication of adverse interests being present between them. C. No, a lawyer who has pursued a claim on behalf of the government may not pursue the same claim on behalf of a later private client after the lawyer has left government service, except when authorized to do so by the government agency. D. No, because the attorney is using the prestige of having worked in government service to attract new clients, which creates a conflict between the attorney's self-interest and the public interest represented by the government agency.

C. No, a lawyer who has pursued a claim on behalf of the government may not pursue the same claim on behalf of a later private client after the lawyer has left government service, except when authorized to do so by the government agency. C is correct because it states that normally a lawyer may not represent a client under these circumstances whether or not the lawyer obtains written informed consent from the government agency.

A client repeatedly calls an attorney to discuss her pending divorce case. The client wants above-guideline child support, alimony, and a large percentage of the estate, even though the parties have only been married two years. The attorney has continuously given his honest opinion about what he believes the client is eligible to receive, and what he believes she may receive in the divorce based on his experience. The client has recently become angry with the attorney because she is unhappy with his opinion. She has even asked, "Are you working for me or my husband?" In an effort to keep the client happy, the attorney begins to tell the client what he believes she is eligible to receive when she asks, but simply states "the court will decide" when the client asks the attorney what he believes she will receive. Are the attorney's actions proper? A. Yes, the attorney may respond to a client's requests for the attorney's opinion in any manner that will maintain the client's morale, including refusing to give advice if the attorney believes the client will not be accepting of his advice. B. Yes, attorneys are not required to give their opinions or advice, but may, at any time, respond to clients by referring them to the appropriate legal authority or by advising them that the court will ultimately decide the issue, if applicable. C. No, an attorney should give his honest opinion about the case when asked, even if the opinion is unsatisfactory to the client. D. No, attorneys should always give advice to clients that encourages the client to have confidence in the client's position.

C. No, an attorney should give his honest opinion about the case when asked, even if the opinion is unsatisfactory to the client. Rule 2.1 requires lawyers to provide candid advice to clients. Here, the lawyer began to appease a difficult client with a vague, almost evasive answer. It is understandable why a lawyer would feel pressured to tell a client what she wants to hear, but that is exactly why we have a rule mandating that lawyers give their clients a realistic picture of the situation.

The former mediator. An attorney, who often serves as a court-appointed mediator, was appointed to mediate the divorce case between a husband and wife. The case settled in mediation and the divorce was finalized soon after. A year later, the husband sought to retain the attorney to represent him in a modification suit against his wife. The attorney accepted the case and sent a letter to the wife advising her that the attorney had been retained by the former husband to represent him in a modification suit. Are the attorney's actions proper? A. Yes, the attorney who previously served as a third-party neutral may represent any party in a suit connected to the previous matter if the attorney provides proper notice to the other party in writing. B. Yes, an attorney who previously served as a third-party neutral may represent any party in a suit connected to the previous matter if the previous case occurred more than one year before the third-party neutral began representation of one of the parties. C. No, an attorney who previously served as a third-party neutral is required to obtain informed consent, confirmed in writing, from all parties to the proceeding prior to representing a party in a suit connected to the previous matter. D. No, an attorney who previously served as a third-party neutral shall not represent any party in a suit connected to the previous matter.

C. No, an attorney who previously served as a third-party neutral is required to obtain informed consent, confirmed in writing, from all parties to the proceeding prior to representing a party in a suit connected to the previous matter. C is correct because it correctly states the rule and what the attorney will be required to do before commencing representation of Husband.

The law student intern. Years ago, as a law student, an attorney worked on a case for the client during a law firm internship. Now, the attorney's firm is representing a defendant in a lawsuit in which the client is the plaintiff. The client's new lawyer moves to disqualify the attorney's firm from the representation when it learns that the attorney worked for another firm on behalf of the client when the attorney was still a law student. Is the attorney's firm subject to disqualification in this case? A. Yes, because when lawyers are associated in a firm, none of them shall knowingly represent a client when any one of them practicing alone would be prohibited from doing so. B. Yes, unless both parties provided written informed consent and waived the conflict of interest at the beginning of representation. C. No, as long as the firm screens the attorney from any personal participation in the matter to avoid communication to others in the firm of confidential information that both the non-lawyers and the firm have a legal duty to protect. D. No, because the attorney was not yet a lawyer during the law student internship, and therefore did not actually provide legal representation for the client in the previous matter.

C. No, as long as the firm screens the attorney from any personal participation in the matter to avoid communication to others in the firm of confidential information that both the non-lawyers and the firm have a legal duty to protect. C is correct because the events that give rise to the conflict of interest occurred while the attorney was a non-lawyer. So long as the firm screens him from personal participation and confidential information, the firm will not be subject to disqualification.

An attorney in a state that has adopted the Model Rules in their current form enters into a fee-sharing agreement with a lawyer admitted in Washington, D.C., which permits fee sharing with non-lawyers and multidisciplinary practices. They collaborate on a case and divide the fees as agreed. The attorney from the Model Rules state is aware that the other attorney will share his part of the fees with non-lawyers in the D.C. office; in fact, the D.C. lawyer's firm has accountants who hold an ownership share in that firm. Is the non-D.C. attorney subject to discipline for indirectly sharing legal fees with non-lawyers, given that he practices in a state that forbids fee sharing with non-lawyers? A. Yes, the attorneys have a duty to uphold the rules in their own jurisdiction, and given that the attorney knows that the other lawyer will share some of the fees with non-lawyers, he has violated the rule in his own state. B. Yes, but only because the lawyer had actual knowledge of the fee-sharing arrangement. C. No, as long as the first attorney shares fees only with another attorney, it does not matter if the other attorney shares fees with non-lawyers as permitted by his home jurisdiction. D. No, because the rule in the attorney's own state, prohibiting fee sharing with non-lawyers, is unconstitutional, according to the Supreme Court.

C. No, as long as the first attorney shares fees only with another attorney, it does not matter if the other attorney shares fees with non-lawyers as permitted by his home jurisdiction. Traditionally, the American legal system absolutely forbade law firms (or firms providing legal representation) having non-lawyer owners (partners or shareholders). Washington, D.C. is the one jurisdiction in the country that permits this in a limited way. This presents problems when lawyers from a D.C. firm collaborate on a case with lawyers from another state (Virginia, Maryland, etc.) and share fees. Thus, the ABA addressed this problem in Formal Opinion 13-464, saying that lawyers from other states may share fees with a lawyer in D.C., even if that lawyer will in turn share them with a non-lawyer partner—as long as the lawyer from the other state does not pay the non-lawyer directly.

A client retained an attorney to represent him in a car accident case. The client sought to recover $5,000 for damage to his vehicle and a few medical expenses the client incurred because of the accident. The attorney failed to timely file a lawsuit for the client before the statute of limitations ran. After realizing that the suit was barred because the attorney failed to timely file, the attorney sent the client a letter with a check for $20,000 and an agreement for the client to sign and return. The agreement stated that keeping the $20,000 check constituted acceptance of the agreement and that acceptance of the agreement included releasing the attorney for any malpractice claims against the attorney. Is the attorney's conduct proper? A. Yes, attorneys can settle claims or potential claims for malpractice so long as the settlement amount is reasonable. B. Yes, attorneys can settle claims or potential claims for malpractice as long as the agreement terms are provided to the client in writing and the settlement amount is reasonable in relation to what the client would expect to receive. C. No, attorneys must advise the client of the importance of obtaining advice of independent counsel and provide reasonable time for the client to obtain such counsel prior to settling a claim or potential claim for malpractice. D. No, attorneys cannot settle claims or potential claims for malpractice with clients.

C. No, attorneys must advise the client of the importance of obtaining advice of independent counsel and provide reasonable time for the client to obtain such counsel prior to settling a claim or potential claim for malpractice. Rule 1.8(h)(2) requires lawyers, when settling a claim (or potential claim) for legal malpractice with a former client or an unrepresented person, to advise the person in writing to seek independent legal counsel in connection with settling the claim, and to give the person a reasonable opportunity (that is, enough time) to do so. The attorney in this case did not advise the former client to seek independent legal counsel and did not provide a reasonable opportunity to do so.

An attorney had a license to practice law in two jurisdictions—his home state where he lived and had his main office, and a neighboring state where he represented several clients each year. The attorney committed serious professional misconduct in his home state and received a public reprimand from the state disciplinary authorities. All the conduct took place in his home state, the client resided in the state, and the representation took place entirely within his home state. The lawyer's conduct would have violated the rules in either of the jurisdictions where he had a license to practice law, because it involved commingling client funds with his own money, and the states had nearly identical rules concerning this activity. After the attorney received a public reprimand in his home state, where the misconduct occurred, the state bar disciplinary authority in the neighboring state (where he also practiced) then commenced disciplinary proceedings against him as well. In the end, the neighboring state bar suspended his license for six months in that state, a much more severe sanction than the public reprimand he received in his home state, where the misconduct actually occurred. The attorney claims that the neighboring state bar has no jurisdiction over conduct that occurred entirely outside of the state. He also objects that the second punishment raises double jeopardy concerns. Is the attorney correct? A. Yes, because even in cases where a second state can administer discipline over the same conduct, double jeopardy rules prevent the second tribunal from imposing a more severe sanction than the first tribunal already imposed on the lawyer. B. Yes, because a lawyer cannot be subject to the disciplinary authority of two jurisdictions for the same conduct if it occurred entirely within one state. C. No, because a lawyer may be subject to the disciplinary authority of two jurisdictions for the same conduct, and may receive different sanctions in each state. D. No, because choice-of-law rules require that each state impose the same sanction.

C. No, because a lawyer may be subject to the disciplinary authority of two jurisdictions for the same conduct, and may receive different sanctions in each state. Rule 8.5(a) says, "A lawyer may be subject to the disciplinary authority of both this jurisdiction and another jurisdiction for the same conduct." If a lawyer has a license to practice in multiple states, a violation committed anywhere could be the basis for a disciplinary action in any of those states, if it violates the professional rules in that jurisdiction. Disciplinary actions are civil-administrative in nature, not criminal, so double jeopardy does not apply.

An attorney represented a newspaper publisher in a defamation case brought by a popular actor. A radio talk show invited the attorney to participate in their afternoon program and respond to calls from the radio listeners. The first caller asked the attorney to explain the case involving the superhero that the popular actor had played in a recent film. The attorney mentioned that the actor (using the actor's legal name as it appeared in the pleadings, rather than his stage name or the character for which the actor was most famous), and the legal name of the publisher the attorney represented. He also explained that the lawsuit was over alleged defamation by the newspaper, and that the newspaper planned to raise an affirmative defense of truth, that is, it would attempt to show that the stories it printed about the actor were factually accurate, even if they were unflattering. The attorney also mentioned that the actor owns a home and a business in the state, which is a matter of public record, and this is the reason the case is in the courts in that state. Did the attorney violate the Rules of Professional conduct by making these statements on a radio talk show program? A. Yes, because a lawyer who is participating or has participated in litigation shall not make an extrajudicial statement that the lawyer knows, or reasonably should know, will be disseminated by means of public communication. B. Yes, because he explained that his side would assert the truth of the unflattering stories it published, which could prejudice the upcoming proceedings, and revealed where the actor lives. C. No, because a lawyer may state the claim, the defense involved, the identity of the persons involved, and matters in the public record. D. No, because a lawyer has a right to explain his client's side of the story and defend his client in public when the client has faced the stigma of a lawsuit.

C. No, because a lawyer may state the claim, the defense involved, the identity of the persons involved, and matters in the public record. Rule 3.6(b)(1) and (2) allow a lawyer to make statements to the media about "the claim, offense or defense involved and, except when prohibited by law, the identity of the persons involved" and "information contained in a public record." In this problem, all of the information the lawyer shares during the radio interview comes under these provisions—the claims, defenses, identities of the parties, and matters already in the public record (the real property owned in that state). Watch for questions on the MPRE that test the specific exceptions to the general rules.

Emergency room patients. An attorney is dating a woman whose sister works as a nurse in a hospital emergency room. The attorney gives the nurse, his girlfriend's sister, a stack of his business cards and law firm brochures, and offers to pay her $200 for any clients who hire him because of her referrals, with the understanding that she will not refer patients to any other lawyers. The nurse recommends several patients per month to the attorney for representation in personal injury claims, and one or two per month actually hire the attorney to represent them. Is such an arrangement proper? A. Yes, because the nurse is closely related to the attorney, given that the attorney is dating her sister. B. Yes, because the attorney is not paying the nurse on a contingent fee basis. C. No, because a lawyer shall not give anything of value to a person for recommending the lawyer's services, with certain exceptions not applicable here. D. No, because the fact that the attorney is dating her sister creates a conflict of interest if the nurse refers clients to the attorney.

C. No, because a lawyer shall not give anything of value to a person for recommending the lawyer's services, with certain exceptions not applicable here. Rule 7.2(b) prohibits paid personal referrals, although there is an exception for non-exclusive reciprocal referral arrangements between lawyers and other professionals. In this question, the lawyer is paying the nurse $200 for each referral that turns into an actual client for the lawyer, which violates Rule 7.2. Of course, it would be appropriate to have a reciprocal referral arrangement with a nurse (another professional for purposes of the rule), but only if it is non-exclusive. The facts here say that the nurse promised not to refer patients to any other lawyers, so this exception does not apply (and probably would not apply due to the payments, unless those were reciprocal as well).

The hearing officer. An attorney was a state hearing officer for the Workers Compensation Board. The attorney left that position and opened his own law firm, primarily representing parties before the state Workers Compensation Board. One of the cases is the final rehearing of a case in which the attorney had presided as hearing officer at an initial preliminary hearing and ruled on preliminary matters, but the attorney left the Board without issuing any final decision in the case and the Board transferred the matter to another hearing officer. The attorney represents the injured worker, the client. All the parties involved give informed consent, confirmed in writing, for the attorney to represent the client. Is the attorney subject to discipline for representing the client in this matter? A. Yes, because a lawyer shall not represent anyone in connection with a matter in which the lawyer participated personally in a substantial way as a judge or other adjudicative officer. B. Yes, because the type of conflict of interest described here is nonconsentable, so it is irrelevant that all the parties provided informed written consent. C. No, because all the parties involved provided informed written consent to the representation, despite the obvious conflicts of interest at stake. D. No, the conflict-of-interest rules do not apply to merely administrative hearing officers who are not actual judges, arbitrators, or mediators.

C. No, because all the parties involved provided informed written consent to the representation, despite the obvious conflicts of interest at stake. C is correct because it correctly states what the attorney must do to continue representation of the client.

A client asked an attorney to represent him in a lawsuit. The attorney conducts some preliminary research and quickly discovers that the lawsuit is a long shot. In fact, based on the attorney's survey of the existing judicial decisions in very similar cases, the attorney estimates that they have only a 15 percent chance of winning, and it will depend on an extraordinarily lopsided jury, a strongly partisan judge whose political leanings go in their favor, as well as a mediocre lawyer representing the other side. Otherwise, all things being equal, the attorney advises the client that he is about 85 percent certain that they will not prevail. The client is willing to take risks, however, and urges the attorney to take the matter. The attorney reluctantly agrees, on the condition that he can charge a higher fee than usual, and files the lawsuit. Could the attorney be subject to discipline for bringing a frivolous claim? A. Yes, because the attorney knows from his research that the claim is very unlikely to prevail and is therefore wasting the court's time. B. Yes, because he should not have charged a higher fee in a case where the client is already facing unfavorable odds of winning, as this puts the client into an even worse position. C. No, because an action is not frivolous even though the lawyer believes that the client's position will not prevail in the end. D. No, because the client should control the overall objectives of the representation, even if the lawyer controls the specific strategies, methods, and tactics.

C. No, because an action is not frivolous even though the lawyer believes that the client's position will not prevail in the end. Comment 2 for Rule 3.1 says that a lawsuit "is not frivolous even though the lawyer believes that the client's position ultimately will not prevail. The action is frivolous, however, if the lawyer is unable either to make a good faith argument on the merits of the action taken or to support the action taken by a good faith argument for an extension, modification, or reversal of existing law." The mere fact that a potential lawsuit is a "long shot" does not mean that it is frivolous. For many clients, bringing or defending a case that has a small chance of winning might still be worthwhile, if the potential payoffs (discounted for the probabilities) outweigh the expected costs.

Just a hunch. An attorney suspects that another lawyer in his firm has violated the Rules of Professional Conduct in a rather serious matter, but has no first-hand knowledge of the situation—his suspicion rests on the fact that the other lawyer seems to be acting paranoid and evasive, and a number of strange coincidences have occurred in his cases. Does the attorney who suspects something seriously wrong is afoot have a duty to report the other lawyer to the state bar disciplinary authority? A. Yes, but he must make an anonymous complaint to the state bar. B. Yes, because a lawyer who knows of a violation of the rules that raises serious questions about the other attorney's honesty must report it to the state disciplinary authority. C. No, because he does not have actual knowledge of the violation. D. No, because lawyers do not have to report violations by other attorneys at their own firm, which would create internal divisions and mistrust between partners.

C. No, because he does not have actual knowledge of the violation. Rule 8.3 requires actual knowledge of the wrongdoing in order to trigger the mandatory reporting requirement: "A lawyer who knows that another lawyer has committed a violation of the Rules of Professional Conduct . . ." (emphasis added). The definition section in Rule 1.0 defines "knows" as denoting "actual knowledge of the fact in question. A person's knowledge may be inferred from circumstances." In this case, the lawyer has suspicion but not actual knowledge as defined in the Model Rules.

An attorney interviewed an expert witness whom he thought he might hire to testify at a client's trial. The attorney explained he was meeting with several expert witnesses and would hire the one who he thought would seem most persuasive to the jury. The expert witness offered to work on a contingent fee basis; if the attorney did not win the case at which the expert testified, then no fee would accrue. The attorney would have to pay the expert witness only if his testimony were compelling enough to produce a favorable outcome in the case. The attorney thought that this would give the expert an incentive to prepare more thoroughly for trial, and that it would be fairer to the client, who would be left bankrupt if they lost at trial and would have trouble paying the expert's fee anyway. Would it be proper for the attorney to hire the expert witness under such terms? A. Yes, it is permissible to pay an expert witness a large fee. B. Yes, because if the client loses the case and would be unable to pay the fees to the lawyer and the expert, the same type of contingency would result either way. C. No, because it is improper to pay the expert witness a contingent fee. D. No, because a lawyer cannot offer any inducement to a witness to testify.

C. No, because it is improper to pay the expert witness a contingent fee. Comment 3 for Rule 3.4 permits lawyers to pay experts to testify (customary fees), as long as it is not a contingent fee, i.e., the fee or the amount of the fee depends on whether the case outcome is favorable. This includes the "if-you-don't-win-you-don't-pay type of contingent fee described here, as well as tiered fees that pay more if the outcome is favorable, or pay a percentage of the total award at trial. The concern is that we do not want experts to have a perverse incentive to distort their findings or conclusions. Of course, experts are aware that they are more likely to have repeat business if their testimony is both favorable and convincing.

The victim. An attorney works for a firm where another lawyer is representing the defendant in a personal injury lawsuit. The other lawyer has represented the defendant for a long time on unrelated, non-litigation matters, but the personal injury lawsuit is a new case. The victim, the plaintiff in the same personal injury lawsuit, was a college classmate of the attorney and he asks the attorney to represent him in the litigation. The attorney has not learned any confidential information yet about the defendant from his fellow associate at the firm, nor has the attorney learned any confidential information from the victim during their preliminary consultation. The firm decides to undertake the representation of the victim as well. The firm will carefully screen the attorney and lawyer from one another, forbidding them to discuss the case with each other or anyone else in the office, and ensuring that they do not have access to each other's files for the case. In addition, neither lawyer will receive a bonus from the fees received for this litigation. Under the Rules of Professional Conduct, is it proper for the attorney to represent the victim, given these circumstances? A. Yes, as long as the firm provides notice to the defendant and the victim about the specific screening procedures it has in place, and gives periodic certifications of compliance with the screening procedures. B. Yes, as long as both the clients provide written informed consent to the conflict of interest, after receiving a detailed explanation of the problems with common representation, and neither party has its fees paid by a third party. C. No, because the Rules of Professional Conduct impute the conflict of the other lawyer to the attorney, and screening procedures do not apply to conflicts between current clients. D. No, unless the attorney has already represented the victim in previous unrelated matters while working at another law firm, and joined the new law firm only recently.

C. No, because the Rules of Professional Conduct impute the conflict of the other lawyer to the attorney, and screening procedures do not apply to conflicts between current clients. C is correct because the defendant is a current client of the firm with a directly adverse interest to the victim. The representation of the victim in this situation would violate the duty of loyalty owed to the defendant.

An attorney worked for several years for a federal government agency in regulatory enforcement. Big Firm then hired the attorney for a substantially higher salary, and the attorney accepted the position and left her government position. One of the attorney's first assigned cases at Big Firm was a new action by the client against the same government agency for which the attorney had previously worked, challenging the constitutionality of a new regulation that the agency had recently promulgated. While at the agency, the attorney had not been involved with the review and promulgation of any new regulations, including the one at issue in the client's challenge, but instead worked exclusively on enforcement litigation matters. Is the attorney subject to disqualification in the client's matter against the attorney's former employer? A. Yes, because Big Firm gave the attorney an unreasonably large salary increase for leaving her public service position and joining Big Firm in the private sector, which creates a conflict of interest. B. Yes, unless the federal government agency is willing to provide written informed consent to the attorney's representation in the case. C. No, because the attorney did not participate personally and substantially in the matter as a public officer or employee. D. No, because the case involves a constitutional challenge to the validity of a regulation, not the financial interests of the client or government as would be recognized if the case involved fines, fees, or penalties.

C. No, because the attorney did not participate personally and substantially in the matter as a public officer or employee. C is correct because it properly states that if the attorney was not personally and substantially involved with the current matter during her previous employment at the government agency she may represent the client.

An attorney worked for several years for a federal government agency in regulatory enforcement. Big Firm then hired the attorney for a much higher salary, and the attorney accepted the position and left her government position. One of the attorney's first assigned cases at Big Firm was a new action by the client against Conglomerate Corporation. The attorney had worked on an enforcement against Conglomerate Corporation and learned confidential government information about the entity during the litigation, but the attorney does not know, and has no reason to know, that the information is confidential government information. The attorney is under the reasonable impression that all the information she learned about Conglomerate Corporation is now public information. The government agency gave its informed consent, confirmed in writing, to the representation. Is the attorney nevertheless subject to disqualification in the client's matter against the attorney's former employer? A. Yes, the attorney has confidential government information about a person acquired while working for the government agency, and therefore may not represent a private client whose interests are adverse to that person in a matter in which the information could be used to the material disadvantage of that person. B. No, because the appropriate government agency gave its informed consent, confirmed in writing, to the representation. C. No, because the attorney does not have confidential government information about Conglomerate that she knows is confidential government information. D. Yes, because the attorney did not previously represent the client or Conglomerate Corporation, so there is no attorney-client privilege or conflict of loyalties here between two clients that the attorney is representing or has represented.

C. No, because the attorney does not have confidential government information about Conglomerate that she knows is confidential government information. C is correct because the attorney has a reasonable belief that what she learned about Conglomerate was public information and therefore does not know that it is confidential.

An attorney has a firm in a state in which the attorney lacks a license to practice law. The attorney's legal work, however, consists entirely of representing local inventors before the United States Patent and Trademark Office in Washington, D.C., either by correspondence or by traveling to appear there in patent proceedings. A relevant federal statute states that non-lawyers may represent patent applicants before the USPTO. The attorney does no other legal work for clients—if clients need representation for family law matters, employment matters, incorporating businesses, or personal injury suits, the attorney refers them to outside counsel. All the attorney's clients, however, are located in the state where the firm has its office, and the attorney is unlicensed there. Is the attorney subject to discipline? A. Yes, because the attorney is regularly engaged in the unauthorized practice of law in that state. B. Yes, because all the clients reside in a state where the attorney is unlicensed. C. No, because the attorney is providing services authorized by federal law, which preempts state licensing requirements. D. No, because the attorney has specialized in a single area of law and refers all other matters to outside counsel.

C. No, because the attorney is providing services authorized by federal law, which preempts state licensing requirements.

An attorney represented a client in tort litigation against a pharmaceutical company over injuries allegedly resulting from one of the company's drugs. During a pretrial hearing about the admissibility of certain evidence, the court ruled against the attorney and ordered that the evidence was inadmissible at trial. The attorney then contacted a reporter from a prominent newspaper and gave him a lengthy interview explaining the case, discussing the upcoming trial, and giving the reporter the very evidence that the court had held should be inadmissible at the trial. The newspaper ran the story on the same day that jury selection began for the trial. Opposing counsel moved to disqualify the attorney due to misconduct in the matter, that is, the public disclosure of the inadmissible material in an attempt to taint the jury pool. The court agreed to disqualify the attorney on the eve of the trial. Another firm was already representing the client as co-counsel, so that firm agreed to continue with the trial work alone. The attorney filed an interlocutory appeal, which he lost at the appellate court and appealed to the Supreme Court. Delaying the trial with this interlocutory appeal was clearly against the client's interest, but it was necessary for the attorney to continue to handle this big case. Is it proper for the attorney to recommend to the client that they appeal his disqualification, if it is not clearly in the client's interest to do so? A. Yes, because the lawyer's interests and the client's interests presumptively align in litigation. B. Yes, because the other lawyer might not obtain as favorable a result for the client as the attorney would. C. No, because the decision to appeal should turn entirely on the client's interest. D. No, because the disqualification was for lawyer misconduct rather than a conflict of interest.

C. No, because the decision to appeal should turn entirely on the client's interest. Rule 2.1 requires lawyers to exercise independent professional judgment, which includes separating the lawyer's own interests from what would be best for the client. The Supreme Court addressed a situation like this in Richardson-Merrell, Inc. v. Koller, 472 U.S. 424, 435 (1985).

An attorney represents a client before a government agency that enforces securities regulations. As part of the representation, the attorney must prepare an opinion concerning the legality of the securities registered for sale under the securities laws, for submission to the government agency, which requires such reporting. The client authorizes the attorney to prepare the written opinion but insists that the attorney exclude any mention of a particular business loss the client's company incurred recently, in order to avoid upsetting the shareholders. To preserve the client's confidential information, the attorney prepares the written opinion without the information the client asked him to withhold. The report does not mention that it excludes some unfavorable information. The attorney prepares the written opinion and gives it to the client, who submits it to the agency. Is it proper for the attorney to follow the client's instructions in preparing this report? A. Yes, because when the lawyer represents the person whose affairs are under examination, the general rules concerning loyalty to client and preservation of confidences apply. B. Yes, because it is the client's decision what to disclose to the agency, and the client alone will bear the consequences if the agency concludes later that the client submitted a misleading report. C. No, because when a lawyer's report categorically excludes certain issues or sources, then the lawyer must describe in the report any such limitations that are material to the evaluation in the report. D. No, because an attorney has a duty to include in the report whatever information the government agency requested, as the agency will rely upon the report in making its decisions.

C. No, because when a lawyer's report categorically excludes certain issues or sources, then the lawyer must describe in the report any such limitations that are material to the evaluation in the report. Comment 4 for Rule 2.3 says that when lawyers prepare a report or evaluation for use by non-client third parties, "certain issues or sources may be categorically excluded, or the scope of search may be limited by time constraints or the noncooperation of persons having relevant information." The report must describe "any such limitations that are material to the evaluation." A lawyer must honor the client's wishes to limit the coverage of a report or evaluation, but the limitation must receive mention in the report, so that third parties relying upon the report will not find it misleading.

The secretary's husband. A legal secretary in a law firm is married to the owner of an independent retail-clothing store. The firm undertakes representation of a clothing wholesaler, who is suing the same independent clothing store over nonpayment for shipments of merchandise. The legal secretary's husband hires another firm to represent his store in the lawsuit, and his lawyer asks the court to disqualify the legal secretary's firm because of her position there. Should the firm be subject to disqualification? A. Yes, because the conflict of interest is too great where the defendant's spouse works for opposing counsel's firm. B. Yes, because the lawsuit involves nonpayment for a shipment of merchandise, and the legal secretary indirectly benefited from her husband keeping these unpaid funds. C. No, if the firm screens the legal secretary from any involvement in the case or from access to any confidential information about the case. D. No, because the legal secretary is not a lawyer, so the Rules of Professional Conduct do not apply to her personal conflicts of interest.

C. No, if the firm screens the legal secretary from any involvement in the case or from access to any confidential information about the case. C is correct because it correctly states that a non-lawyer's conflict of interest does not prohibit representation.

The abortion clinic. An attorney works for a firm. She also describes herself as an outspoken advocate for the rights of unborn children, that is, she passionately favors legal restrictions on abortion. A local abortion clinic asks the firm to represent it in litigation over recent zoning measures that would significantly limit its hours of operation and therefore the number of clients the clinic could accept. The firm agrees to the representation. The attorney firmly refuses to have any part in the representation, and though no formal screening measures are in place, everyone else in the firm avoids discussing the case with her or around her because they are afraid of receiving another lecture about the wrongfulness of abortion. Early in the litigation, the judge considers disqualifying the firm because it employs the attorney, who has a reputation in the community for her advocacy against legalized abortion. Neither the clinic nor the opposing party (the municipal zoning authority) provided written consent to a conflict of interest. Should the firm be subject to disqualification in this case? A. Yes, because the firm did not implement formal screening measures to ensure that the attorney receives no confidential information about the case and cannot influence the other lawyers working on the case. B. Yes, because the firm did not obtain informed written consent from both parties to the potential conflict of interest. C. No, the firm should not be disqualified where one lawyer in a firm could not effectively represent a given client because of strong political beliefs, but that lawyer will do no work on the case and the personal beliefs of the lawyer will not materially limit the representation by others in the firm. D. No, because preserving women's access to legalized abortion is such an important fundamental right that it would be improper to limit the abortion clinic's options for representation in the matter, and other firms may also have conflicts of interest that would preclude representation.

C. No, the firm should not be disqualified where one lawyer in a firm could not effectively represent a given client because of strong political beliefs, but that lawyer will do no work on the case and the personal beliefs of the lawyer will not materially limit the representation by others in the firm. C is correct. The other lawyers in the firm may represent Abortion Clinic because the attorney refuses to work on the case and the attorney's personal belief on abortion is not limiting the other lawyers' ability to effectively represent Abortion Clinic.

In preparing a DUI defense on behalf of her client, an attorney's investigator interviewed the bartender at a local pub. The bartender told the investigator about the client's drinking habits at the pub. The investigator subsequently reported the bartender's comments to the attorney. The bartender's comments are protected by A. Both Rule 1.6 and attorney-client privilege. B. Attorney-client privilege only. C. Rule 1.6 only. D. Neither attorney-client privilege nor Rule 1.6.

C. Rule 1.6 only. The correct answer is C. The communication is confidential under the broad protections of Rule 1.6. Attorney-client privilege applies to communications between the attorney and client (with narrow exceptions). Answers A and B are incorrect because attorney-client privilege protects communications between the attorney and client; it does not typically include communications by a third party.

An attorney is in-house counsel for a large international corporation and has daily contact with higher-level executives and managers. One day, a senior executive mentions casually to the attorney that he has offered lucrative stock options, worth millions of dollars, to a foreign government official who has agreed to give the firm an exclusive contract to provide certain goods and services to the foreign state. The executive seems to think this is normal and good for the company, but the attorney believes it constitutes bribery of foreign officials, which would violate the Foreign Corrupt Practices Act, and could subject the company to enormous fines and penalties. The attorney explains her concerns to the executive, including her concern that he could face personal criminal charges in addition to bringing liability on the corporation, and she reminds him that she represents the corporation, not him personally. The executive is dismissive of her concerns, even though she approaches him several times about the matter. How must the attorney proceed? A. She should report the matter immediately, in writing, to the Department of Justice, and tell no one in the company that she has done so. B. She should keep her conversations with the executive confidential but try to document everything that she knows about the situation in case the Department of Justice brings an enforcement action. C. She should approach the executive's immediate corporate superior, advising those next up the chain of authority to stop the transaction and take appropriate actions against the executive involved. D. She should immediately notify the company's Board of Directors, advising them about the potential liability and threatening to report the activities to the Department of Justice if they take no action.

C. She should approach the executive's immediate corporate superior, advising those next up the chain of authority to stop the transaction and take appropriate actions against the executive involved. In-house lawyers represent the firm itself and have a duty of loyalty to the company overall, rather than to individuals within the company, according to Rule 1.13. In cases where one manager or executive is jeopardizing the company's future by committing federal crimes (like bribery of government officials), an attorney should try to correct the problem, and should start working up the chain of command if the individuals in charge are uncooperative.

What is currently the requirement under the Rules of Professional Conduct regarding lawyers having liability insurance for legal malpractice claims? A. The Model Rules require every practicing lawyer in the private sector to carry at least minimal liability insurance, but not government lawyers. B. The Model Rules require lawyers practicing in certain areas, like real estate and family law, to carry malpractice insurance, but not lawyers doing criminal defense work. C. The Model Rules do not require lawyers to have malpractice insurance, but many states require disclosure to clients if the lawyer is uninsured. D. The Model Rules forbid lawyers to carry malpractice insurance because of the moral hazard problem—insurance provides a perverse incentive to take more risks or to be less careful.

C. The Model Rules do not require lawyers to have malpractice insurance, but many states require disclosure to clients if the lawyer is uninsured. Malpractice insurance is common today for law firms, especially the larger firms. Insurance is a prudent way to manage risk, especially in an era when malpractice verdicts against firms have grown significantly in size and frequency. Nevertheless, most states do not require lawyers to have malpractice insurance, though several require uninsured lawyers to provide notice of their lack of insurance to prospective clients (or to the bar, in some jurisdictions). Of course, most lawyers have firms incorporated in some form (LLC, LLP, PC, etc.), which can provide some protection against personal liability. The MPRE has tested students' knowledge of this point on several recent exams.

An attorney worked for several years for a federal government agency in regulatory enforcement. The attorney was involved in several enforcement matters against Conglomerate Corporation. Big Firm has always represented Conglomerate Corporation in all its litigation and regulatory compliance matters. The attorney made a good impression on the Big Firm partners when serving as opposing counsel in the same litigation. At the end of a deposition of Conglomerate Corporation's executives during the discovery phase of an enforcement proceeding, Big Firm partners approached the attorney privately and asked if the attorney would be interested in leaving the agency for a position at Big Firm. The attorney explained that they would have to match his current salary at the government agency for him to consider the proposal. Big Firm then scheduled an employment interview with the attorney, at the end of which they offered to double his salary if he left the agency and accepted a position at Big Firm. The attorney decided to postpone deciding until the pending agency enforcement matters against Big Firm's client were complete, in order to avoid the appearance of a conflict of interest. The matters dragged on for another year, however, and Big Firm eventually withdrew its offer. Is the attorney subject to discipline? A. No, because the attorney decided to postpone deciding until the pending agency enforcement matters against Big Firm's client were complete, in order to avoid the appearance of a conflict of interest. B. No, because Big Firm eventually withdrew its offer and the attorney never actually went to work for Big Firm. C. Yes, because a lawyer currently serving as a public officer or employee shall not negotiate for private employment with any person who is involved as a party or as lawyer for a party in a matter in which the lawyer is participating personally and substantially. D. Yes, because Big Firm offered to double the attorney's salary instead of merely matching his current government salary, which creates a substantial conflict of interest for the attorney in any pending or future matters.

C. Yes, because a lawyer currently serving as a public officer or employee shall not negotiate for private employment with any person who is involved as a party or as lawyer for a party in a matter in which the lawyer is participating personally and substantially. C is correct because it states all the correct elements that will subject the attorney to discipline for his actions.

One for all and all for one. Three sisters sustained injuries in a car accident last year. Because they did not have enough money to get separate attorneys, they decided to consent to using the same attorney. Before trial, the opposing party made a settlement offer. One of the sisters had sustained severe injuries, but the other two had only superficial scrapes and bruises. Their attorney believed that the settlement offer was reasonable, and that it would be in the clients' best interest to accept it to avoid the costs of trial. Even so, the attorney expected the sister with the more serious injuries to be resistant to a settlement offer, because she might obtain a larger judgment if she presented her case in front of a jury. If the attorney obtains consent from the other two sisters to accept the defendant's settlement offer will he be subject to discipline if he accepts the offer without discussing it the third sister, who had the worst injuries? A. No, because when a client retains a lawyer, he or she authorizes the lawyer to accept or reject settlement offers on his or her behalf. B. Yes, because an attorney must inform each client about all the material terms of the settlement before accepting any settlement offer on behalf of multiple clients. C. Yes, because an attorney must inform and obtain written consent from each client about all the material terms of the settlement before accepting any settlement offer on behalf of multiple clients. D. No, because the three sisters consented to being represented by the attorney and he believed it was in their best interest as a whole.

C. Yes, because an attorney must inform and obtain written consent from each client about all the material terms of the settlement before accepting any settlement offer on behalf of multiple clients. C is correct because it states the applicable rule, Rule 1.8(g), that "a lawyer who represents two or more clients shall not participate in making an aggregate settlement of the claims of or against the clients . . . unless each client gives informed consent, in a writing signed by the client. The lawyer's disclosure shall include the existence and nature of all the claims or pleas involved and of the participation of each person in the settlement."

A criminal court found that an attorney had engaged in domestic violence against his partner, and convicted the attorney of misdemeanor-level battery, for which he served a six-month term of probation. Could the attorney be subject to professional discipline as well? A. Yes, because any illegal activity by a lawyer constitutes professional misconduct. B. No, because crimes of violence have no specific connection to fitness for the practice of law. C. Yes, because crimes of violence indicate a lack of the character traits required for law practice. D. No, because only felonies (not misdemeanors) can constitute professional misconduct.

C. Yes, because crimes of violence indicate a lack of the character traits required for law practice. Comment 2 for Rule 8.4 explicitly states that any crimes "involving violence, dishonesty, breach of trust, or serious interference with the administration of justice" would "indicate lack of those characteristics relevant to law practice." Domestic violence could be the basis for suspension or disbarment.

Written agreements not always required. An attorney has represented a client on various small matters in the past. The client now needs representation for a more substantial matter involving a business transaction. During a phone call, the attorney agrees to represent the client at a slightly higher hourly rate, given the complexity of the matter, and when they meet to discuss the transaction in more detail, the attorney double-checks with the client about the fee arrangement verbally, explaining it carefully and answering any questions the client may have. The attorney and the client never formalize the fee arrangement in writing, but the attorney does send printed bills to the client periodically. Eventually, the client starts to feel that the representation is costing too much, and objects to one of the bills. Was it permissible for the attorney to have an oral agreement over hourly fees, without putting the fee agreement into writing? A. Yes, because the matter is more complex than the previous work the attorney has done for the client. B. No, because fee arrangements must be in writing, to avoid disputes between lawyers and their clients later on. C. Yes, because even though it is always preferable to have fee agreements in writing, it is not required in this type of case. D. No, because the attorney should have reduced his hourly fee, rather than raising it, if the matter is more complex and will generate more hours of work for the lawyer.

C. Yes, because even though it is always preferable to have fee agreements in writing, it is not required in this type of case. The purpose of this question is to remind students that the absolute requirement for written fee agreements applies in only two scenarios— contingent fees and fees shared between lawyers from different firms that worked on the case. There is a strong preference for written fee agreements in all other situations (and it is certainly a prudent habit in legal practice), but it is not an absolute requirement.

An attorney represented a client in a prosecution for murder, and the prosecutor was seeking the death penalty. The trial was not going well, and the judge had not sequestered the jury, so the attorney sent his secretary to visit some of the jurors in their homes one evening, bringing them cookies and talking to them about the seriousness of sentencing a fellow human being to death. The secretary did not say she worked for the attorney, but instead introduced herself as a member of an advocacy group that seeks to abolish the death penalty, and she left pamphlets about abolishing the death penalty in each juror's home. Could the attorney be subject to discipline for this activity? A. No, because the attorney did not in fact speak to any of the jurors directly and therefore had no ex parte contact with them. B. No, because the attorney did not have the secretary discuss the merits of the case or the evidence, but only the morality of the death penalty, which is a serious public policy issue. C. Yes, because he was communicating ex parte with the jurors through the secretary during the proceeding. D. Yes, because the secretary did not inform the jurors that she worked for the attorney.

C. Yes, because he was communicating ex parte with the jurors through the secretary during the proceeding. Rule 3.5(b) prohibits communicating ex parte with jurors during a proceeding, and this includes indirect communication, as well as communications that the juror may not realize have come from the attorney. The point of this problem is to emphasize that the prohibition of ex parte communications does not depend on the juror's subjective impressions or knowledge, but on the actions of the lawyer.

The attorney hires a nationally known Internet-marketing specialist, a tech guru, to help develop the firm's reputation and attract new clients. The Internet specialist has made millions on previous tech startups, while the attorney is not well known and has been practicing for only two years. The tech guru demands certain terms in the contract that require the attorney to confer with the tech guru about accepting clients that were former clients of the tech guru, to avoid conflicts of interest. The attorney must also clear any litigation positions, approaches, or strategies that pertain to intellectual property or Internet marketing liability with the tech guru, to avoid positions that would jeopardize the guru's other business. Is the attorney subject to discipline for this arrangement? A. No, because the attorney is merely hiring an advertising specialist and can pay normal rates for such services. B. No, because the contract merely reflects the lawyer's duty under the Model Rules to avoid conflicts of interest between current clients. C. Yes, because in this case, a non-lawyer has a contractual right to direct or control the professional judgment of the lawyer. D. Yes, because the attorney is advertising online, which means Internet users in other states can see the firm's advertisements and offers of representation, even though the attorney does not have a license to practice in most of those jurisdictions.

C. Yes, because in this case, a non-lawyer has a contractual right to direct or control the professional judgment of the lawyer. This problem is another example of a non-lawyer having too much control over a lawyer's judgment. Note that this individual is not a client—the client should have control over the objectives of the representation, and may have some input about the methods for achieving those goals (especially if some methods will be much more costly or risky for the client). The non-lawyer in this question is a consultant that the lawyer hired to help attract more clients to his firm. Practitioners must be watchful for contracts with publicists and consultants that could interfere with the decisions about representation.

After it's over. An attorney agreed to represent a client as plaintiff in a patent infringement lawsuit. The attorney was part of a partnership that specialized in intellectual property law. The attorney prepared, and the client signed, a written fee agreement that specified the attorney would receive a tiered contingent fee in the case: 25 percent if the case settled before trial, 30 percent if they went to trial and won, and 35 percent if the case went up on appeal and they prevailed in the appellate stage. In addition, the agreement specified that the contingent fee would come from total award before court costs and other expenses, and that the client would be responsible for court costs and expenses out of his own pocket, either along the way as expenses arose during the proceedings, or from the client's share of the award after the attorney received his contingent fee. The attorney never revealed that his partnership agreement required him to share his part of the fees with three other partners in the firm, or that his fees would go toward a general firm operating budget from which the partnership paid the salaries of non-lawyer staff, such as paralegals and secretaries. The attorney obtained a favorable settlement before trial. He telephoned the client with the good news and explained that he would deduct his 25 percent contingent fee, as they had agreed, and would send the client the remainder of the settlement funds. At that time, there were no outstanding unpaid expenses or court costs. The client was glad to hear the news, and the attorney promptly sent the client a check for 75 percent of the total amount received from the other party. The attorney and the client had no other contact except to exchange holiday greeting cards. Were the attorney's actions improper? A. Yes, because the attorney failed to obtain written informed consent from the client to share fees with other lawyers in the firm, and because the attorney charged a tiered contingent fee in a patent litigation case. B. No, because contingent fees in patent litigation are proper as long as there is a written fee agreement at the beginning of the representation. C. Yes, because the attorney failed to provide the client with a written statement stating the outcome of the matter and showing the remittance to the client and the method of its determination. D. No, because the attorney properly followed the agreement with the client, and there were no outstanding court costs or unpaid expenses at the time of the settlement.

C. Yes, because the attorney failed to provide the client with a written statement stating the outcome of the matter and showing the remittance to the client and the method of its determination. Rule 1.5(c) requires attorneys to provide a written statement at the conclusion of the representation, stating the outcome and delineating any unused retainer funds that the lawyer is returning. This requirement avoids client confusion about the outcome of their case (for example, a client may not understand that a "remand" means they will have to go through another trial), and it avoids misunderstandings about whether the lawyer is continuing to represent the client going forward. The MPRE regularly tests this rule.

An attorney practices personal injury law, representing plaintiffs on a contingent fee basis. The attorney employs a paralegal to assist with preparing documents for litigation. The paralegal's salary arrangement is 10 percent of the firm's total net revenue each year. In years when the attorney wins several large cases, the paralegal receives higher wages, and in years when the attorney has no big wins, the paralegal receives almost nothing. The paralegal does not bring clients to the firm, but does participate in judgments about which clients to represent, how to structure contingent fee arrangements, and how much to seek in damages after a verdict, as these matters directly affect the paralegal's income as well as the attorney's. Could the attorney be subject to discipline for this arrangement? A. Yes, unless the paralegal has a law degree and is admitted in another state. B. No, because non-lawyers may participate in a firm compensation plan based on overall profit sharing. C. Yes, because this paralegal is sharing profits with the attorney, and is able to influence the professional judgment of the lawyer under this arrangement. D. No, because the rules treat paralegals the same as lawyers for purposes of sharing fees or profits.

C. Yes, because this paralegal is sharing profits with the attorney, and is able to influence the professional judgment of the lawyer under this arrangement. Rule 5.4(d)(3) forbids situations in which "a nonlawyer [not the client] has the right to direct or control the professional judgment of a lawyer." This question is nearly identical to the last one, except that in this question, the paralegal participates in decisions about representing clients. That makes all the difference—this situation clearly violates the Model Rules. This question also highlights the hazard of profit-sharing plans with law firm staff— eventually, those whose compensation depends on the firm's profits may want to voice opinions about decisions that will affect the firm's profitability. The MPRE has included questions in recent years similar to this, or highlighting the same point.

Screening at the firm. An attorney was a judge but has left that job and joined Big Firm. Another lawyer at Big Firm represents the client in a case on the docket at the same court where the attorney worked as a judge. In fact, as a judge, the attorney ruled on some of the pretrial motions in the case, mostly evidentiary motions. The firm has screening measures in place to screen the attorney from any participation in the matter. The attorney will receive no part of the fee from the matter, and timely notice went to the parties about the screening measures in place. The other parties, however, did not provide informed written consent to Big Firm's representation of the client. Is it proper for the other lawyer at Big Firm to continue representing the client in this matter? A. No, because a lawyer shall not represent anyone in connection with a matter in which the lawyer participated personally and substantially as a judge, and if a lawyer is disqualified, no lawyer in a firm with which that lawyer is associated may knowingly undertake or continue representation in the matter. B. No, because a lawyer shall not represent anyone in connection with a matter in which the lawyer participated personally and substantially as a judge, and the other parties did not provide informed consent, confirmed in writing, to the representation. C. Yes, so long as Big Firm also provides timely notice to the appropriate tribunal as well, so that the tribunal may ascertain compliance with screening measures. D. Yes, so long as the attorney is not receiving a salary or partnership share established by prior independent agreement.

C. Yes, so long as Big Firm also provides timely notice to the appropriate tribunal as well, so that the tribunal may ascertain compliance with screening measures. C is correct because it correctly states the rule and what further steps would be necessary for Big Firm to continue representation of the client.

What are some of the objectives of legal professionalism? A. Generating the mutual respect of attorneys and the respect of judges, clients, and society in general B. Improving the efficacy of legal advocacy and the impressions on the legal decision makers of legal advocates C. Standardizing a culture of professional conduct D. All of the above

D. All of the above

A client hires an attorney to help with the legal documents necessary to liquidate most of his investments so that he can use the cash to fund a new business venture. The client explains that he plans to quit his regular job and start a new career working from home as a "day trader," buying and selling stocks online every day in hopes of making large profits. The client has no experience or training in finance or investments, but he attended a seminar that featured testimonials from others who claimed to have made millions as day traders. The attorney thinks this is a foolish idea, but the client does not ask the attorney for his advice. Does the attorney have an ethical duty to caution the client against his seemingly reckless decision? A. Yes, because a lawyer has a duty to offer sound advice and not wait for a client to ask questions to solicit the specific information. B. Yes, if the attorney suspects that the client will eventually have trouble paying his legal fees. C. No, because many day traders are indeed successful, and this client could be one of the fortunate ones. D. No, because a lawyer is not expected to give advice until asked by the client.

D. No, because a lawyer is not expected to give advice until asked by the client. Comment 5 for Rule 2.1 opens with a general rule of thumb followed by a caveat: "In general, a lawyer is not expected to give advice until asked by the client. However, when a lawyer knows that a client proposes a course of action that is likely to result in substantial adverse legal consequences to the client, the lawyer's duty to the client under Rule 1.4 may require that the lawyer offer advice if the client's course of action is related to the representation." In this question, the client's proposal is to undertake a notoriously risky line of work at the expense of his regular (presumably more secure) job. It would be going too far, though, to say that this is "likely to result in substantial adverse consequences to the client," which would obligate the lawyer to interject a word of caution. A few people succeed at day trading, and others who fail are able to minimize their losses or return to a secure job easily—it depends on the person.

A client fired an attorney after the attorney had completed 80 percent of the work involved in the representation. The client refuses to pay any of the fees that were in the original agreement at the beginning of representation. The client also demands that the attorney turn over all papers and documents relating to the representation. Must the attorney immediately return the client's documents regardless of the fees owed? A. Yes, a lawyer must surrender all papers and property to the client as soon as representation ends, even if it ends with an untimely discharge of the lawyer. B. Yes, because the client has not received what she bargained for if she wants to discharge the lawyer before the representation is complete. C. No, because a client forfeits any right to papers and documents related to the representation if she discharges the lawyer without cause before the representation is complete. D. No, because a lawyer may retain papers relating to the client to the extent permitted by law.

D. No, because a lawyer may retain papers relating to the client to the extent permitted by law. D is correct—unless state law provides otherwise, a lawyer may hold on to client documents as security until the client pays the fees that the client actually owes to the lawyer for work already performed.

A client hired an attorney to represent him in a simple real estate matter. When the attorney asked some standard questions about the financial arrangements for the sale and purchase of the property, the client was somewhat evasive on a few points, but provided the information necessary to complete the legal work for the transaction. The attorney also heard from a friend that the client frequently cavorted with prostitutes. The attorney finds the client suspicious and has many unanswered questions, but none surrounding the transaction that occasioned the representation. Does the attorney have an ethical duty to inquire into the affairs of a suspicious client? A. Yes, because it is possible that the client is engaging in some kinds of illegal activity, and it is important to uncover whatever that might be. B. Yes, because the attorney has a right to know what kind of person he is representing in this simple real estate transaction. C. No, because a lawyer must never invade the privacy of a client in any way. D. No, because a lawyer ordinarily has no duty to initiate investigation of a client's affairs or to give advice that the client has indicated is unwanted.

D. No, because a lawyer ordinarily has no duty to initiate investigation of a client's affairs or to give advice that the client has indicated is unwanted. Comment 5 for Rule 2.1 says, "A lawyer ordinarily has no duty to initiate investigation of a client's affairs or to give advice that the client has indicated is unwanted, but a lawyer may initiate advice to a client when doing so appears to be in the client's interest." Lawyers often have clients whose personal decisions might seem imprudent or socially unacceptable, but the lawyer does not have an automatic duty to function as the client's life coach or to pry into a client's other matters that are unrelated to the representation.

During opening arguments in a criminal trial before a jury, an attorney, who was representing the defendant, closed his statements by declaring, "My client is innocent; I know it in my heart. By the end of the trial, I am confident that you will agree with me that this is an innocent man." Are such comments proper for a defense lawyer to make during trial? A. Yes, because we presume that every defendant is innocent until proven guilty. B. Yes, because the fact that the defendant has pleaded not guilty has already put that assertion before the jury. C. No, because such comments could manipulate and prejudice a jury, even though the comments would be acceptable in a bench trial. D. No, because at trial, a lawyer shall not state a personal opinion as to the guilt or innocence of an accused.

D. No, because at trial, a lawyer shall not state a personal opinion as to the guilt or innocence of an accused. Rule 3.4(e) states that during a trial, a lawyer must not "state a personal opinion as to the justness of a cause, the credibility of a witness, the culpability of a civil litigant or the guilt or innocence of an accused." This rule surprises many students, as television and movie courtroom scenes often include the lawyers asserting their own opinions and beliefs on behalf of their client. A lawyer may argue that the opposing party has not met their burden of proof, or that the evidence points conclusively in the lawyer's direction, but it is improper for the lawyer to assert mere opinions, personal convictions, or feelings about a matter. It is improper for a prosecutor to claim a personal opinion about the guilt of the accused, and it is improper for a defense attorney to assert a personal conviction about the defendant's innocence. The MPRE has recently included similar questions on this point.

That was then, this is now. An attorney has been practicing for five years, but on her application to the bar five years earlier, she had stated that she had attended a particular private high school, when in fact she had attended a public high school. An unhappy client recently filed a grievance against the attorney, which was frivolous, but the state disciplinary authority had to conduct a routine, preliminary inquiry into the matter in order to make a determination that the complaint merited dismissal. The disciplinary board member assigned to the case had attended the elite private high school from which the attorney claimed to have graduated, and made a mental note of the attorney's high school when he did a cursory review of her bar admission files. He thought it was strange that he had never seen or heard her name at any alumni or reunion functions, as they had supposedly graduated the same year and the classes were small. On a hunch, the board member checked the alumni lists for the school and discovered that the attorney had lied on her application to the bar five years earlier. When asked about this issue, the attorney said she could not be subject to discipline now for the misstatement she made several years ago, and that the board lacked jurisdiction because it was unrelated to the current grievance complaint. Is she correct? A. Yes, because she has been practicing now for five years and has demonstrated her character and fitness to practice law, making the application queries moot. B. Yes, because it was improper for the board member to conduct a self-initiated investigation into her high school attendance merely because he had graduated from the same high school that the attorney listed on her original bar application. C. No, because the fact that she lied about her high school makes it likely that the current client complaint has merit as well. D. No, because if a person makes a material false statement in connection with an application for admission, it may be the basis for subsequent disciplinary action if the person is admitted. Submit

D. No, because if a person makes a material false statement in connection with an application for admission, it may be the basis for subsequent disciplinary action if the person is admitted. Comment 1 for Rule 8.1 says, "Hence, if a person makes a material false statement in connection with an application for admission, it may be the basis for subsequent disciplinary action if the person is admitted, and in any event may be relevant in a subsequent admission application." There is no statute of limitations in the Model Rules, at least for false statements to the state bar. Note that previously overlooked problems with a bar application can easily become known during the routine file review involved in handling grievances against practitioners, even if the grievance itself ends in a dismissal due to lack of merit. This question has similar facts to an actual case in which the applicant had lied about which high school he had attended, claiming he attended a private high school rather than a public high school. It was the only false statement on the application, but it became the basis for disciplinary action later.

A patient of a well-known doctor suffers complications after her surgery, and believes she is the victim of medical malpractice. The patient writes to a lawyer, describing a medical-malpractice suit that the patient is contemplating, and she asks about retaining the lawyer to represent her in the lawsuit. The lawyer never responds. Eventually, many months later, the statute of limitations expires for her claim. The patient then files a legal malpractice lawsuit against the attorney due to the failure to file a claim on the original case on time. Could the lawyer be liable for malpractice to the patient? A. Yes, because lawyers have some duties even to potential clients, and it was foreseeable to the lawyer that if he did not respond, eventually the statute of limitations would expire on her claim. B. Yes, because a lawyer can be liable for malpractice even to a non-client if the lawyer violated the ethical rules in his interactions with the person. C. No, because the lawyer has no duties to a person if there is no client-lawyer relationship explicitly in place. D. No, because it was not reasonable for the patient to have relied upon the lawyer, as the lawyer never communicated to the patient.

D. No, because it was not reasonable for the patient to have relied upon the lawyer, as the lawyer never communicated to the patient. This problem is intentionally similar to one of the examples in Section 14 of the Restatement of the Law Governing Lawyers. A lawyer can be liable in malpractice to a non-client—either a prospective client or a third-party beneficiary of one of the lawyer's clients—if the person reasonably relied upon the lawyer and then suffered an injury as a result.

An attorney worked for several years for a federal government agency in regulatory enforcement. Big Firm then hired the attorney for a substantially higher salary, and the attorney accepted the position and left her government position. One of the attorney's first assigned cases at Big Firm was a new action by the client against the same government agency for which the attorney had previously worked, defending against an enforcement action that the attorney had initiated while at the agency. The defense will involve challenging the constitutionality of a new regulation that the agency had recently promulgated. While at the agency, the attorney had not been involved with the review and promulgation of any new regulations, including the one at issue in the client's challenge, but instead worked exclusively on enforcement litigation matters. The government agency gives informed consent, confirmed in writing, to the representation. Is the attorney nevertheless subject to disqualification in the client's matter against the attorney's former employer? A. Yes, because allowing Big Firm to give government lawyers an unreasonably large salary increase for leaving her public service position and joining Big Firm in the private sector creates a conflict of interest for all lawyers in government service. B. Yes, because the attorney participated personally and substantially in the matter as a public officer or employee. C. No, because the case involves a constitutional challenge to the validity of a regulation, and the attorney was not personally and substantially involved in the drafting or promulgation of the regulation. D. No, because the appropriate government agency gave its informed consent, confirmed in writing, to the representation.

D. No, because the appropriate government agency gave its informed consent, confirmed in writing, to the representation. D is correct because once the attorney obtained written informed consent from the government agency, she will fall under the exception to the rule that will allow her to represent the client despite the conflict of interest.

An attorney hires three new associates upon their graduation from law school in a neighboring state. The associates passed the bar in the neighboring state, but they are still unlicensed in the attorney's state. The associates confine their work to conducting research, reviewing documents, and attending meetings with witnesses in support of the attorney, who is responsible for all the litigation. The research done by the associates, however, is far beyond the capabilities of a paralegal or a typical law student associate. Is the attorney subject to discipline for this arrangement? A. Yes, because the attorney has facilitated the unauthorized practice of law by the associates. B. No, because the associates are licensed in a neighboring state, which presumably has similar laws and precedents. C. Yes, because the attorney is relying on research done by lawyers unlicensed in that jurisdiction. D. No, because the associates merely conduct delegated work under the attorney's supervision, for which the attorney is ultimately responsible.

D. No, because the associates merely conduct delegated work under the attorney's supervision, for which the attorney is ultimately responsible. Comment 2 for Rule 5.5 says, "This Rule does not prohibit a lawyer from employing the services of paraprofessionals and delegating functions to them, so long as the lawyer supervises the delegated work and retains responsibility for their work." The MPRE has included questions about law students and recent law graduates. Until a law school graduate has received formal bar admission (that is, not only passing the bar, but also the swearing-in admission process), they cannot provide legal services without engaging in the unauthorized practice of law.

After much effort, an attorney located a witness who could fully corroborate his client's story and could impeach the testimony of the opposing party's star witness. The witness, however, was afraid of retaliation from others if she testified, and did not want to be involved. The attorney offered the witness $10,000 to appear at the trial for one afternoon and testify for an hour or two. The witness reluctantly agreed. Was it proper for the attorney to offer to pay a favorable witness to undergo the trouble of testifying at the trial? A. Yes, because expert witnesses routinely charge large sums to testify at trial, so it is proper for a non-expert to receive a modest amount of compensation, especially if she is fearful of adverse consequences from testifying. B. Yes, because the goal of the trial is to determine the facts of what happened, and it is important to have every material witness testify in order to corroborate the truth and impeach the false statements of others. C. No, because the lawyer offered the witness an unreasonably large amount of money. D. No, because the common law rule in most jurisdictions is that it is improper to pay an occurrence witness any fee for testifying apart from expenses.

D. No, because the common law rule in most jurisdictions is that it is improper to pay an occurrence witness any fee for testifying apart from expenses Lawyers can pay experts to testify (as long as it is not a contingent fee), but paying non-expert witnesses, such as an eyewitness to the events that triggered the case, is improper, except for reimbursing a witness for the costs of testifying. The traditional rule is that lawyers can compensate a witness's travel expenses (including airfare), lodging, food, lost wages, if any, for the time away from work, and possibly for the time it takes to prepare for testimony, such as reviewing old e-mails or corporate records. Reimbursing or compensating a witness can apply to testifying at trial or at a deposition. It is never proper to pay a witness a contingent fee—that is, to offer to pay the witness only if the case goes well, or to pay more if the case goes well. Nor is it proper to pay the witness a large sum that looks like an inducement to testify rather than reimbursement for costs, as the lawyer did in this problem. As Comment 3 for Rule 3.4 says, ". . . [I]t is not improper to pay a witness's expenses or to compensate an expert witness on terms permitted by law. The common law rule in most jurisdictions is that it is improper to pay an occurrence witness any fee for testifying. . . ."

The owner of a local grocery store uses ABC Law Firm to defend minor slip and fall cases that occur in its store. An attorney believes she can provide equivalent quality legal services at a better price. She and the owner each have 7 year-old children who play soccer on a team called the Bumblebees. The attorney walks up to the owner of the store at a soccer game and offers to represent the owner in litigation. The owner was offended that the attorney would try to solicit business at a kids' sporting event, and told her to leave him alone. She did so, and never approached the owner again. The owner filed a grievance with the state disciplinary authority. Is the attorney subject to discipline? A. Yes, because the attorney solicited professional employment by live person-to-person contact with the owner, and her motivation was pecuniary gain. B. Yes, because the owner is not a lawyer or a family member of the attorney. C. No, because the owner told the attorney not to solicit professional employment at the soccer game. D. No, because the owner routinely uses another law firm to defend slip-and-fall cases.

D. No, because the owner routinely uses another law firm to defend slip-and-fall cases. The grocery store owner routinely uses the type of legal services offered by the attorney. See Rule 7.3(b)(3). That's the significance of the fact that he usually employs the ABC Law Firm. (If you're looking at an older version of Rule 7.3(b), the language would be "an experienced user of the type of legal services involved.") I changed the language a little, hoping not to telegraph the answer, but it's saying the same thing as Rule 7.3(b)(3).

A government agency contacts an attorney, who works as in-house counsel for Corporation, and requests a report about some of Corporation's activities that come under the agency's regulatory jurisdiction. As the attorney begins to investigate the matter to prepare the report, he learns that the information requested by the agency will subject Corporation to significant regulatory enforcement sanctions, and if the information became public, would adversely affect Corporation's share price. At this point, the agency has not issued a subpoena and compliance with the request is voluntary, although the agency could compel the disclosure eventually. The managers and directors of Corporation instruct the attorney not to submit the report until the agency issues a subpoena, to buy some time to mitigate their regulatory violations. May the attorney prepare the report and submit it to the agency at this time? A. Yes, because a lawyer should have whatever latitude of investigation seems necessary as a matter of professional judgment. B. Yes, because the agency will inevitably subpoena the information anyway, and delaying merely provides the managers with an opportunity to conceal their wrongdoing. C. No, because even if the managers and directors consented to the disclosures, the attorney should not disclose information that will adversely affect the shareholders. D. No, because when a lawyer knows or reasonably should know that the evaluation is likely to affect the client's interests materially and adversely, the lawyer shall not provide the evaluation unless the client gives informed consent.

D. No, because when a lawyer knows or reasonably should know that the evaluation is likely to affect the client's interests materially and adversely, the lawyer shall not provide the evaluation unless the client gives informed consent. Rule 2.3(b) instructs a lawyer to withhold an evaluation (or postpone performing it) if the lawyer realizes that it necessarily includes disclosures that will have material adverse consequence on the client. In this case, the lawyer has done the investigation at the behest of the client (the management) and has discovered information that will subject the client to a regulatory enforcement action. The client asked the lawyer to withhold the report, and the lawyer should do so. The situation might be different if an investigation were already underway and a subpoena had been issued, but at this point, that has not occurred—the lawyer can wait until the agency begins demanding information, and then revisit the issue. As the subsection says, "When the lawyer knows or reasonably should know that the evaluation is likely to affect the client's interests materially and adversely, the lawyer shall not provide the evaluation unless the client gives informed consent."

A client fired an attorney after two weeks of representation, long before the matter was complete. The client had prepaid a large refundable retainer, against which the attorney was to draw his fees as the representation went on. The client therefore has fully paid her fees up to that point to the attorney. The attorney is upset about the client discharging him without cause and believes it is unfair and wrongful. The attorney refuses to return the remainder of the fees and refuses to turn over any documents from the representation to the client. Is it proper for the attorney to take this course of action, if indeed the client had no good reason to discharge him? A. Yes, because a client must obtain court permission to discharge a lawyer before the representation is complete. B. Yes, it is proper for an attorney to retain the remaining funds and the documents. C. No, it is improper for an attorney to retain the unused funds, but an attorney may withhold the documents. D. No, it is improper for an attorney to retain either the unused funds or the documents.

D. No, it is improper for an attorney to retain either the unused funds or the documents. D is correct—it is normally improper for a lawyer to retain either unused client funds or client documents, no matter how unfair it was for the client to discharge the lawyer.

The lateral move. Big Law Firm represented Conglomerate Corporation, but the primary lawyer who handled Conglomerate's matters left Big Law Firm, and Conglomerate Corporation followed the lawyer to his new firm for further representation on other matters. Some time later, Giant Company consulted with Big Law Firm about legal representation that would be materially adverse to Conglomerate Corporation. A partner at Big Law Firm accepts Giant's Company's new case. Would it be improper for the partner or other lawyers still working for Big Law Firm to provide representation to Giant Company in a lawsuit against Conglomerate Corporation, if the new matter has no substantial relationship to Conglomerate's previous legal matters? A. Yes, attorneys are imputed with knowledge of current or previous members of the firm, and attorneys with imputed knowledge shall not accept cases of potential clients whose interests would be materially adverse to those of a prior client of the firm. B. Yes, an attorney shall obtain the informed consent, confirmed in writing, of a client of a prior attorney's clients if the attorney is going to represent a different client with materially adverse interests. C. No, when a lawyer leaves a law firm, the rules regarding conflicts of interest and imputation do not apply. D. No, prior lawyers' knowledge is not imputed unless the matter is the same or substantially related and another lawyer in the firm has information that is material to the matter.

D. No, prior lawyers' knowledge is not imputed unless the matter is the same or substantially related and another lawyer in the firm has information that is material to the matter. D is correct because it correctly states the rule and accurately presents Model Rule 1.10(b).

An attorney worked for several years for a federal government agency in regulatory enforcement. Big Firm then hired the attorney for a substantially higher salary, and the attorney accepted the position and left his government position. One of the attorney's first assigned cases at Big Firm was a new action by the client against the same government agency for which the attorney had previously worked, defending against an enforcement action that the attorney had initiated while at the agency. The defense will involve challenging the constitutionality of a new regulation that the agency had recently promulgated. While at the agency, the attorney had not been involved with the review and promulgation of any new regulations, including the one at issue in the client's challenge, but instead worked exclusively on enforcement litigation matters. The government agency refuses to consent to the attorney representing the client, who is the adverse party to the agency, in this matter, and seeks to disqualify Big Firm from representing the client. Is Big Firm subject to disqualification in the client's matter against the attorney's former employer? A. Yes, because allowing Big Firm to give government lawyers an unreasonably large salary increase for leaving his public service position and joining Big Firm in the private sector creates a conflict of interest for all lawyers in government service. B. Yes, because the attorney participated personally and substantially in the matter as a public officer or employee, and cannot recuse himself from representing the client, and the appropriate government agency gives its informed consent, confirmed in writing, to the representation. C. No, so long as Big Firm has policies and procedures in effect to supervise the attorney's work closely enough to ensure compliance with the Rules of Professional Conduct. D. No, so long as Big Firm screens the attorney in time from any participation in the matter and provides the agency with prompt written notice about the screening measures in effect.

D. No, so long as Big Firm screens the attorney in time from any participation in the matter and provides the agency with prompt written notice about the screening measures in effect. D is correct because Big Firm will not be disqualified from representing the client as long as the lawyer who would be disqualified is screened from participation in the matter and notice of those screening measures are provided to the government agency.

An attorney is representing the defendant in a highly publicized trial. On his way into the courthouse on the day of jury selection, reporters gather around the attorney hoping for comments. The attorney explains that the prosecutor already held a press conference in which she shared that the defendant had refused to take a polygraph test, and that DNA tests had confirmed the defendant's guilt. The attorney explains that polygraph tests are inadmissible due to their unreliability, and that the DNA results are in dispute and will be the subject of expert testimony at trial. He adds that the sleazy prosecutor has a habit of holding such press conferences to prejudice the proceedings before every criminal trial, and that it merely reveals that the prosecutor's cases are too weak to win on the merits without such stunts. His client, he says, is now guilty until proven innocent, which is a shame considering the serious criminal charges in the case. He also mentions that the state's star witness is a dangerous convicted felon who is testifying in exchange for early release from prison. Were the defense attorney's statements proper? A. Yes, if others have already made prejudicial statements publicly, responsive statements may have the salutary effect of lessening any resulting adverse impact on the adjudicative proceeding. B. Yes, because the First Amendment and Sixth Amendment protect a defendant's right to defend himself publicly through his attorney against false accusations. C. No, there is a presumption of prejudicial effect on the proceedings when a lawyer comments publicly about the performance or results of any examination or test or the refusal or failure of a person to submit to an examination or test. D. No, such responsive statements should contain only such information as is necessary to mitigate undue prejudice created by the statements made by others.

D. No, such responsive statements should contain only such information as is necessary to mitigate undue prejudice created by the statements made by others. This problem deliberately extends the facts of the previous problem to highlight the limits on the self-defense exception to Rule 3.6. Rule 3.6(c) concludes its provision about the exception with the following caveat: "A statement made pursuant to this paragraph shall be limited to such information as is necessary to mitigate the recent adverse publicity." Comment 7 for Rule 3.6 expresses it this way: "Such responsive statements should be limited to contain only such information as is necessary to mitigate undue prejudice created by the statements made by others." Students should watch for test questions in which the responsive lawyer (the one availing herself of the self-defense exception) goes beyond refuting the existing negative publicity and goes on the offensive, escalating the level of attacks between the lawyers or parties.

Which is not an acceptable way to obtain pro bono clients? A. Through court appointment B. Through participation in legal services organizations or legal reform activities C. Through involvement in limited legal services D. None of the above

D. None of the above

Client and Lawyer have a 33 percent contingency fee agreement. After a heated argument about politics, Client fires Lawyer and retains New Lawyer. Lawyer had worked 100 hours on the case. Represented by NewLawyer, Client goes to trial and obtains a $100,000 judgment. New Lawyer worked 200 hours on the case. How much may Lawyer recover from Client? A. Contract damages of $33,000, because Client terminated Lawyer without good cause. B. Contract damages of $33,000, because Lawyer would be entitled to that amount regardless of whether Client had good cause for termination. C. Restitution damages of 100 hrs. x Lawyer's reasonable hourly rate, because Client terminated representation for a frivolous reason. D. Restitution damages of 100 hrs. x Lawyer's reasonable hourly rate, provided that grounds for fee forfeiture do not exist.

D. Restitution damages of 100 hrs. x Lawyer's reasonable hourly rate, provided that grounds for fee forfeiture do not exist. The correct answer is D. Restatement §40 summarizes the law concerning fees upon termination of the attorney-client relationship. The most important thing to note is that the lawyer's entitlement to compensation does not coincide with the lawyer's entitlement to continue representing the client. In other words, the client may have a right to terminate the lawyer-client relationship (see Rule 1.16(a)(3)), but that does not mean the client is relieved from having to pay the lawyer for her services. The law of remedies, specifically the entitlement to recover in quantum meruit to avoid unjust enrichment, provides that the lawyer may receive the fair value of her services. Quantum meruit recovery balances the client's interests in being able to terminate the professional relationship with the lawyer's interests in getting paid. That is why, in general, courts do not allow the lawyer to recover her fee on the contract. In some instances, however, courts will allow the lawyer to recover the contractually specified fee. The most common situation is where the client in a contingent fee representation discharges the lawyer right before the contingency occurs, to avoid paying the fee. See Rest. §40, cmt. c. In this example the discharge occurred far enough in time from the contingency that the lawyer's recovery should be given by quantum meruit principles.

The appellate judge. An attorney spent several years working on the state intermediate appellate court as one of its nine justices in a state in which such judges run for election in the general elections every four years. When the attorney ran for re-election, she lost, and needed to return to private practice. The client wants the attorney to represent her in her appeal of a state trial verdict. The case previously came up on appeal before the state intermediate appellate court, but the attorney was not on the panel that decided the case. The state Supreme Court subsequently reversed the decisions of both the appellate court and the trial court and remanded the case for a new trial. The new trial resulted in an unfavorable verdict for the client, so she wants to appeal the case again. Would it be proper for the attorney to represent her in this matter? A. No, because the appeal will come before the very court for which the attorney worked as a judge, and the panel could include some of the attorney's former colleagues. B. No, because the state Supreme Court already reversed the decision of the state intermediate appellate court, so it is improper for the client to appeal the remanded case back to the same state intermediate appellate court again, as this could thwart the intentions of the Supreme Court. C. Yes, because it was not the judge's fault that the state Supreme Court reversed the previous appellate decision, making a new trial and subsequent appeals necessary, and that the opposing party has not settled the case in the meantime. D. Yes, because a judge who was a member of a multimember court, and thereafter left judicial office to practice law, is not prohibited from representing a client in a matter pending in the court, but in which the former judge did not participate.

D. Yes, because a judge who was a member of a multimember court, and thereafter left judicial office to practice law, is not prohibited from representing a client in a matter pending in the court, but in which the former judge did not participate. D is correct because the rule requires that the lawyer participated personally in the previous matter to be disqualified from representing a client. The attorney was part of the multimember court but was not part of the panel that decided the case.

Disclosing adverse legal authority. While conducting research on a litigation matter, an attorney finds a very new case from the highest court in his jurisdiction that is directly adverse to his client's legal position in the case. The opposing party did not mention the case in its briefs, and the attorney realizes that the opposing party's lawyer has been recycling his firm's briefs for this type of case for several years without updating his research. Does the attorney have an ethical duty to disclose the unfavorable binding precedent to the court? A. No, because it is the other lawyer's duty to find the cases favorable to his own side, so providing the research to the opposing side would be facilitating the other lawyer's neglect of diligent representation. B. No, because it would be a breach of the attorney's duty of loyalty to his own client to disclose a case unnecessarily that undermines their position. C. Yes, because it is very common for litigators to recycle their briefs for years at a time, and everyone should help each other out with updating their legal research on issues that arise frequently in that area of litigation. D. Yes, because a lawyer must disclose to the tribunal legal authority in the controlling jurisdiction known to the lawyer to be directly adverse to the position of the client and not disclosed by opposing counsel.

D. Yes, because a lawyer must disclose to the tribunal legal authority in the controlling jurisdiction known to the lawyer to be directly adverse to the position of the client and not disclosed by opposing counsel. Rule 3.3(a)(2) says that a lawyer must "disclose to the tribunal legal authority in the controlling jurisdiction known to the lawyer to be directly adverse to the position of the client and not disclosed by opposing counsel." This rule may be more counterintuitive to law students than the other portions of Rule 3.3 (which prohibit outright lying about facts). For purposes of multiple-choice test questions, remember that this rule applies only to controlling precedent, not to persuasive authority from lower courts or sister jurisdictions. For purposes of legal practice, it may be possible for lawyers to distinguish a case due to factual differences, even when the case would otherwise be controlling precedent from that jurisdiction. The situation here is sadly commonplace, as some legal practitioners fail to update their research when reusing arguments from previous briefs.

In anticipation of a hearing before a federal agency in Washington, D.C., an attorney travels to a Washington suburb in Virginia from her own state to meet with her client (from her home state), interview witnesses, and review relevant documents. The attorney makes weekly trips there over the course of a year and spends most of her workweek there each time (four or five days), as the agency hearing pertains to a complex antitrust matter. The attorney solicits no new clients there. She works only on the matter for the client from her home state but is nonetheless unlicensed in Virginia. Is the attorney's conduct proper? A. Yes, the rules pertaining to unauthorized practice of law do not apply to any federal agency hearings. B. No, because her activity there continued for a full year, and therefore is not "temporary," so she is engaged in the unauthorized practice of law. C. No, because she is spending more time there than in her home state where she holds a license, despite this being a temporary arrangement. D. Yes, because a lawyer rendering services in a foreign jurisdiction on a temporary basis does not violate the rules merely by engaging in conduct in anticipation of a proceeding or hearing in a jurisdiction in which the lawyer is authorized to practice law.

D. Yes, because a lawyer rendering services in a foreign jurisdiction on a temporary basis does not violate the rules merely by engaging in conduct in anticipation of a proceeding or hearing in a jurisdiction in which the lawyer is authorized to practice law. Rule 5.5(c)(2) allows lawyers to practice in another jurisdiction temporarily if they "are in or reasonably related to a pending or potential proceeding before a tribunal in this or another jurisdiction, if the lawyer, or a person the lawyer is assisting, is authorized by law or order to appear in such proceeding or reasonably expects to be so authorized." Similarly, 5.5(c)(4) permits it for instances that "arise out of or are reasonably related to the lawyer's practice in a jurisdiction in which the lawyer is admitted to practice." Both of these provisions could be applicable here. Further, Comment 10 to Rule 5.5 says, "A lawyer rendering services in this jurisdiction on a temporary basis does not violate this Rule when the lawyer engages in conduct in anticipation of a proceeding or hearing in a jurisdiction in which the lawyer is authorized to practice law or in which the lawyer reasonably expects to be admitted pro hac vice."

A share of the divorce proceeds. An attorney agrees to represent a client in a divorce proceeding against her husband. The client is particularly concerned about obtaining her fair share of the marital property or assets—as much as possible, in fact—as well as a suitable level of child support for their children. The client agrees to pay the attorney his usual flat fee for divorce cases, $5,000, but also offers to pay him 10 percent of whatever he wins in terms of payments and distribution of assets, on top of his usual fee. After a protracted, acrimonious divorce proceeding, the attorney obtains a settlement worth approximately $2 million for the client. Is the attorney subject to discipline in this scenario? A. No, because the client proposed the arrangement and agreed to it beforehand. B. No, because the contingent fee was much lower than the typical contingent fee in personal injury cases, and the trial was long and acrimonious. C. Yes, because the attorney made a mixed flat-fee/contingent-fee arrangement, which is improper under the Rules of Professional Conduct. D. Yes, because the attorney entered into an arrangement for a fee in a domestic relations matter, the amount of which was contingent upon the amount of alimony, support, or property settlement.

D. Yes, because the attorney entered into an arrangement for a fee in a domestic relations matter, the amount of which was contingent upon the amount of alimony, support, or property settlement. Rule 1.5(d)(1) strictly forbids contingent fees in domestic relations cases. The policy rationale for this prohibition is that contingent fees would give lawyers a perverse incentive to make the divorce more acrimonious or unnecessarily traumatic for the parties in order to obtain a larger share for their client, and therefore, a larger sum for the lawyer. This is a bright-line rule that lends itself easily to multiple-choice questions, so students should prepare to encounter it on the MPRE.

A client is struggling through a deposition, during which opposing counsel is subjecting him to intense questioning. The attorney, who represents the client, tries objecting a few times in order to break the opposing counsel's momentum, but it was to no avail. The attorney then stood up, shouted, and with a heave overturned the conference table around which the lawyers, court reporter, and deponent were sitting. Notes, cell phones, and open briefcases flew across the room, and the stenographer's equipment tumbled to the floor. The attorney and the client gathered their things and stormed out of the room. A few days later, the attorney called opposing counsel and halfheartedly apologized, and agreed to reschedule the deposition if opposing counsel would agree to behave himself this time. Opposing counsel reported the attorney to the state bar disciplinary authority. Could the attorney be subject to discipline for the way in which he disrupted the deposition? A. No, because the disruption was merely at a deposition, which is not as formal as a trial or hearing. B. No, because his response was appropriate given the aggressiveness of opposing counsel in the deposition. C. Yes, because the attorney did not properly apologize for his own conduct or take responsibility for his actions. D. Yes, because the duty to refrain from disruptive conduct applies to any proceeding of a tribunal, including a deposition.

D. Yes, because the duty to refrain from disruptive conduct applies to any proceeding of a tribunal, including a deposition. Comment 5 for Rule 3.5(d) says, "The duty to refrain from disruptive conduct applies to any proceeding of a tribunal, including a deposition." Movies and television dramas often feature lawyers "making a scene" in a trial or in a deposition, but the Model Rules prohibit this, and the lawyer could be subject to discipline for being disruptive. Note that even provocation—such as annoying behavior by opposing counsel—does not justify a disruptive outburst from an attorney.

"I have needs." A wealthy heiress hired an attorney to represent her and her family in a complex federal case involving the family business and charges of securities fraud and racketeering. Early in the representation, the attorney and the client start dating and become sexually involved, to the consternation of the rest of the family. The client and her attorney rationalize the relationship by saying that they each have "needs" that their new romantic partner meets, and they have even discussed marriage as an eventuality. Is the attorney subject to discipline for this relationship? A. No, the Constitution does not allow a state bar or the judiciary to interfere in private matters such as a lawyer's consensual sexual relationships. B. Yes, the lawyer is representing the other family members as well, and they disapprove of the relationship at this time; if they approved of the relationship, it would be fine. C. No, because it appears this will be a long-term or permanent relationship, perhaps leading to marriage, so there is very little risk of the lawyer exploiting his client or the lawyer having clouded judgment. D. Yes, unless the sexual relationship predates the beginning of legal representation, the lawyer absolutely cannot represent a client with whom he has such a relationship. Submit

D. Yes, unless the sexual relationship predates the beginning of legal representation, the lawyer absolutely cannot represent a client with whom he has such a relationship. Submit D is correct, because according to Rule 1.8(j), the lawyer could be subject to discipline for starting a sexual relationship with a client, and it would undermine his defense if other family members sued him for malpractice later on. Note that the rules contain a single exception—sexual relationships that start before the representation begins.

Jane has represented Harry and his family in all their legal matters for a number of years. Jane has recently become aware that Harry is using her services to commit fraud. She wants to withdraw as his counsel, but she signed a retainer agreement in which she agreed to represent him in all his legal matters. The agreement covers the next six months. Jane also knows her withdrawal will result in a material adverse effect on Harry and his family. May Jane withdraw?

Even though Jane's withdrawal will cause a material adverse effect on the client, Jane is required to withdraw if she knows that Harry is using her services to commit fraud. If, after knowing that her services are being used to commit fraud, she does not withdraw, she will have civil liability to those whom Harry defrauds with her services.

Norman Bates is arrested on murder charges. He hires you to represent him. You have serious doubts about his mental competence. He discharges you in a fit of rage. What should you do?

First, you should make a "special effort" to help Norman consider the consequences of firing you. MR 1.16, Comment [6]. Then, if that doesn't change Norman's mind, you should think about taking reasonably necessary protective action as provided in MR 1.14(b). MR 1.16, Comment [6].

During the representation of Client, Lawyer discovers evidence that leads Lawyer to reasonably believe that Client is using Lawyer's services to lay the foundation for a future tax fraud. Must Lawyer withdraw from the representation? A. Yes, because the continued representation will result in a violation of other law. B. Yes, because Client's course of action involves Lawyer's services in fraudulent conduct. C. No, because Lawyer does not know that the representation will result in a violation of other law. D. No, because tax fraud is not a violation of the rules of professional conduct.

I believe the answer is (C). I'm hedging a bit because the rule is frustratingly silent on the mens rea to be applied. Look at Rule 1.16(b)(2). It permits a lawyer to withdraw where the client persists in a course of action that the lawyer reasonably believes is criminal or fraudulent. That's helpful; it clarifies that the lawyer need not have actual, subjective knowledge that the client's conduct is criminal or fraudulent—reasonable believe is enough for permissive withdrawal. But this question is asking about mandatory withdrawal. Look at Rule 1.16(a)(1). It says a lawyer must withdraw when the representation will result in violation of the rules or other law (including crimes and frauds), but it does not say whether the lawyer must know that this will be the result, must reasonably believe it to be the case, may merely suspect it to be the case, or whether the lawyer's mental state is irrelevant (that is, Rule 1.16(a) is a strict liability rule). It's unlikely that the rule really imposes strict liability, because the rule requires the lawyer to do something. A lawyer has to have some mental state with respect to a factual predicate before she can be required to do something. For this reason, (A) and (B) are probably not the answer. Look at those options and make sure you understand how they differ from (C) in terms of the lawyer's mental state.

Defendant Jasmine was a corporate CEO charged with major financial defalcations and frauds. She was desperate to delay the significant lawsuit pending against her, hoping that she could land a new position at another corporation before the frauds could be proven. In her answers to the plaintiff's interrogatories, she falsely swore that she has a severe, untreatable cancer and is expected to die within three months. Jasmine's lawyer filed the answers to interrogatories, believing them to be true. Two weeks later, Jasmine saw her lawyer at a trendy urban nightspot, and confided in him that the statements regarding cancer were entirely false, meant only to buy her time. What are permissible courses of conduct for Jasmine's lawyer to take? System Font16px

Jasmine's lawyer cannot continue the litigation and protect Jasmine's confidential information, as this course of action would involve him in his client's ongoing fraudulent conduct and expose him to disciplinary, civil, and perhaps even criminal liability for the frauds. Jasmine's lawyer can reveal the false answers to interrogatories to the plaintiff. This is a permissible option under Model Rule 1.6. Jasmine's lawyer's services have been used to perpetrate a fraud and advance a serious financial malfeasance. The confidentiality rule permits Jasmine's lawyer to reveal his client's confidences in such circumstances. Alternatively, Jasmine's lawyer can seek the court's permission to withdraw from the matter and give notice to the plaintiff that he disavows responsibility for the answers to interrogatories. This is the noisy withdrawal option, which is permitted under Model Rule 1.16.

Mark Antony drives his chariot through a traffic light and strikes the chariot of Julius Caesar, who is traveling at excessive speed. In the collision, Cleopatra, an innocent bystander, is severely injured. Cleopatra files a joint and several claim against Antony and Caesar. You represent Antony, and you conclude Antony may well get nailed to the wall for some big-time damages. You approach Caesar's lawyer and offer to contribute to a joint settlement, but Caesar refuses to settle. The jurisdiction doesn't recognize contribution among joint tortfeasors. If Antony concedes his liability, he'll be liable for all Cleopatra's damages. If Antony denies liability, the jury may decide that only Caesar should be liable. If you file an answer denying Antony's liability, are you subject to discipline for offering a frivolous defense?

No. Although the defense may not be supported by your analysis of the facts, you have no alternative but to assert it in the interest of your client. You cannot anticipate the jury's decision on these facts. The jury may find only Caesar liable. Denying liability is a good faith tactical decision. Meagher v. Kavli, 97 N.W.2d 370 (Minn. 1959).

You are prosecuting a murder case against Richard Kimble, accused of murdering his wife. You don't fully investigate witnesses who might corroborate Kimble's story that the person who actually murdered his wife was a one-armed man. Kimble is convicted and appeals, and his conviction is overturned on appeal for reasons related to your neglect in investigating the witnesses. Kimble files a malicious prosecution charge against you. Can he prevail?

No. Failing to investigate and disclose evidence favorable to the defense subjects a prosecutor to discipline under MR 3.8(d) by the lawyer discipline process in the state in which the prosecutor practices. The federal courts have refused to intercede when a state has sought to discipline a federal prosecutor. See In re Doe, 801 F. Supp. 478 (D.N.M. 1992). But a prosecutor may not be sued in tort for his actions as prosecutor. He enjoys absolute immunity against tort claims for acts in the performance of his duties. Kimble will not prevail.

You represent Alice Wunderkind in a personal injury suit against the O'Well Drilling Co. O'Well has an excellent contributory negligence defense. Your jurisdiction still recognizes contributory negligence, even though judges in neighboring jurisdictions have all begun to adopt comparative negligence in recent years. You conduct careful research and urge the court to find for Alice on a comparative negligence theory. Are you subject to discipline for advancing a frivolous argument if no court in your state has endorsed it?

No. If you believe in good faith that your argument is sound and defensible, you may assert it even if it flies in the face of existing law. You are permitted to argue for extending, modifying, or reversing an existing law. MR 3.1. Otherwise, the law would be frozen forever. Here, the fact that the trend is toward comparative negligence suggests that your good faith argument may prevail in a given case. The lawyer's knowledge, standing alone, that his position may not ultimately prevail, or even his personal belief that it shouldn't prevail, is not a basis for discipline.

You represent a debtor against a creditor who has violated the state's Fair Debt Collection Act. You assert several defenses that are sustained by the trial judge, and you win. The creditor appeals and the First District Appellate Court reverses, rejecting all your arguments on the law. Another debtor in the state's Second District retains you in another suit by the same creditor, and you assert the same defenses as in the first action even though you know that the First District has rejected your arguments. Are you subject to discipline?

No. If you believe in good faith that your arguments are valid as a matter of law, you are entitled to pursue them until the law is firmly established by the court of highest jurisdiction. You are always entitled to argue for the "extension, modification or reversal of existing law," so long as you make your arguments in the good faith belief that they have merit. MR 3.1. Here, the judge in the second case may decide in your favor and the Appellate Court in the Second District may affirm his decision, creating a conflict between the two appellate courts. In resolving this conflict, the highest court in the state may decide in your favor.

You represent Foul Mouth Frank in a landlord-tenant dispute. (His landlord evicted him from his apartment after neighbors complained of a near-constant emission of foul language, ever-audible through the walls.) You have filed the initial pleadings, discovery has taken place, and there are no court dates or other deadlines in the next couple of months. After working closely with Frank for months, you decide you just can't take the persistent use of foul language any longer. You do not want to hurt Frank's feelings, but you tell him he will need to find substitute counsel because you can no longer handle his case. Will you be subject to discipline for withdrawing from his case under the Model Rules?

No. MR 1.16(b) allows you to withdraw from a case for any reason at all—including a low tolerance for foul language—provided you can do so "without material adverse effect" to your client's interests. Here, you have done the preliminary work necessary to initiate the action, there are no pending court hearings or other deadlines in the near future, and you have given Frank plenty of notice that he needs to find a new attorney. Therefore, there would be no material adverse effects on Frank if you withdraw, and you may do so.

You are a prosecutor with the State Attorney's office. In an extremely well-publicized case, a famous actor's girlfriend is murdered. The actor often plays unsavory characters, and this may help explain the public outcry that the actor be arrested. There is a racial element as well, in that the actor is white and his girlfriend was black. People are insisting that justice demands that the actor be arrested. However, you have little or no evidence to tie him to the crime beyond the public opinion polls. To prevent bad publicity and possible racial disturbances, may you bring charges and then drop them if nothing substantial is uncovered in the investigation?

No. Prosecutors are prohibited from prosecuting charges that the prosecutor knows are not supported by probable cause. Although a public interest is identified here (the avoidance of racial tensions, etc.), bringing charges would subject the lawyer to discipline.

You are appointed by the court to represent Butch Cassidy on an armed robbery charge. During your first meeting with Butch, he admits he committed the crime but insists on pleading "not guilty" to the charge. You have reason to believe Butch is guilty but file a "not guilty" plea anyway. Are you subject to discipline under the Model Rules?

No. Under MR 3.1, a lawyer for a criminal defendant (or the respondent in a case that may result in incarceration) may conduct the defense in a manner that forces the prosecution to establish every element of the case. Obviously, the defendant's guilt or innocence is the most crucial element of all. A criminal defense lawyer must abide by the wishes of the defendant, after consultation, regarding the plea to be entered. MR 1.2(a). The right to determine the plea is part of the client's decision-making authority.

You are prosecuting an armed robbery case against Bonnie and Clyde. You tell a witness to the robbery, Matilda Snitch, not to talk to defense counsel. Matilda is the nervous type who folds under pressure, and you're afraid that talking with defense counsel will make her ineffective as a witness. Is your conduct proper?

No—your conduct will subject you to discipline. A prosecutor cannot prevent defense counsel from interviewing government witnesses; nor can he tell witnesses not to cooperate with defense counsel. Gregory v. U.S., 369 F.2d 185 (D.C. Cir. 1966). Defense counsel is free to interview government witnesses without the prosecutor's consent.

After their experience in Oz, Dorothy, Tin Man, Scarecrow, and Cowardly Lion garner a great deal of publicity. However, the glare of publicity destroys their friendship and things get nasty. One day, Dorothy approaches you to bring a defamation claim against Scarecrow for publicly stating that "Dorothy slept with the Wizard for a free ride back to Kansas." Dorothy has made a mint on book and TV movie deals since her return to Kansas. She tells you in your first meeting, "It's certainly not the money I'm after; it's Scarecrow. I want to make him spend his life savings defending this suit, even if I don't get a cent." You investigate the facts a little bit and determine that Dorothy's defamation claim is substantively valid. If you take the case, will you be subject to discipline?

Probably not. Under MR 3.1, a lawyer shall not bring (or defend) a proceeding unless there is a non-frivolous basis for doing so. You have determined that Dorothy's claim is substantially valid. According to MR 3.1, Comment [2], a lawyer is required to inform herself about the facts of the case and the applicable law and determine that she can make a good faith argument in support of the client's position. That being the case, the client's motivation in bringing the action is immaterial.

An attorney was representing a client in a criminal matter. At the bail hearing, the prosecutor told the court that the defendant was a flight risk, and he asked the court either to confine the defendant until trial or to set bail at $15,000. When it was the attorney's turn to speak, he assured the judge that the client had a medical condition that would prevent him from leaving the area, and that the client did not intend to flee the jurisdiction, but was confident that he could stand trial and clear his name of all charges. The attorney knew, however, that the client already had plane tickets to Venezuela, a non-extradition country, and that the client had already fully recovered from his serious medical condition. Is the attorney subject to discipline for making these statements to the court? A. Yes, because there is no constitutional right to have bail in state court. B. Yes, because a lawyer may not knowingly make a false statement of fact or law to a tribunal. C. No, because the statements made at a bail hearing would not affect the merits or outcome of the case. D. No, because the lawyer does not know with certainty that the client will actually flee the jurisdiction, and he cannot say with medical certainty that the client's medical condition will not relapse.

Rule 3.3(a)(1) says that a lawyer must not knowingly "make a false statement of fact or law to a tribunal or fail to correct a false statement of material fact or law previously made to the tribunal by the lawyer." This rule applies in bail hearings, sentencing hearings, and administrative hearings, in addition to the normal trial context. Note that Comment 8 to Rule 3.3 clarifies this to actual knowledge: "The prohibition against offering false evidence only applies if the lawyer knows that the evidence is false. A lawyer's reasonable belief that evidence is false does not preclude its presentation to the trier of fact. A lawyer's knowledge that evidence is false, however, can be inferred from the circumstances." (See also Rule 1.0(f).) In this problem, the lawyer has actual knowledge that the client is planning to flee the jurisdiction in order to avoid trial and punishment.

A witness testified on a client's behalf at trial. That evening, when the attorney was reviewing exhibits and documents to prepare for the next day of trial, he noticed a document that completely negated the witness' testimony from earlier that day. The testimony was material evidence in the case. The witnesses left the jurisdiction after his testimony concluded, and he is no longer available to correct the false statements. The opposing party's lawyer waived his opportunity to cross-examine the witness, because the testimony was unfavorable to his side and he was eager to move on to a more favorable witness. Does the attorney have a duty to take remedial measures to correct the false testimony, such as disclosing the falsehood to the court? A. Yes, because no proper cross-examination occurred, which violated the other party's constitutional rights. B. Yes, because if a witness called by the lawyer has offered material evidence, and the lawyer comes to know of its falsity, the lawyer shall take reasonable remedial measures, including, if necessary, disclosure to the tribunal. C. No, if a witness called by the lawyer has offered material evidence and the lawyer comes to know of its falsity, the lawyer has no duty to correct the information if the opposing counsel waived his right to cross-examination. D. No, because the lawyer did not realize at the time of the testimony that it was false, and therefore did not knowingly offer any false statements to the tribunal.

Rule 3.3(a)(3) says that when "a witness called by the lawyer, has offered material evidence and the lawyer comes to know of its falsity, the lawyer shall take reasonable remedial measures, including, if necessary, disclosure to the tribunal." Of course, the first recourse should be to try to get the witness to correct the false statement and tell the truth, but they are not always willing to do so. Even though the lawyer has a general duty of loyalty to the client, which includes a duty of zealous advocacy, the duty of candor to the court overrides it, out of concerns for the integrity of the legal system. This problem illustrates that the duty to correct falsehoods also applies when a lawyer "comes to know" of the falsity of the statement only after it is already in evidence.

Prosecutor Steve has a very strong case against Defendant, with excellent evidence and accurate, reliable witnesses. Steve believes his case would be bolstered just a little bit more ("the last nail in the coffin," in Steve's words) with testimony from Defendant's former attorney concerning things that fell outside the attorney-client privilege. May Steve subpoena the defendant's former lawyer?

Steve should not subpoena the lawyer in this case. Generally, when such testimony is not a necessity to the case (as here), a prosecutor should if at all possible avoid calling a former attorney to testify. MR 3.8(e).

At what point does someone become your "client" for purposes of conflict of interest analysis?

The Model Rules are silent on this issue. Under general common law principles, a lawyer-client relationship is formed when either (1) you agree to represent a person or organization; or (2) it becomes reasonable for a client to believe that you owe him a duty of loyalty or confidentiality.

Lawyer discovers that Client has used Lawyer's services to prepare and distribute a private placement memorandum for a securities offering which contains materially misleading financial information. Which of the following is a correct statement? A. Lawyer may withdraw from the representation of Client, but need not do so. B. Lawyer must withdraw from the representation of Client. C. Lawyer may withdraw only if withdrawal will not result in prejudice to Client. D. Lawyer may not withdraw from representing Client without court permission. System Font16px

The answer is (A). See Rule 1.16(b)(3). (B) is arguably not the answer because withdrawal is mandatory only if the violation of the rules or other law is in the future. Read Rule 1.16(a)(1) carefully. Although it is beyond the scope of many professional responsibility courses, you should know that, in some circumstances and under some theories of fraud, a client's past fraud may have continuing effect, and a lawyer's failure to withdraw might be tantamount to assisting the fraud. There is a famous case from the 1980s called OPM, in which a law firm's failure to withdraw previous false statements made in opinion letters was deemed to have aided and abetted the client's continuing fraud on several lenders. If you answered (B) for this reason, good for you, but you may not have to incorporate this much understanding of the general law of fraud into your understanding of professional responsibility. (At least for the purposes of this class; in the real world, failure to think through the intersection between the law of fraud and the duties of lawyers can have disastrous consequences.) (C) is more clearly not the answer, but make sure you see why: Rule 1.16(b)(1) and (b)(3) are alternatives. The lawyer may withdraw if it can be accomplished without material adverse effect on the client (that is, "prejudice") or the client has used the lawyer's services to perpetrate a crime or fraud. (D) is not the answer because the requirement to seek court permission applies only where the lawyer is representing the client in litigation.

There are two things that an applicant for bar admission, or a lawyer in connection with a bar admission, is specifically forbidden to do. What are they?

The applicant or lawyer may not: (1) "knowingly make a false statement of material fact"; or (2) "fail to disclose a fact necessary to correct a misapprehension known by the person to have arisen in the matter, or knowingly fail to respond to a lawful demand for information from an admission or disciplinary authority[.]"

Oliver Twist, indigent, is arrested on theft charges. The court appoints you as Twist's lawyer. May Twist discharge you?

This isn't controlled by the Model Rules, but rather by state statutes, which vary. Those that permit the client to discharge his lawyer also provide that the lawyer should explain to the client the legal consequences that may follow his dismissal. In fact, some states provide that when the dismissal is unjustified, the client may be forced to represent himself—he's not entitled to new counsel. MR 1.16, Comment [5].

Jury selection is underway in a capital murder case. Defendant is accused of committing a heinous murder, and Prosecutor is determined to see that Defendant is convicted. Judge asks the jurors if they are related to any of the witnesses listed to testify and if any of them have ever been represented by any of the lawyers in the case. The jurors all answer "no" to these two questions. In fact, before their divorce five years ago, Juror Jane was married to the deputy sheriff who will testify for the state in the case. And Prosecutor, when he was formerly in private practice, was her lawyer in the divorce matter. Prosecutor is concerned that Jane may have bad feelings toward the deputy sheriff, and asked him how he and Jane get along. Once assured that they have a fine relationship, Prosecutor decides to say nothing and the case proceeds. Is Prosecutor subject to discipline?

Yes, Prosecutor had a duty to disclose both the juror's false answer to the question and his own knowledge of the relationship between the juror and the witness. Any lawyer, especially a prosecutor, and especially in a capital case, has a duty to inform the court of juror misconduct. Here, the prosecutor must know that he represented the potential juror and she has answered that none of the lawyers ever represented her. His duty is all the more clear because he is the very lawyer involved in her false or mistaken answer, and because he made sure that her relationship with his witness would not prejudice his side. Williams v. Taylor, 181 F. Supp. 2d 604 (E.D. Va. 2002).

Steve Austin, nicknamed the "$6 million dollar man," sues his doctor, Rudy Wells, for medical malpractice. You represent Dr. Wells. Wells agrees to pay $6,000 for the representation, payable in monthly installments of $2,000 each for the first three months of representation. Wells pays the first month, and not the next two. Two months later—a week before the trial is to begin—you petition the court to withdraw from the case due to Wells's failure to abide by the fee schedule (after warning Wells that you would do so). Is the court likely to grant your request to withdraw?

Yes, probably. Since a lawsuit has been filed, the court has authority to order you to continue the representation, even if there is good cause to withdraw. See MR 1.16(c) ("When ordered to do so by a tribunal, a lawyer shall continue representation notwithstanding good cause for terminating the representation."). So the question is whether the court will use its discretion to issue such an order. MR 1.16(b)(5) recognizes as a grounds for withdrawal (even where withdrawal would have a material adverse effect on the client) the fact that the client has failed "substantially to fulfill an obligation to the lawyer" under the lawyer-client agreement and has been warned that such a failure will result in withdrawal. Since Wells has breached the payment terms of the contract, and he has been warned that withdrawal will occur as a result, withdrawal should be permitted.

You are a district attorney working the 4 p.m.-to-midnight shift in Big City. At 11 p.m. you get a call from Sergeant Friday at the Midcity police station, asking you to help file felony charges against Oliver Twist, charged with burglary. You go to the station and interview Twist. You tell Twist of his Miranda rights and you stress his right to counsel. Twist says he'll call his attorney in the morning. To save time, you ask Twist if he wants to waive his right to a preliminary hearing. Does asking this question subject you to discipline?

Yes. "A prosecutor may not seek to obtain from an unrepresented accused a waiver of important pre-trial rights, such as the right to a preliminary hearing." MR 3.8(c). Twist has conveyed to you his intent to call his lawyer and you may not thereafter request a waiver of any of his rights.

Lurch has plastic surgery performed at the Remakem Plastic Surgery Clinic. He is unhappy with the results, although objectively they are just fine and proving malpractice would be next to impossible. Lurch asks you to represent him in a claim against Remakem. You have found on numerous occasions that Remakem abhors the publicity of litigation and will often settle unmeritorious suits just to avoid the newspapers. You agree to take Lurch's case and file suit against Remakem. Are you subject to discipline?

Yes. A lawyer must be able to make a "good faith argument" in support of her client's position and on the merits of the action taken. MR 3.1, Comment [2].

Cinderella finds a copy of her adoptive mother's new will. She is furious to discover that her mother hasn't included her in it; she's left the entire estate to her two evil sisters. Since her mother is wealthy, Cinderella is desperate to be named in her will. She hocks her glass slippers to get together enough money to hire a lawyer. She asks you to file a claim having her mother certified incompetent so that the will can be invalidated and Cinderella can inherit a share of her mother's estate via intestacy. You've met Cinderella's mother socially and know that she's relatively normal. Nonetheless, Cinderella is insistent, so you file the claim seeking commitment. Are you subject to discipline?

Yes. A lawyer must not bring a proceeding unless there is a non-frivolous basis for doing so under current law. (A "good faith argument" for changing current law counts as a non-frivolous basis.) MR 3.1, 1.16(a)(1). Since Cinderella's mother is clearly competent, there's no non-frivolous basis for having her committed, so you've violated the rules by bringing the claim (even if you then withdraw it).

You are prosecuting an indictment that alleges that Sleepy, Dopey, and Doc conspired to kidnap Snow White. Each is represented by separate counsel. Doc has agreed to cooperate as a government witness against Sleepy and Dopey in exchange for a reduced charge. Sleepy's attorney asks you whether or not Doc has signed an agreement to cooperate. You need Doc's testimony to nail Sleepy, so you don't want to disclose any facts that might impeach Doc. You refuse to answer. Are you subject to discipline for your silence?

Yes. A prosecutor has a duty to disclose any evidence that could impeach a government witness, as well as any exculpatory evidence. Brady v. Maryland, 373 U.S. 83 (1963); MR 3.8(d). You should have disclosed the existence of Doc's agreement to cooperate.

You are a public prosecutor. You are investigating a case against Tweedle Dee, who is charged with armed robbery. You learn that Tweedle Dee has an identical twin from whom he was separated at birth, and that it's possible the twin, and not Tweedle Dee, was present at the scene of the crime. Must you disclose this to Tweedle Dee's lawyer?

Yes. A prosecutor's duty is to seek justice, not convictions. Pursuant to this duty, a prosecutor must make timely disclosure to defense counsel of any evidence tending to negate the defendant's guilt, mitigate the offense, or call for reduction in the appropriate punishment at sentencing. MR 3.8(d).

Prosecutor Scott is involved in the investigation of a drug smuggler. The investigator reports on several possible leads. Two are promising: a report of the phone calls the Defendant made in the last month and a source that can relate Defendant's recent travels. Other leads look less helpful: Several witnesses intimated that others were involved in the crime and minimized Defendant's role. If Scott directs the investigator to look at the phone records and travel logs, but not follow up on the witness information, has he violated any ethical duty?

Yes. If Scott pursues all of his favorable leads without looking into possible exculpatory evidence, he will violate an ethical obligation. Prosecutors must investigate leads evenhandedly, without regard to the possible help or hindrance they might become in the case.

Herman and Lily Munster are married. Lily concludes that Herman is a monster after all, so she asks for a divorce. To avoid a bitter property dispute, Lily offers to surrender her claim to $7 million of stock that Herman owns, if he will leave it all in a valid will to their "minor son," ten-year-old Eddie. Herman agrees and the document is executed. Ten years later, Herman decides he has no further obligation to observe the will and sells the stock. Lily sues him for breaching the agreement. Herman hires you to represent him. You do extensive research and conclude that the provision was effective only so long as Eddie was a "minor" and no longer applies. Herman asks you if the argument will succeed. You say, "It's a novel argument and I haven't found any cases to support it, but it's certainly a valid and sound argument." Herman tells you to pursue the case as far as needed. Should you make the argument?

Yes. In the interest of serving your client competently and diligently, it's your duty and responsibility to make every reasonable argument you can muster. On these facts, a reasonable argument can be made that the parties' intent was only to protect the child during his infancy, not to prevent Herman from disposing of the stock when the child became old enough to provide for himself. MR 1.1, 3.1.

Businessman has been charged with fraud and money laundering. In light of this, the authorities have frozen his financial accounts. Prior to the freezing of his accounts, Businessman hired a criminal defense attorney. The criminal defense attorney has been preparing for trial for over a year, conducting investigation and finding expert witnesses. After a year of preparation, the criminal defense attorney has still not received any payment from Businessman. He wants to withdraw from the case, and informs the court of his decision. However, the judge tells the criminal defense attorney that he cannot quit, even though he has yet to be paid. Is the judge's action permissible under the Model Rules?

Yes. MR 1.16(b) specifically authorizes permissive withdrawal in cases where the client has failed to pay the lawyer, after given reasonable warning the lawyer will withdraw if the client doesn't pay. However, in this case litigation is pending before the court, and therefore the court's permission is generally required. A court likely will deny permissive withdrawal where interests of those other than the client will be prejudiced or where withdrawal will impede the administration of justice. Here, if Businessman had to find new counsel, trial preparation would have to start all over, possibly delaying the trial for another year. It is in the court's power to deny permissive withdrawals if it would impede the administration of justice, as it would in this case.

You are prosecuting a cattle rustling case against Hoss Cartwright. You come across exculpatory evidence—Mr. Ed tells you that he remembers seeing Little Joe standing in the corral where the horses were kept at the time of the crime. If you don't disclose this evidence to the defense, are you subject to discipline?

Yes. Prosecutors must make timely disclosure to the defense of all evidence known to them "that tends to negate the guilt of the accused or mitigates the offense." MR 3.8(d). At sentencing, they must disclose all unprivileged mitigating circumstances known to them.

You enter into a retainer agreement with the Alexander T. Great, a management consulting firm. Under the agreement, you agree to furnish all of Alexander T. Great's non-litigation legal services for $1,000 per month, a reasonable fee. Litigation services are to be negotiated as necessary. You send a bill on the first of each month, payable on receipt. After doing so faithfully for four months, you find you have not yet been paid for even one month's services. Disgusted, you withdraw from representation (in the middle of a time-sensitive negotiation you're handling for the firm) without notifying anyone at Alexander T. Great about the delinquency. Under the Model Rules, are you subject to discipline?

Yes. The existence of the time-sensitive negotiation means that your withdrawal would have a "material adverse effect" on the firm. (If such an effect wouldn't occur, you could withdraw for any or no reason, [MR 1.16(b)] though you'd still have to comply with reasonable-notice provisions.) Where withdrawal would cause a material adverse effect on the client, the Model Rules allow withdrawal only under limited circumstances. The only such circumstance that even arguably applies on these facts is that withdrawal is allowed if "the client fails substantially to fulfill an obligation to the lawyer regarding the lawyer's services and has been given reasonable warning that the lawyer will withdraw unless the obligation is fulfilled." MR 1.16(b)(5). Since you didn't give Alexander T. Great advance warning that you'd withdraw if the firm didn't pay, you don't qualify for this exception. Also, MR 1.16(d) says that upon "termination of representation," you must "take steps to the extent reasonably practicable to protect a client's interests, such as giving reasonable notice to the client [or] allowing time for employment of other counsel[.]" Here, by withdrawing so precipitously in the middle of a negotiation, you have not taken the steps required to protect the firm, so you'd be subject to discipline for that, as well.

You represent the Smyle & Behappy Medical Clinic. Surrogate motherhood is recognized as valid in the jurisdiction. The clinic wants to get in on the action and asks you to formulate a contract covering all aspects of the relationship between the surrogate mother and the couple who are to raise the baby. You find the whole concept of surrogate motherhood personally repugnant. Smyle & Behappy will be materially prejudiced if you withdraw from the representation. May you withdraw under the Model Rules?

Yes. Under MR 1.16(b)(4), withdrawal is permissive when the client insists that the lawyer pursue lawful objectives the lawyer finds "repugnant." That's the case here, so you can withdraw despite the harmful effect withdrawal will have on Smyle & Behappy.

You represent Anna Freud, a wealthy widow, and have done so for many years. One day she storms into your office and tells you to disinherit her good-for-nothing son, Sigmund. He forgot Mother's Day yet again and obviously has no appreciation of his mother's importance. Anna and Sigmund have had a hot-and-cold relationship for a long time. While there is nothing illegal about Anna's request, you have heard this kind of thing from her before. You believe the issue will blow over, as it always has in the past, and that she will be reconciled with Sigmund shortly. If you follow her wishes, you believe it will only lead to bitterness between her and Sigmund, and you believe this wouldn't be in Anna's best interests. If you tell her your opinion and advise her to leave Sigmund in her will, are you acting properly?

Yes. Under MR 2.1, a lawyer "shall exercise independent professional judgment and render candid advice." The duty to give advice includes the right to refer to extra-legal considerations, like moral ones, costs, or effects on other people that could bear on the client's situation. Id. Thus, your advice is proper.

You have been appointed by the court to represent Oren "Tang" Simpleton, a man accused of bludgeoning someone to death with a pair of Bruno Magli shoes. In your pre-trial meetings with Simpleton, he tells you the case should be easy to win. "All you have to do is put me on the stand and let me say that I buy all my shoes at PayLess [a very down-scale, non-designer shoe chain]. I've always paid for my shoes with cash and I've already thrown out all my designer shoes so there shouldn't be any evidence to contradict me." You tell him you do not want to put him on the stand under those circumstances, but he insists. Can you withdraw from the case?

Yes. You know that Simpleton intends to perjure himself on the stand. You cannot offer his testimony under these circumstances without violating the rules of professional responsibility. See MR 3.3(a)(3) (a lawyer may not offer evidence that she knows to be false). If Simpleton cannot be talked out of his plan to give perjurious testimony, you not only may but must withdraw. See MR 1.16(a)(1) (withdrawal required if "the representation will result in violation of the rules of professional conduct or other law."). The facts state, however, that you are appointed counsel. You must therefore get court permission to withdraw. You cannot disclose Simpleton's confidence by informing the court of his intentions. Your request for permission to withdraw must be worded carefully and generally—e.g., "Professional considerations require that I terminate my representation."

You represent Leona Helmfeed, the richest woman in world. Leona asks you to assert a claim against the federal government challenging the constitutionality of the personal income tax, an issue that has been resolved by the Supreme Court and confirmed in many of its decisions. You explain to her that there are absolutely no grounds for filing the suit. Leona warns you that if you don't file the suit, you're yesterday's lawyer. Your whole practice would collapse without Leona, so you file the suit after requiring Leona to sign a letter acknowledging your advice and instructing you to sue. Are you subject to discipline?

Yes. You may not bring an action that has no reasonable basis in law or fact. It's true that the Supreme Court does reverse itself on rare occasions and that a lawyer may assert an argument for extending, modifying, or reversing existing law. MR 3.1. (Witness the argument against Miranda in the Court's 2000 term.) But, it's also true that some legal principles are so firmly established that they cannot be challenged seriously or in good faith. The personal income tax is so irrevocably and firmly a cornerstone of our federal government that it is transparently frivolous to argue against it.

You're the attorney for Nickel & Dime Bank, whose slogan is: "You've got a friend." The bank's president, Vainen Pompous, asks you to examine some old mortgage files. In examining the mortgages, some of which are still outstanding, you find that there is a provision allowing the bank to change the interest rate every five years. Since many of these mortgages were financed at 3%, you research the issue and know that they legally could be raised by almost 300%. You reasonably believe that neither Pompous nor any other bank officer has realized this. However, raising mortgage interest by 300% all at once would undoubtedly create a public outcry against Nickel & Dime Bank. Should you tell Pompous about the interest provision?

Yes; in fact, you must do so. Under MR 1.4(b), a lawyer "shall explain a matter to the extent reasonably necessary to permit the client to make informed decisions." Since the bank has the legal right to take advantage of what you now know to be in the documents, the bank's officers need to be told the underlying facts in order to decide whether to exercise that legal right—this will be a business decision, not a legal decision, and it therefore falls within the client's purview. However, in addition, in your role as advisor, you should (though aren't required to) tell Pompous about the non-legal ramifications of suddenly and dramatically raising the interest rates—namely, the adverse impact on the bank's image. Thus, MR 2.1 says that "[i]n rendering advice, a lawyer may refer not only to law but to other considerations such as moral, economic, social and political factors that may be relevant to the client's situation."


Ensembles d'études connexes

Independent living chapter 9 review

View Set

Module 15: Life Development - Chapter 9 Quiz

View Set

Chemistry Semester 2 Test 3 Mini Test

View Set

Female & Male Reproductive problems DYNAMIC ATI

View Set